Number Systems 01 | Harbourpress

102
1 1 MATHEMATICS - 9 DDITION L A A PR CTICE A Worksheet-1 Section A 1. Rationalising factor of 3 2 + is ( ) 3 2 - 2. ( ) 5 3 2 + = ( ) ( ) 5 3 2 5 3 2 2 + + = 5 + 3 + 2 15 = 8 + 2 15 2 4 15 = + ( ) 3. ( ) ( ) 3 5 3 5 + - = (3) 2 – ( ) 5 2 = 9 – 5 = 4 Therefore ( ) ( ) 3 5 3 5 + - is a rational number. 4. An irrational number between 2 3 and ( ) 2 2 = 2 ( ) 3 2 = 3 An irrational number between 2 3 and is 1.5. 5. ( ) 64 1 2 1 3 2 - - = ( ) () 8 8 2 1 2 1 3 2 2 1 2 1 3 2 - - Γ—- Γ—- Γ— = = () (( ) ) 8 2 2 3 3 2 3 = = () 2 3 2 3 Γ— = 2 2 = 4 6. Solve the equation for x: 4 2x = 1 32 Sol. Given 4 2x = 1 32 β‡’ (2 2 ) 2x = 1 2 5 β‡’ 2 4x = 2 –5 Compairing the power of 2 β‡’ 4x = –5 β‡’ x = – 5/4 7. 32 48 8 12 + + = 2 2 2 2 2 2 2 2 2 3 2 2 2 3 Γ— Γ— Γ— Γ— + Γ— Γ— Γ— Γ— + = 4 2 4 3 2 2 2 3 4 2 3 2 2 3 2 2 3 2 3 + + = + + = + + ( ) ( ) ( ) ( ) = 2 8. 64 125 2 3 - = 4 5 4 5 3 3 2 3 3 2 3 = - - = 4 5 4 5 3 2 3 2 = Γ— - - = 5 4 25 16 2 = 9. 32 243 45 / = 2 2 2 2 2 3 3 3 3 3 45 Γ— Γ— Γ— Γ— Γ— Γ— Γ— Γ— / = 2 3 5 45 / Number Systems 01 Chapter = 2 3 5 4 5 Γ— = 2 3 4 10. Rationalising factor of 3 7 - is 3 7 + = ( ) ( ) 3 7 3 7 - + = (3) 2 – ( ) 7 2 [∡ a 2 – b 2 = (a + b) (a – b)] = 9 – 7 = 2 Section B 11. Express: 23 47 . in the form of p/q. Let x = 23 47 . ...(i) Multiplying both sides by 100, we get 100x = 2347 47 . ...(ii) Subtracting (i) and (ii), we get 100x – x = 2347 47 23 47 . . - 99x = 2324 x = 2324 99 23 47 . = 2324 99 12. 5 7 11 13 and R 1 = 1 2 5 7 11 13 + = 1 2 65 77 91 + = 1 2 142 91 Γ— R 1 = 71 91 R 2 = 1 2 5 7 71 91 + R 2 = 1 2 65 71 91 + R 2 = 1 2 136 91 R 2 = 68 91 13. 64 2x – 5 = 4 Γ— 8 x – 5 2 6(2x – 5) = 2 2 Γ— 2 3(x – 5) β‡’ 2 12x – 30 = 2 3x – 13 β‡’ 12x – 30 = 3x – 13 9x = 17 x = 17 9 14. If 2 = 1.414 1 3 2 + Rationalising the denominator. = 1 3 2 3 2 3 2 + Γ— - -

Transcript of Number Systems 01 | Harbourpress

1 1

MATHEMATICS - 9DDITION LA APR CTICEA

Worksheet-1

Section A 1. Rationalising factor of 3 2+ is ( )3 2βˆ’

2. ( )5 3 2+ = ( ) ( )5 3 2 5 32 2+ +

= 5 + 3 + 2 15

= 8 + 2 15 2 4 15= +( )

3. ( ) ( )3 5 3 5+ βˆ’ = (3)2 – ( )5 2

= 9 – 5 = 4

Therefore ( ) ( )3 5 3 5+ βˆ’ is a rational number.

4. An irrational number between 2 3and

( )2 2 = 2

( )3 2 = 3

An irrational number between 2 3and is 1.5.

5. ( )6412

13

2

βˆ’βˆ’

= ( ) ( )8 8212

13

2

212

13

2βˆ’βˆ’

Γ— βˆ’ Γ— βˆ’ Γ—

=

= ( ) (( ) )8 223 3

23= = ( )2

323

Γ— = 22 = 4

6. Solve the equation for x:

42x = 132

Sol. Given 42x = 132

β‡’ (22)2x = 12

5

β‡’ 24x = 2–5 Compairing the power of 2β‡’ 4x = –5β‡’ x = – 5/4

7. 32 488 12

++

= 2 2 2 2 2 2 2 2 2 3

2 2 2 3

Γ— Γ— Γ— Γ— + Γ— Γ— Γ— Γ—+

= 4 2 4 32 2 2 3

4 2 32 2 3

2 2 32 3

++

= ++

= ++

( )( )

( )( )

= 2

8. 64125

23

βˆ’

= 45

45

3

3

23 3

23

=

βˆ’ βˆ’

= 45

45

323

2

=

Γ— βˆ’ βˆ’

= 54

2516

2

=

9. 32243

4 5

/

= 2 2 2 2 23 3 3 3 3

4 5Γ— Γ— Γ— Γ—Γ— Γ— Γ— Γ—

/

= 23

5 4 5

/

Number Systems01Chapter

= 23

545

Γ—

= 23

4

10. Rationalising factor of 3 7βˆ’ is 3 7+

= ( ) ( )3 7 3 7βˆ’ +

= (3)2 – ( )7 2 [∡ a2 – b2 = (a + b) (a – b)]

= 9 – 7 = 2

Section B 11. Express: 23 47. in the form of p/q.

Let x = 23 47. ...(i)Multiplying both sides by 100, we get

100x = 2347 47. ...(ii)Subtracting (i) and (ii), we get

100x – x = 2347 47 23 47. .βˆ’ 99x = 2324

x = 232499

23 47. = 232499

12. 57

1113

and

R1 = 12

57

1113

+

= 12

65 7791+

= 12

14291

Γ—

R1 = 7191

R2 = 12

57

7191

+

R2 = 12

65 7191+

R2 = 12

13691

R2 = 6891

13. 642x – 5 = 4 Γ— 8x – 5

26(2x – 5) = 22 Γ— 23(x – 5)

β‡’ 212x – 30 = 23x – 13

β‡’ 12x – 30 = 3x – 13 9x = 17

x = 179

14. If 2 = 1.414

13 2+

Rationalising the denominator.

= 1

3 23 23 2+

Γ— βˆ’βˆ’

2

MATHEMATICS - 9DDITION LA APR CTICEA

= 3 2

3 23 29 2

3 272 2

βˆ’βˆ’

= βˆ’βˆ’

= βˆ’( ) ( )

= 3 1 414

71 5867

βˆ’ =. . = 0.2266

15. (343)2/n = 49

73

2Γ—n = 72

76/n = 72

6/n = 2

62

= n

n = 3

16. a. 3 126 27

3 2 2 3

6 3 3 3=

Γ— Γ—Γ— Γ—

= 3 2 36 3 3

6 318 3

13

Γ—Γ—

= =

b. 8 3 2 3 4 3βˆ’ +

= 3 (8 – 2 + 4) = 3 (10)

= 10 3

17. By Pythagorus theorem:

AC2 = AB2 + BC2

AC2 = 12 + 12

AC2 = 2

AC = 2 β‡’ AD = 2

[Since radii of same circle are equal]

18. ( ) ( )2 3 5 22 2+ + βˆ’

= ( ) ( ) ( ) ( )2 3 2 2 3 5 2 2 5 22 2 2 2+ + + + βˆ’

= 2 + 3 + 2 6 + 5 + 2 – 2 10

= 12 + 2 6 2 10βˆ’

= 2 ( )6 6 10+ βˆ’

19. If x = 6 2+

x – 1x

= 6 21

16 2

+ βˆ’+

= ( ) .6 2 16 2

36 2 2 6 2 16 2

2+ βˆ’+

= + + βˆ’+

= 37 12 26 2++

Section C

20. Rationalise the denominator of 3 25 2

++

3 25 2

++

Γ— 5 25 2

βˆ’βˆ’

On Rationalising the denominator.

= 3 5 2 2 5 2

5 22 2( ) ( )

( ) ( )βˆ’ + βˆ’

βˆ’

= 5 3 6 5 2 4

25 2βˆ’ + βˆ’

βˆ’

= 5 3 6 5 2 2

23βˆ’ + βˆ’

= 5 3 6 5 2 2

23βˆ’ + βˆ’

21. ( )1 2 33 3 312+ + = ( )1 8 27

12+ +

= ( )3612 (am)n = amn

= ( )6212

= ( )62

12

Γ— = 6

22. 4 54 5

4 54 5

+βˆ’

+ βˆ’+

Rationalising the denominator.

= 4 54 5

4 54 5

+βˆ’

Γ— ++

+ 4 54 5

4 54 5

βˆ’+

Γ— βˆ’βˆ’

= ( )( ) ( )

( )( ) ( )

4 54 5

4 54 5

2

2 2

2

2 2+βˆ’

+ βˆ’βˆ’

= 16 5 2 4 5

16 516 5 8 5

16 5+ +

βˆ’+ + βˆ’

βˆ’. .

= 21 8 5

1121 8 5

11+ + βˆ’

= 21 8 5 21 8 511

+ + βˆ’

= 4211

23.

24. LHS = x

x

a

b

a b2

2

1

+

x

x

b

c

b c2

2

1

+ x

x

c

a

c a2

2

1

+

= ( )x a b a b2 21

βˆ’ +( ) x xb c b c c a c a2 2 2 21 1

βˆ’ + βˆ’ +( ) ( ) = ( )

( ).( )x

a ba b

2 2 1βˆ’+ ( ) ( )

( ). ( )x x

b cb c

c ac a

2 2 2 21 1βˆ’+

βˆ’+

= xa b a b

a b( )( )

( )+ βˆ’

+ . ( )( )( )

( )xb c b c

b c+ βˆ’

+ . ( )( )( )

( )xc a c a

c a+ βˆ’

+

= (x)a – b . (x)b – c. (x)c – a = (x)a – b + b – c + c – a

= x0

L.H.S = 1 = R.H.SHence Proved.

3 3

01. Nu

mb

er System

s

MATHEMATICS - 9DDITION LA APR CTICEA

25. 23

32

2

x x

. = 8116

23

23

2

βˆ’x x

. = 32

4

23

2

βˆ’x x

= 23

4

βˆ’

Comparing the power of (2/3), we get x – 2x = – 4 –x = – 4 x = 4

26. 3 45 125 200 50βˆ’ + βˆ’ = 3 3 3 5 5 5 5Γ— Γ— βˆ’ Γ— Γ—

+ 5 5 2 2 2 5 5 2Γ— Γ— Γ— Γ— βˆ’ Γ— Γ—

= 3 3 5 5 5 10 2 5 2Γ— βˆ’ + βˆ’

= 9 5 5 5 5 2βˆ’ +

= 4 5 5 2+

27. 8116

259

25

3 4 3 2 3

Γ—

Γ—

βˆ’ βˆ’ βˆ’/ /

= 3 3 3 32 2 2 2

34Γ— Γ— Γ—

Γ— Γ— Γ—

βˆ’

Γ— 5 53 3

32Γ—

Γ—

βˆ’

Γ— 25

3

βˆ’

= 32

53

25

434 2

32 3

Γ—

Γ—

βˆ’ βˆ’βˆ’

= 32

53

32

434

232

3

Γ—

Γ—

Γ— βˆ’ Γ— βˆ’

= 32

53

32

3 3 3

Γ—

Γ—

βˆ’ βˆ’

= 32

32

53

3 3 3

Γ—

Γ—

βˆ’ βˆ’

= 32

35

3 3 3

Γ—

βˆ’ +

= 32

35

0 3

Γ—

= 127125

Γ—

= 27125

28. x = 9 4 5+

β‡’ 1x

= 1

9 4 5+

β‡’ 1x

= 1

9 4 59 4 59 4 5+

Γ— βˆ’βˆ’

β‡’ 1x

= 9 4 5

9 4 59 4 581 802 2

βˆ’βˆ’

= βˆ’βˆ’( ) ( )

= 9 – 4 5

Now, xx

βˆ’

12

= ( ) .x xx x

22

21 1βˆ’ +

β‡’ xx

βˆ’

12

= x – 2 + 1x

β‡’ xx

βˆ’

12

= 9 + 4 5 – 2 + 9 – 4 5

β‡’ xx

βˆ’

12

= 16

∴ xx

βˆ’ 1 = 16 = 4

(By taking square root of both sides)

Section D 29.

3216

2256

22432 3 3 4 1 5( ) ( ) ( )/ / /βˆ’ + +

= 3

62

42

33 2 3 4 3 4 5 1 5( ) ( ) ( )/ / /βˆ’ + +

= 3

6

2

4

2

33

23

434

515( ) ( ) ( )

Γ— βˆ’ Γ— Γ—+ +

= 36

24

232 3( ) ( )βˆ’ + +

= 3 Γ— 62 + 264

23

+ = 108 + 132

23

+

= 108.70

30. 1

1 21

2 32

3 5++

++

+

= 1 2

1 2 1 22 3

2 3 2 3βˆ’

+ βˆ’+ βˆ’

+ βˆ’( ) ( )( )

( ) ( )

+ 2 3 5

3 5 3 5( )

( ) ( )βˆ’

+ βˆ’

= 1 2

1 22 3

2 32 3 53 52 2 2 2 2 2

βˆ’βˆ’

+ βˆ’βˆ’

+ βˆ’βˆ’( ) ( ) ( )

( )( ) ( )

= 1 21 2

2 32 3

2 3 53 5

βˆ’βˆ’

+ βˆ’βˆ’

+ βˆ’βˆ’

( )

= 1 2

12 31

2 3 52

βˆ’βˆ’

+ βˆ’βˆ’

+ βˆ’βˆ’

( )

= 1 2 2 3

12 3 2 5

2βˆ’ + βˆ’

βˆ’+ βˆ’

βˆ’

= 1 3

12 3 2 5

2βˆ’βˆ’

+ βˆ’βˆ’

= 2 1 31 2

2 3 2 52

( )( )

βˆ’βˆ’ Γ—

+ βˆ’βˆ’

= 2 2 3 2 3 2 5

2βˆ’ + βˆ’

βˆ’

= 2 2 5

2βˆ’βˆ’

= βˆ’ βˆ’( )2 2 5

2

= βˆ’ +2 2 5

2 =

βˆ’ + = βˆ’ +22

2 52

1 5( )

31. x = 2 3+

1x

= 1

2 3+

1x

= 1

2 32 32 3+

Γ— βˆ’βˆ’

β‡’ 1x

= 2 3

2 32 34 3

2 32 2βˆ’βˆ’

= βˆ’βˆ’

= βˆ’( ) ( )

( )

xx

331+ = x

xx

xx

x+

βˆ’ +

13

1 13

. .

= xx

xx

+

βˆ’ +

13

13

4

MATHEMATICS - 9DDITION LA APR CTICEA

= ( ) ( )2 3 2 3 3 2 3 2 33+ + βˆ’ βˆ’ + + βˆ’ = (4)3 – 3 Γ— 4 = 64 – 12 = 52

32. 1

11

1βˆ’+

+x x =

1 11

212 2

+ + βˆ’βˆ’

=βˆ’

x xx x

β‡’ 2

12

12 2βˆ’+

+x x =

2 2 2 21

41

2 2

4 4+ + βˆ’

βˆ’=

βˆ’x x

x x

β‡’ 4

14

14 4βˆ’+

+x x =

4 4 4 41

81

4 4

8 8+ + βˆ’

βˆ’=

βˆ’x x

x x

β‡’ 8

18

18 8βˆ’+

+x x = 8 8 8 8

116

1

8 8

16 16+ + βˆ’

βˆ’=

βˆ’x x

x x

∴ 11

11

21

41

812 4 8βˆ’

++

++

++

++x x x x x

= 16

1 16βˆ’ x

33. x = 3 8+

β‡’ 1x

= 1

3 8+

β‡’ 1x

= 1

3 83 83 8+

Γ— βˆ’βˆ’

β‡’ 1x

= 3 8

3 83 89 8

3 812 2

βˆ’βˆ’

= βˆ’βˆ’

= βˆ’( ) ( )

∴ 1x

= 3 81

3 8βˆ’ = βˆ’

x2 + 12x

= ( ) ( )3 8 3 82 2+ + βˆ’

= (3)2 + 2 Γ— 3 Γ— 8 + ( )8 2

+ 32 – 2Γ— 8 Γ— 3 + ( )8 2

= 9 + 6 8 + 8 + 9 – 6 8 + 8 = 34

34. We have: 1

2 3 5+ + =

12 3 5

2 3 52 3 5( )

( )( )+ +

Γ— + βˆ’+ βˆ’

= ( )

( ) ( )2 3 5

2 3 52 2+ βˆ’

+ βˆ’ =

2 3 52 2 2 3 3 52 2

+ βˆ’+ + βˆ’( ) . . ( )

= 2 3 5

2 2 6 3 52 3 5

2 6+ βˆ’

+ + βˆ’= + βˆ’

= 2 3 5

2 666

+ βˆ’ Γ—

= ( )2 3 5 6

2 6+ βˆ’ Γ—

Γ— =

12 18 3012

+ βˆ’

= 2 2 3 2 30

12+ βˆ’

35. 81x – 2 = 3 Γ— 272x – 3

β‡’ 34x – 8 = 3 Γ— 36x – 9

β‡’ 34x – 8 = 36x – 8

4x – 8 = 6x – 8 2x = 0 x = 0

36. 2 62 3

2 32 3

6 26 3

6 36 3+

Γ— βˆ’βˆ’

++

Γ— βˆ’βˆ’

– 8 36 2

6 26 2+

Γ— βˆ’βˆ’

= 2 6 2 32 3

6 2 6 36 32 2 2 2

( )( ) ( )

( )( ) ( )

βˆ’βˆ’

+ βˆ’βˆ’

– 8 3 6 26 22 2( )

( ) ( )βˆ’

βˆ’

= 2 12 2 18

2 36 12 6 6

6 38 18 8 6

6 2βˆ’βˆ’

+ βˆ’βˆ’

βˆ’ βˆ’βˆ’

= 2 2 3 2 3 21

12 3 6 63

8 3 2 8 64

Γ— βˆ’ Γ—βˆ’

+ βˆ’ βˆ’ Γ— βˆ’

= 4 3 6 2

16 2 3 6

38 3 2 6

4βˆ’βˆ’

+ βˆ’ βˆ’ βˆ’( ) ( )

= 4 3 6 21

2 2 3 6 6 2 2 6βˆ’βˆ’

+ βˆ’ βˆ’ βˆ’( ) ( )

= βˆ’ + + βˆ’ βˆ’ +4 3 6 2 4 3 2 6 6 2 2 6

= 0

37. a = 3 23 2

3 23 2

βˆ’+

Γ— βˆ’βˆ’

a = ( )3 2

3 23 2 2 6

15 2 6

2βˆ’βˆ’

= + βˆ’ = βˆ’

b = 3 23 2

3 23 2

+βˆ’

Γ— ++

= ( )3 2

3 23 2 2 6

15 2 6

2+βˆ’

= + + = +

a2 + b2 – 5ab

= ( ) ( )5 2 6 5 2 6 52 2βˆ’ + + βˆ’ ( ) ( )5 2 6 5 2 6βˆ’ +

= (25 + 24 – 2 Γ— 5 Γ— 2 6 ) + (25 + 24 + 2 Γ— 5 Γ— 2 6 ) –5 (25 – 24)

= 49 – 20 6 + 49 + 20 6 – 5 = 98 – 5 = 93

38. 0.6 + 0 7 0 47. .+ ...(i)

Case I: Let x = 0 7.β‡’ x = 0.777..... ...(ii)β‡’ 10x = 7.77..... ...(iii)Subtracting (ii) from (iii), we get 9x = 7

x = 79

Case II. Let x = 0 47.β‡’ x = 0.4777.....β‡’ 10x = 4.777..... ...(iv)β‡’ 100x = 47.777..... ...(v)Subtracting (iv) from (v), we get 90x = 43

x = 4390

From eqn (i), we get:

0.6 + 0 7 0 47. .+

= 610

79

4390

+ +

= 54 70 43

9016790

+ + =

Worksheet-1I

Section A 1. A rational number between –3/7 and 4/7.

R1 = 12

37

47

Γ— βˆ’ +

R1 = 12

17

114

Γ—

=

2. Rationalising factor of 1

7 4βˆ’ is 7 4+

5 5

01. Nu

mb

er System

s

MATHEMATICS - 9DDITION LA APR CTICEA

3. 11 2βˆ’

= 1

1 21 21 2βˆ’

Γ— ++

= 1 2

1 21 21 2

1 212 2

+βˆ’

= +βˆ’

= +βˆ’( ) )

= 1 1 414

12 414

1+

βˆ’=

βˆ’. .

= – (2.414)

4. – (81)3/4 = – (34)3/4 = – ( )34

34

Γ—

= (– 3)3 = –27 5. (62 + 82)1/2

= ( )36 6412+

= ( )10012

= ( )10012

= [( ) ]10 212

= 102

12

Γ—

= 10 6. (64)1/2 [641/2 + 1]

( )8212 [(82)1/2 + 1]

= ( ) ( )8 8 12

12

212

Γ— Γ—+

= 8 (8 + 1) = 8 Γ— 9 = 72

7. Four number between 12

34

and

R1 = 12

12

34

12

2 34

+

= +

= 12

54

58

=

R2 = 12

34

58

12

6 58

12

118

+

= +

=

R2 = 1116

R3 = 12

1116

58

12

11 1016

+

= +

= 21

2 162132Γ—

=

R4 = 12

1116

2132

+

= 12

22 2132

Γ— +

R4 = 12

4332

4364

Γ— =

∴ Four rational numbers are 581116

2132

4364

, , ,

8. 27 413

12.

= ( ) . ( )3 2313 2

12

= ( ) . ( )3 23

13

212

Γ— Γ—

= 3 Γ— 2 = 6

9. 64125

2 3

βˆ’ /

= 4 4 45 5 5

2 3Γ— Γ—Γ— Γ—

βˆ’ /

= 45

45

45

3 2 3 323

2

=

=

βˆ’ Γ— βˆ’ βˆ’/

= 54

2516

2

=

10. ( )( )

( )

( )

/

/8181

3

3

1 4

1 4

414

414

βˆ’ βˆ’

= = ( )

( )

3

3

33

414

414

1Γ— βˆ’

Γ—

βˆ’=

=

133

= 13

13

19

Γ— =

Section B 11. 5 45

755

Γ·

5 3 3 5Γ— Γ— Γ— 575

= 5 3 55

5 5 315 5

55 3

Γ— Γ—Γ— Γ—

= Γ—

= 35 53

3 53

153

33

. = Γ— = Γ—

= 15 33

5 3=

12. Two rational number between 4 and 5

First rational number = 4 52

92

+ = = 4.5

Second rational number = 4 5 52

9 52

. .+ = = 4.75

Two rational no. are between 4 and 5 = 4.5 and 4.75

13. Find the value of 4

216

1

25623

34( ) ( )

βˆ’

= 4

6

1

4323 4

34( ) ( )

βˆ’

= 46

142 3βˆ’

= 436

164

βˆ’ = 19

164

βˆ’

= 55576

14. 0 47. 0.4777..... can be expressed form p/q.

Let x = 0 47. ...(i)

10x = 4 7. ...(ii)

100x = 47 7. ...(iii)Subtract eqn. (ii) from (iii), we get 90x = 43

x = 4390

= p/q

∴ pq

==

43

90

15. 36 3636

6 66

72

92

5 2

272 2

92

2 5 2βˆ’ = βˆ’

βˆ’

βˆ’

βˆ’

βˆ’( )( ) ( )

( )/ /

= ( ) ( )

( ) /6 6

6

272

292

2 5 2

Γ— Γ— βˆ’

Γ— βˆ’βˆ’

= 6 6

6

7 9

5βˆ’ βˆ’

βˆ’( )

( ) = (67 – (6)–9)65

6

MATHEMATICS - 9DDITION LA APR CTICEA

= 612 – (6)–4 = 612 – 164

= 6 16

16

4βˆ’

16. x = 0 456. in the form of p/q.

Let x = 0 456. ...(1) 1000x = 456.456 ...(2)From (1) and (2) 1000x – x = 456 999x = 456

x = 456999

17. Simplify: 25 343

16 8 7

32

35

54

43

45

Γ—

Γ— Γ— =

( ) ( )

( ) ( )

5 7

2 2 7

232 3

35

454 3

43

45

Γ—

Γ— Γ—

= 5 7

2 2 7

232

335

454

343

45

Γ— Γ—

Γ— Γ—

Γ—

Γ— Γ—

= 5 7

2 2 7

5 72

395

5 445

395

45

5 4Γ—

Γ— Γ—

= Γ—βˆ’

+

= 5 72

3 1

9Γ—

= 125 7512

875512

Γ— =

18. Simplify: [5(251/2 + 161/2]3

= 5 5 4212 2

12

3

(( ) ( ) )+

= [5 Γ— (5 + 4)]3

= (5 Γ— 9)3

= (45)3 = 91125 19.

20. (5)x + 1 = (125)3x – 1

(5)x + 1 = [(53)]3x – 1

(5)x + 1 = (53)3x – 1

(5)x + 1 = (5)9x – 3

Comparing the power of 5, we get x + 1 = 9x – 3 x – 9x = – 3 – 1 – 8x = – 4

x = 48

x = 12

21. Let x = 32 1235. ...(i)

Multiply both side by 100

100 x = 3212 35. ...(ii) Multiply both side by 100

10000 x = 321235 35. ...(iii)Subtract (ii) from (iii) 9900 x = 318023

x = 3180239900

22. 27y = 93y

β‡’ (33)y = 32 Γ— (3)–y

β‡’ (3)3y = 32 – y

Comparing the power of 3, both sides, we get; 3y = 2 – y 3y + y = 2 4y = 2

y = 24

12

= 23. Show that:

x y xyx y xy

βˆ’ βˆ’

βˆ’ βˆ’βˆ’1 1

1 1

1 1

= x y xyx y xy

βˆ’ βˆ’

βˆ’ βˆ’βˆ’1 1

1 1

1

= x yx y

0 0

0 0

1βˆ’ = 1 – 1 = 0

24. ( ) ( )3 5 5 3

7 2 5βˆ’ +

βˆ’

= 15 9 25 15

7 2 53 5

7 2 52

7 2 5+ βˆ’ βˆ’

βˆ’= βˆ’

βˆ’= βˆ’

βˆ’

= βˆ’

βˆ’Γ— +

+2

7 2 57 2 57 2 5

= βˆ’ +

βˆ’= βˆ’ βˆ’

βˆ’ Γ—2 7 2 5

7 2 514 4 549 4 52 2

( )( ) ( )

= βˆ’ βˆ’ = βˆ’ +14 4 5

292 7 2 529

( )

25.

26. 1

1 2 3+ βˆ’

= 1

1 2 31 2 31 2 3+ βˆ’

Γ—+ ++ +

= 1 2 3

1 2 32 2+ +

+ βˆ’( ) ( )

= 1 2 31 2 2 2 3

1 2 32 2

+ ++ + βˆ’

= + +

= 1 2 3

2 222

1 2 3 24

+ + Γ— = + +( )

= 2 6 2

4+ +

7 7

01. Nu

mb

er System

s

MATHEMATICS - 9DDITION LA APR CTICEA

27. Simplify by rationalising the denominator.

5 2 3 518 6

βˆ’+

on rationalising the denominator.

β‡’ 5 2 3 518 6

18 618 6

βˆ’+

Γ— βˆ’βˆ’

= 5 36 5 12 3 90 3 30

18 62 2βˆ’ βˆ’ +

βˆ’( ) ( )

= 5 6 5 2 3 9 10 3 30

18 6Γ— βˆ’ Γ— βˆ’ +

βˆ’

= 30 10 3 9 10 3 30

12βˆ’ βˆ’ +

= 112

30 10 3 9 10 3 30[ ]βˆ’ βˆ’ +

28. x = 1 + 2 2 find the value of x2 +12x

x2 = (1 + 2 2 )2

1x

= 1

1 2 21 2 21 2 2

1 2 21 2 22 2+

Γ— βˆ’βˆ’

= βˆ’βˆ’( ) ( )

1x

= 1 2 21 8

1 2 27

βˆ’βˆ’

= βˆ’βˆ’

12x

= 1 2 2

7

2βˆ’βˆ’

x2 = ( )1 2 2 2+ [(a + b)2 = a2 + b2 + 2ab]

= 1 + 4 Γ— 2 + 4 2 = 1 + 8 + 4 2

= 9 + 4 2

x2 + 12x

= 9 + 4 2 1 2 27

2

+ βˆ’βˆ’

= 9 + 4 21 4 2 4 2

49+ + Γ— βˆ’

= 9 + 4 21 8 4 2

49+ + βˆ’

= 9 + 4 29 4 249

+ βˆ’

Section D

29. Simplify: 3 26 3

2 36 2

4 36 2βˆ’

++

βˆ’βˆ’

= 3 2 6 36 3 6 3

2 3 6 26 2 6 2

( )( ) ( )

( )( ) ( )

+βˆ’ +

+βˆ’

+ βˆ’

βˆ’ +βˆ’ +

4 3 6 26 2 6 2

( )( ) ( )

= 3 2 6 33

2 3 6 22

( ) ( )+ + βˆ’ βˆ’ +4 3 6 24

( )

= 2 6 3 3 6 2 3 6 2( ) ( ) ( )+ + βˆ’ βˆ’ +

= 12 6 18 2 3 18 6+ + βˆ’ βˆ’ βˆ’

= 12 2 3βˆ’ = 2 3 2 3βˆ’ = 0

30. Let x = 0 541.Multiply both side by 10

10x = 5 41. ...(i)Multiply both side by 100

1000x = 541 41. ...(ii)Subtract (i) from (ii)

1000x – 10x = 541 41. – 5 41. 990x = 536

x = 536990

= 268495

31. Find a, b if

7 17 1

7 17 1

βˆ’+

βˆ’ +βˆ’

= a + b 7

= 7 17 1

7 17 1

7 17 1

7 17 1

βˆ’+

Γ— βˆ’βˆ’

βˆ’ +βˆ’

Γ— ++

= ( )( ) ( )

( )( ) ( )

7 17 1

7 17 1

2

2 2

2

2 2βˆ’βˆ’

βˆ’ +βˆ’

= 7 1 2 7

7 17 1 2 7

7 1+ βˆ’

βˆ’βˆ’ + +

βˆ’

= 8 2 7

68 2 7

6βˆ’ βˆ’ +

= 8 2 7 8 2 7

6βˆ’ βˆ’ βˆ’

= βˆ’ = +4 76

7a b

a

b

=

= βˆ’ = βˆ’

046

23

32. 5 25 2

5 25 2

+βˆ’

βˆ’ βˆ’+

5 25 2

5 25 2

+βˆ’

Γ— ++

–

5 25 2

5 25 2

βˆ’+

Γ— βˆ’βˆ’

= ( )( )

( )( ) ( )

5 25 2

5 25 2

2

2 2

2

2 2+

βˆ’βˆ’ βˆ’

βˆ’

= ( ) ( )5 21

5 21

2 2+ βˆ’ βˆ’

= ( ) ( )5 2 5 22 2+ βˆ’ βˆ’

= ( ) ( )5 4 4 5 5 4 4 5+ + βˆ’ + βˆ’

= 9 4 5 9 4 5+ βˆ’ + = 8 5

33. a. 3 13 1

3βˆ’+

= βˆ’a b

3 13 1

3 13 1

βˆ’+

Γ— βˆ’βˆ’

( )( ) ( )

3 13 1

2

2 2βˆ’βˆ’

= 3 1 2 3

3 1+ βˆ’

βˆ’

4 2 3

2βˆ’

= 2 – 3 = a – b 3

∴ [a = 2, b = 1]

b. 2 53 5

+βˆ’

= a + b 5

2 53 5

3 53 5

+βˆ’

Γ— ++

= 6 2 5 3 5 5

3 52 2+ + +

βˆ’( ) ( )

= 11 5 59 5

114

5 54

+βˆ’

= + = a + b 5

∴

a

b

=

=

114

54

34. 5 113 2 11

+βˆ’

= x + y 11

L.H.S. = 5 113 2 11

3 2 113 2 11

+βˆ’

Γ— ++

= ( )( )( ) ( )

5 11 3 2 113 2 112 2

+ +βˆ’

8

MATHEMATICS - 9DDITION LA APR CTICEA

= 15 10 11 3 11 2 11

9 4 11+ + + Γ—

βˆ’ Γ—

= 37 13 119 44+βˆ’

= 37 13 11

35+βˆ’

3735

1335

11βˆ’

+βˆ’

= x + y 11

∴

x

y

=βˆ’

=βˆ’

3735

1335

35. a. 258

= 3.125

8)25(3.125 24 10 8 20 16 40 40 0

b. 2912

= 2 416.

c. 11 Γ· 24 = 0.4583

d. 4 Γ· 7

0.571428

36. (a) Let x = 0 54. ...(i)Multiply both sides by 100 100x = 54.54 ...(ii)Subtract (i) from (ii) 99x = 54

x = 5499

= 1833

611

=(b) Let x = 0.621 ...(i)Multiply both sides by 10 10x = 6.21 ...(ii)Multiply both sides by 100 1000x = 621.21 ...(iii)Subtract (ii) from (iii) 990x = 615

x = 615990

= 205330

4166

=(c) Let x = 4.7 (i)Multiply both sides by 10 10x = 47.7 (ii)Subtract (i) from (ii) 9x = 43

x = 439

(d) Let x = 2.665 ...(i)Multiply both sides by 100 100x = 266.5 ...(ii)Multiply both sides by 10 1000x = 2665.5 ...(iii)Subtract (ii) from (iii) 900x = 2399

x = 2399900

2399900

=

37. x = 3 23 2

βˆ’+

, y = 3 23 2

+βˆ’

Check whether: x2 + xy + y2 = 99

x = 3 23 2

3 23 2

βˆ’+

Γ— βˆ’βˆ’

3 2 2 6

3 2+ βˆ’

βˆ’ = 5 2 61

βˆ’

y = 3 23 2

3 23 2

+βˆ’

Γ— ++

= 3 + 2 + 2 6 = 5 + 2 6 x2 + xy + y2

= ( ) ( ) ( )5 2 6 5 2 6 5 2 62βˆ’ + βˆ’ + + +( )5 2 6 2

= 25 24 20 6 25 24 25 24+ βˆ’ + βˆ’ + + + 20 6 = 99

38. Simplify: 16 6 343 18 243 1964 3 5βˆ’ + βˆ’

= ( ) . .2 6 7 18 3 1444 33 55 2βˆ’ + βˆ’

= ( ) . ( ) ( ) ( )2 6 7 18 3 14414 3

13 5

15 2

12βˆ’ + Γ— βˆ’

= ( ) . ( ) ( ) ( )2 6 7 18 3 144

14

313

515

212

Γ— Γ— Γ— Γ—βˆ’ + Γ— βˆ’

= 2 – 42 + 54 – 14 = 0

9 9

MATHEMATICS - 9DDITION LA APR CTICEA

Worksheet-1

Section A 1. (x – 1) (x – 2) (x – 3) (x – 4) = 0

So, (x – 1) = 0 x = 1 (x – 2) = 0 x = 2 (x – 3) = 0 x = 3 (x – 4) = 0 x = 4\ Zeros of the polynomial are 1, 2, 3, 4

2. p(x) = x3 – 27 = x3 – 33

= (x – 3) (x2 + 9 + 3x) = (x – 3) (x2 + 3x + 9)Using (a3 – b3) = (a – b) ( a2 + b2 + ab)Here a = x b = 3To find zeroes of p(x)Put p(x) = 0 (x – 3) (x2 + 3x + 9) = 0 x – 3 = 0, x2 + 3x + 9 = 0 x = 3,So, we have only one real root x = 3.

3. p(x) = x3 – x2 + x + 1 if x = – 1, then

p(– 1) = (–1)3 – (– 1)2 + (– 1) + 1 = – 1 – (1) – 1 + 1 = – 3 + 1 = – 2 p(1) = (1)3 – (1)2 + (1) + 1 = 1 – 1 + 1 + 1 = 2 Putting the values of p(1), p(– 1), we get:

p p( ) ( )1 12

+ βˆ’

= 2 22

2 22

02

+ βˆ’ = βˆ’ =( ) = 0.

\ p p( ) ( )βˆ’ +1 12

= 0

4. x x+

+

12

32

⇒  x x x+

+ +

32

12

32

Polynomials02Chapter

β‡’ x x x2 32

12

34

+ + +

β‡’ x x2 42

34

+ +

β‡’ x x2 234

+ +

4 8 3

4

2x x+ + =

14

(4x2 + 8x + 3)

5. (– 12)3 + (7)3 + (5)3

By using the identity x3 + y3 + z3 = 3xyz This identity is possible if

x + y + z = 0, thenwhere as x = – 12 y = 7 z = 5 x + y + z = – 12 + 7 + 5 = 0Hence verified.So, we can use the identity x3 + y3 + z3 = 3xyz. 3xyz = 3(– 12) (7) (5) = – 1260\ (– 12)3 + (7)3 + (5)3 = – 1260

6. p(x) = x3 – a2x + x + 2 x – a, is a factor of the polynomial, then

p(a) = (a)3 – a2(a) + a + 2 = 0β‡’ a3 – a3 + a + 2 = 0β‡’ a + 2 = 0β‡’ a = – 2\ Value of a is – 2.

7. p(x) = x31 + 31 q(x) = x + 1q(x) = 0 β‡’ x = – 1Putting the value of x = – 1 in p(x) p(– 1) = (– 1)31 + 31 = – 1 + 31 = 30.Remainder will be 30.

8. 2x2 – 7x = 0β‡’ x(2x – 7) = 0Zeroes will be

0 and 72

∡ 2 7 0

72

x

x

βˆ’ =

=

9. 2x3 – kx2 + 7x – 1 = p(x) Since when p(x) is divided by x – 1, remainder is 3, So

p(1) = 3⇒  2(1)3 – k(1)2 + 7(1) – 1 = 3 ⇒  2 – k + 7 – 1 = 3⇒  – k + 8 = 3⇒  8 – 3 = k⇒  5 = k  

10

MATHEMATICS - 9DDITION LA APR CTICEA

10. Given P(x) = 3x3 – 2x2 – x + 4 Sol. If P = – 1, then

P(–1) = 3(– 1)3 – 2(– 1)2 – (–1) + 4 = 3 Γ—(– 1) – 2(1) + 1 + 4 = – 3 – 2 + 5 = – 5 + 5 = 0. If P = 1, then P(1) = 3(1)3 – 2(1)2 – (1) + 4 = 3 – 2 – 1 + 4 = 1 – 1 + 4 = 4Putting the value of P(1) and P(–1), then P(1) + P(– 1) = 4 + 0 = 4\ P(1) + P(–1) = 4.

Section B 11. 185 Γ— 185 – 15 Γ— 15

⇒  (185)2 – (15)2

By using the identity (x2 – y2) = (x + y) (x – y), we getβ‡’ (185)2 – (15)2 = (185 + 15) (185 – 15) β‡’ (185)2 – (15)2 = 200 Γ— 170β‡’ (185)2 – (15)2 = 34000\ 185 Γ— 185 – 15 Γ— 15 = 34000.

12. p(x) = 2x2 – 3x + 7aSince x = 2 is a root of p(x) β‡’ p(2) = 0β‡’ 2 (2)2 – 3 (2) + 7a = 0β‡’ 8 – 6 + 7a = 0β‡’ 2 +7a = 0

β‡’ a =

13. Givenx – 1 is a factor of the polynomial x2 + x + k.\ x – 1 = 0 x = 1Putting the value x = 1 in the equation x2 + x + k = 0So, (1)2 + (1) + k = 0 1 + 1 + k = 0 2 + k = 0 k = – 2\ Value of k = – 2.

14. Given P(x) = x2 – 4x + 4To find p(2) + p(– 2) + p(1)

Sol. If x = 2, then p(2) = (2)2 – 4(2) + 4 = 4 – 8 + 4 = 0 ...(1)If x = – 2, then p(–2) = (–2)2 – 4(– 2) + 4 = 4 + 8 + 4 = 16 ...(2)

If x = 1 p(1) = (1)2 – 4(1) + 4 = 1 – 4 + 4 = 1 ...(3)Putting the value of p(2), p(–2), p(1) in x2 – 4x + 4, we getβ‡’ 0 + 16 + 1 = 17.\ p(2) + p(–2) + p(1) = 17.

15. (99)3

99 can also be expressible in the form (100 – 1), then (99)3 β‡’ (100 – 1)3

Using the identity (x – y)3 = x3 – y3 – 3xy (x – y), we getwhere x = 100 y = 1 (100 – 1)3 = (100)3 – (1)3 – 3(100) (100 – 1) = 1000000 – 1 – 29700 = 1000000 – 29701 = 970299\ (99)3= 970299

16. βˆ’ + +

xy

214

2

= (a + b + c)2 = a2 + b2 + c2 + 2ab + 2bc + 2ca

= βˆ’

+ +

+ βˆ’

x y x y2

14

22

22

2

( ) ( )

+

+

βˆ’

2 1

42 14 2

( )y x

= x

y xy y x22

4116

22 2 4

+ + + βˆ’

+ + βˆ’

= x

y xyy x2

2

4116 2 4

+ + βˆ’ + βˆ’

17. x4 – 125 xy3

= x(x3 – 125y3) = x(x3 – (5y)3) = x(x – 5y) (x2 + 25y2 + 5xy)

18. x2 + y2 = 58, x + y = 10 To find : x3 + y3

x3 + y3 = (x + y) (x2 + y2 – xy) = (10) (58 – xy) ...(i)Consider (x + y)2 = x2 + y2 + 2xy (10)2 = 58 + 2xy 100 – 58 = 2xy 21 = xy ...(ii)

Put (ii) in (i) x3 + y3 = 10 (58 – 21) = 370

19. Factorise: 64x3 – 27y3 + z3 + 36xyz = (4x)3 + (– 3y)3 + z3 – 3(4x) (– 3y)z = [4x + (– 3y) + z][(4x)2 + (– 3y)2 + (z)2 – (4x) (– 3y) – (– 3y)z

– (z) (4x) = (4x – 3y + z) (16x2 + 9y2 + z2 + 12xy + 3yz – 4zx)Identity : a3 + b3 + c3 – 3abc = (a + b +c) (a2 + 2 + c2 – – bc – ca)

20. (x – 1)3 = 8 (x – 1)3 = 8 (x – 1)3 = (2)3

x – 1 = 2 x = 2 + 1 = 3 (x + 1)2 = (3 + 1)2 = (4)2

= 16

11 11

02. Polyn

omials

MATHEMATICS - 9DDITION LA APR CTICEA

21. x3 – 23x2 + 142x – 120Put x = 113 – 23 (1)2 + 142 (1) – 120 = 0\ x – 1 is a factor

Therefore x2 – 22x + 120 = (x2 – 22x + 120) = (x2 – 12x – 10x + 120) = (x(x – 12) – 10 (x – 12) = (x – 12) (x – 10) are factors.So, x3 – 23x2 + 142x – 120 = (x – 1) (x – 12) (x – 10)

22. We know that: (a + b) = 7 a2 + b2 + 2ab = 49 a2 + b2 = 49 – 20 = 29

23. P(x) = Kx3 + 9x2 + 4x – 8 divided by (x + 3) leaves remiander 10 – 10K – 27K + 81 – 12 – 8 = 10 – 10K – 27K + 61 = 10 – 10K 61 – 10 = – 10K + 27K 51 = 17K

5117

= K

\ Value of K = 3 24. p(x) = x4 – 2x3 + 3x2 – mx + 3

p(1) = R1

1 – 2 + 3 – m + 3 = R1

– 1 + 6 – m = R1

5 – m = R1

p(– 1) = R2

(– 1)4 – 2 (– 1)3 + (– 1)2 – m (– 1) + 3 = R2

1 + 2 + 3 + m + 3 = R2

9 + m = R2

3R1 + R2 = 35β‡’ 3 (5 – m) + 9 + m = 35β‡’ 15 – 3m + 9 + m = 35β‡’ – 11 = 2m

So, m =

One value of m can be 8 and other can be 11 25. x3 + 15px + p3 – 125

= x3 + 15x (5 – x) + (5 – x)3 – 125 = x3 + 75x – 15x2 + 125 – x3 – 75x + 15x2 – 125 = 0So, x3 + 15px + p3 – 125 = 0

26. 2 + ax – 2x2 – 3x3

Put x = – 1 2 + a(–1) – 2(–1)2 – 3(–1)3 = 0 2 – a – 2 + 3 = 0 2 – a + 3 = 2 2 – a = –1 a = 3 p(x) = 2 + 3x – 2x2 – 3x3

So, p(x) = (x + 1) (–3x2 + x + 2) = (x + 1) (–3x2 + 3x – 2x + 2) = (x + 1) [–3x (x – 1) – 2(x – 1)] = (x + 1) (x – 1) (–3x – 2)

27. Put a = –b in (a + b + c)3 – (a3 + b3 + c3) [–b + b + c]3 – [–b3 + b3 + c3] = c3 – c3

= 0β‡’ (a + b) is a factor of (a + b + c)3 – (a3 + b3 + c3)Put b = –c[a – c + c]3 – [a3 – c3 + c3] = a3 – a3

= 0β‡’ (b + c) is a factor of(a + b+ c)3 – (a3 + b3 + c3)Put c = –a[a + b – a]3 – [a3 + b3 – a3] = b3 – b3

= 0β‡’ (c + a) is a factor of (a + b + c)3 – (a3 + b3 + c3)

28. 2x4 + 3x3 – 26x2 – 5x + 6Put x = 3 2(3)4 + 3(3)3 – 26(3)2 – 5(3) + 6 = 162 + 81 – 234 – 15 + 6 249 – 249 = 0

29. (a + b + c)3 – a3 – b3 – c3

= (a + b + c)3 – a3 – b3 – c3

= (a + b)3 + c3 + 3c (a + b) (a + b + c) – a3 – b3 – c3

= a3 + b3 + 3ab (a + b) + c3 + 3c (a + b) (a + b + c) – a3 –b3 –c3 = 3ab (a + b) + 3 c (a + b) (a + b + c) = 3(a + b) (ab + c (a + b + c)) = 3(a + b) (a (b + c) + c (b + c)) = 3(a +b) (a + c) (b + c)

30. p(x) = x4 – 2x3 + 3x2 – ax + b p(1) = 5 1 – 2 + 3 – a + b = 5 2 – a + b = 5 – a + b = + 3 ...(i)

12

MATHEMATICS - 9DDITION LA APR CTICEA

p(– 1) = 19(– 1)4 – 2 (– 1)3 + 3 (–1)2 – a (– 1) + b = 19 1 + 2 + 3 + a + b = 19 a + b = 13 ...(ii)Solving (i) and (ii), we get a = 5 b = 8So, p (x) = x4 – 2x3 + 3x2 – 5x + 8Now p(2) = 24 – 2 (2)3 + 3 (2)2 – 5 (2) + 8 = 24 – 24 + 12 – 10 + 8 = 10⇒  Remainder is 10 when p (x) is divided by x –2.

31. (a) x + 1x

= 6 find x2 +12x

xx

+

1 2

= (6)2

= x2 + 12x

+ 2 = 36

x2 + 12x

= 34

(b) To find x4 + 14x

Now, xx

22

21+

= x4 + 14x

+ 2

β‡’ (34)2 = xx

241

2+ +

1154 = xx

441+

32. x = 2y + 6, x3 – 8y3 – 36xy – 216 = (2y + 6)3 – 8y3 – 36(2y + 6)y – 216 = 8y3 + 216 + 72y2 + 216y – 8y3 – 72 2 – 216y – 216 = 0

33. p (1) = 0 13 – a (1)2 + 13(1) + b = 0 1 – a + 13 + b = 0 a – b = 14 ...(1) p(– 3) = 0 (– 3)3 – a (– 3)2 + 13 (– 3) + b = 0 – 27 – 9a – 39 + b = 0 – 9a + b = 66 ...(2)Solve (1) and (2) a = – 10 , b = – 24

Worksheet-II

Section A 1. 3x2 + x

= x(3x + 1) 2. Factorise: 20x2 – 9x + 1 β‡’ 20x2 – (5 + 4) x + 1 ⇒  20x2 – 5x – 4x + 1 ⇒  5x(4x – 1) – 1(4x – 1) ⇒  (4x – 1)(5x – 1) 3. Evaluate: (9)3 + (– 3)3 + (– 6)3

a = 9, b = – 3, c = – 6 a3 + b3 + 3 = 3abcif (a + b + c) = 0 = 3(9) . (– 3) . (– 6) = 27 Γ— 18 = 486

4.

So the other factor is 2x + 1 5. Find the value: (28)3 + (– 15)3 + (– 13)3

a3 + b3 + c3 = 3abc = 3(28) . (– 15) . (– 13) = 84 Γ— 195 = 16, 380

6. x2 + 8x + kPut x = – 1⇒  (– 1)2 + 8(– 1) + K = 0β‡’ 1 – 8 + K = 0⇒  – 7 + K = 0 K = 7

7. Constant Polynomial: A polynomial of degree zero is called a constant polynomial.

The degree of constant polynomial is zero.

8. Factorise: x y2 2

4 4βˆ’

= x y2 2

2 2

βˆ’

=

x y2 2

βˆ’

x y2 2

+

9. 4x3 – 3x2 + 2x – 4 is divided by x + 12

If x + 12

= 0

Put x = –12

in given polynomial.

Remainder

= 4 βˆ’

βˆ’ βˆ’

+ βˆ’

βˆ’12

3 12

2 12

43 2

= 4 Γ— –18

314

βˆ’ . – 1 – 4

R β‡’ – 12

34

51

βˆ’ βˆ’

β‡’ βˆ’ βˆ’ βˆ’2 3 20

4

R β‡’ βˆ’ 254

\ Remainder is βˆ’ 254

10. Find the value of: (55)3 – (25)3 – (30)3

= (55)3 + (– 25)3 + (– 30)3

= 3 Γ— 55 Γ— – 25 Γ— –30 = 165 Γ— 750 = 123750

Section B 11. Factorise: x4 – y4

= (x2)2 – (y2)2

= (x2 + y2) (x2 – y2) [x2 – y2 = (x + y) (x – y)] = (x2 + y2) (x + y) (x – y)

12. f(x) = x3 + 12x2 + 3x – 7Put x = – 3 f(–3) = (– 3)3 + 12(– 3)2 + 3(–3) – 7

13 13

02. Polyn

omials

MATHEMATICS - 9DDITION LA APR CTICEA

  f( – 3) = – 27 + 12 Γ— 9 – 9 – 7  = – 27 – 16 + 108 f(– 3) = – 43 + 108 = 65

13. x + y + z = 12x2 + y2 + z2 = 64Find xy + yz + zx, we know that, (x + y + z )2 = x2 + y2 + z2 + 2xy + 2yz + 2zx (12)2 = 64 + 2 (xy + yz + zx) 144 – 64 = 2 Γ— (xy + yz + zx)

802

= xy + yz + zx

\ xy + yz + zx = 40

14. x xx x

2

27 122 3

+ ++ βˆ’

= x x x

x x x

2

24 3 123 3

+ + ++ βˆ’ βˆ’

= x x xx x x( ) ( )( ) ( )

+ + ++ βˆ’ +4 3 43 1 3

= ( ) ( )( ) ( )x xx x+ ++ βˆ’4 33 1

= ( )( )xx+βˆ’41

15. x3 + 3x2 – kx – 3 ...(i)One factor x + 3 = 0 x = – 3Put x = – 3 in eqn. (1), we get:x3 + 3x2 – kx – 3 = (– 3)3 + 3(– 3)2 – k(– 3) – 3β‡’ – 27 + 27 + 3k – 3 = 0 3k – 3 = 0 3k = 3

k = 33

k = 1 16. Expand: (– 3x + 5y – 23)2

(x + y + z)2 = x2 + y2 + z2 + 2xy + 2yz + 2zx = (– 3x)2 + (5y)2 + (– 23)2 + 2(– 3x) . 5y

+ 2 Γ— 5y . (– 23) + 2(– 23) . (– 3x) = 9x2 + 25y2 + 529 – 30xy – 230y + 138x

17. 2 2 3 33 3a bβˆ’

( ) ( )2 33 3a bβˆ’

= ( )2 3a bβˆ’ (( ) ( ) )2 2 3 32 2a a b b+ + [Q x3 – y3 = (x – y) (x2 + xy + y2)

= ( )2 3a bβˆ’ ( )2 6 32 2a ab b+ +

18. 2 x2 + 9x + 4 2

2 x2 + (8 + 1)x + 4 2

= 2 x2 + 8x + 1x + 4 2

= 2 x (x + 4 2 ) + 1 (x + 4 2 )

= (x + 4 2 ) ( 2 x + 1) 19. g(x) = x – 6 = 0

x = 6 p(x) = x3 – 4x2 + x + 6 p(6) = 63 – 4(6)2 + 6 + 6 = 216 – 144 + 12 = 84 β‰  0So, g(x) is not a factor of p(x)

20. x2 – 16for x = 24 x2 – 16 = x2 – (4)2

⇒  (x + 4) (x – 4)⇒  (24 + 4) (24 – 4)⇒  28 Γ— 20⇒  560

Section C 21.

Quotient = 2x3 – x2 + x + 4Remainder = 2

22. x3 – 3x2 – 9x – 5 ...(1)

Put x = – 1 in eqn. (1) p(– 1) = (– 1)3 – 3(–1)2 – 9(–1) – 5 = – 1 – 3 + 9 – 5 = – 4 – 5 + 9 = – 9 + 9 = 0\ (x + 1) is a factor of p(x). p(x) = (x + 1) [x2 – 4x – 5] = (x + 1) [x2 + x – 5x – 5] = (x + 1) [x(x + 1) – 5(x +1)] = (x + 1) (x + 1) (x – 5)

23.

14

MATHEMATICS - 9DDITION LA APR CTICEA

Using Remainder Theorem, put x = 1, in the given polynomial 3(1)4 – 4(1)2 – 5(1) + 3 = 3 – 4 – 5 + 3 = – 3

24. 4x3 – 16x2 – x + 4If 2x + 1 = 0

2x = – 1 β‡’ x = –12

β‡’ 4 βˆ’

βˆ’ βˆ’

βˆ’ βˆ’

+1

216 1

212

43 2

β‡’ 418

1614

12

4Γ— βˆ’ βˆ’ Γ— + +

β‡’ βˆ’ βˆ’ + +12

412

4 = 0

\ (2x + 1) is a factor of given polynomial. 2x – 1 = 0

2x = 1 β‡’ x = 12

β‡’ 4 12

16 12

12

43 2

βˆ’

βˆ’

+

β‡’ 418

1614

12

4Γ— βˆ’ Γ— βˆ’ +

β‡’ 12

412

4βˆ’ βˆ’ + = 0

\ (2x – 1) is also a factor of given polynomial. x – 9 = 0 β‡’ x = 9β‡’ 4(9)3 – 16(9)2 – 9 + 4β‡’ 4 Γ— 729 – 16 Γ— 81 – 9 + 4 = 2916 – 1296 – 5 = 1615 ≠ 0So (x – 9) is not a factor of given polynomial because Remainder is not zero.

25. Polynomial: 6x3 + 11x2 – x – 6 ...(1)Put x = – 1 in eqn (1), we getβ‡’ 6x3 + 11x2 – x – 6β‡’ 6(– 1)3 + 11(–1)2 – (–1) – 6β‡’ – 6 + 11+ 1 – 6β‡’ 12 – 12 = 0 x = – 1 is a zero of given polynomial.

x = βˆ’ 32

β‡’ 6 32

11 32

32

63 2βˆ’

+ βˆ’

βˆ’ βˆ’

βˆ’

β‡’ 6278

1194

32

6Γ— βˆ’ + Γ— + βˆ’

  = βˆ’ + + βˆ’814

994

32

6

= –814

61

994

32

βˆ’ + +

= βˆ’ βˆ’ + + = βˆ’ +81 24 99 6

4105 105

4 = 0

x = βˆ’ 32

 is a zero of given polynomial.

(x + 1) x +

32

= (x + 1) 12

(2x + 3)

= 12 (x + 1) (2x + 3)

= 12

(2x2 + 3x + 2x + 3)

= 12

(2x2 + 5x + 3)

Put 3x – 2 = 0

x = 23

is a zero of polynomial.

26.

Quotient = 8x2 + 7x + 9Remainder = 6

27. V = 12k(y)2 + 8ky – 20k = 4k (3y2 + 2y – 5) = 4k (3y2 + 5y – 3y – 5) = 4k [y(3y + 5) – 1 (3y + 5)] = 4k (3y + 5) (y – 1)Volume of cuboid = length Γ— breadth Γ— heightβ‡’ Possible expressions for dimenstions of cuboid are length = 4k breadth = 3y + 5 height = y – 1

28. p3 – q3 = (p – q) (p2 + q2 + pq) = (p – q) [(p – q)2 + 2pq + pq] = (p – q) ((p – q)2 + 3pq)

= 109

109

353

2

+

= 109

10081

5+

=

109

Γ— 50581

= 5050729

29. p(x) = x4 – 2x3 + 3x2 – ax + 8

(2)4 – 2(2)3 + 3(2)2 – 2a + 8 = 0 16 – 16 + 12 – 2a + 8 = 10

20 – 2a = 10 2a = 20 – 10

a = 102

= 5

30. p(x) = x3 + 3x2 – 3px + q – 1 + 3 + 3p + q = 0 2 + 3p + q = 0 3p + q = – 2 ...(1) – 8 + 12 + 6p + q = 0 4 + 6p + q = 0 6p + q = – 4 ...(2)

15 15

02. Polyn

omials

MATHEMATICS - 9DDITION LA APR CTICEA

From (1) and (2), we get,

p = βˆ’23

q = 0 31.

Since remainder is 0, x – 2 is a factor of the given polynomial.

Therefore, x3 – 2 2 x2 – 10x + 12 2

= ( ) ( )x x xβˆ’ βˆ’ βˆ’2 2 122

= ( ) ( )x x x xβˆ’ βˆ’ + βˆ’2 3 2 2 2 122

= ( )[ ( ) ( )]x x x xβˆ’ βˆ’ + βˆ’2 3 2 2 2 3 2

= ( ) ( ) ( )x x xβˆ’ + βˆ’2 2 2 3 2

32. If P(x) = x3 – 4x2 + x + 6. P(3) = (3)3 – 4(3)2 + 3 + 6 = 27 – 36 + 9 = 36 – 36 = 0

\ (x – 3) is a factor of P(x).

(x – 3) (x2 – x – 2) = (x – 3) (x2 – 2x + x – 2) = (x – 3) (x + 1) (x – 2)

33. x + 1x

= 7

xx

+

1 3

= 73

x3 + 1

31 1

3xx

xx

x+ +

= 343

x3 + 13x

= 322

34. 125 a3 – 27b3 + 75 a2 b – 45 ab2

= (5a)3 – (3b)3 + 15 ab (5a – 3b) = (5a – 3b) (25 a2 + 9b2 + 15 ab) = (5a – 3b) (25 a2 + 9b2 + 15 ab + 15 ab) = (5a – 3b) (25 a2 + 9b2 + 30 ab)

35. b cbc

c aca

a bab

+( ) ++( ) +

+( )2 2 2

3 3 3

= βˆ’( ) +

βˆ’( ) +βˆ’( )a

bcbca

cab

2 2 2

3 3 3

= abc

bca

cab

2 2 2

3 3 3+ +

(Since a + b + c + = 0)

= abc

bca

cab

2 2 2

3 3 3+ +

= a b c

abc

3 3 3

3+ +

= 33abcabc

= 1

As a + b + c = 0a3 + b3 + c3 = 3abc

36. Factorise : (a + b – c)3 + (a – b + c)3 – 8a3 = (a + b – c)3 + (a – b + c)3 + (– 2a)3

Hence, x = a + b – c y = a – b + c z = – 2a x + y + z = a + b – c + a – b + c – 2a = 0\ x3 + y3 + z3 = 3xyzi.e. (a + b – c)3 + (a – b + c)3 – 8a3

= 3x(a + b – c) Γ— (a – b + c) Γ— (–2a) = – 6a (a + b – c) (a – b + c) 37. We have:

p(x) = 3x4 + 5x3 – 7x2 + 2x + 3 g(x) = x2 + 3x + 1By Division algorithm p(x) = g(x) Γ— q(x) + r(x) p(x) – r(x) = g(x) Γ— q(x)So, if we subtract r(x) with p(x), the resulting polynomial is divisible by g(x), on dividing p(x) by g(x), we get

34x + 5

3x – 72x + 2x

+ 93x3

4x

x x2+ 3 + 1

–

–

3x2–

+

+

2+

+ 3

4x

4x3

43x

–

–

102x

122x

22x

22x

6x6x

1

++

+

–

2x + 3

4x+

– –

+

+

–

3

2

32x

–

+

––

R

\ Remainder is 1Hence, if 1 is subtracted from p(x) the resulting polynomial is exactly divisible by g(x).

38. We have: x4 + x2y2 + y4 = (x4 + 2x2y2 + y4) – x2y2

= (x2 + y2)2 – (xy)2

= (x2 + y2 – xy) (x2 + y2 + xy)\ x4 + x2 y2 + y4

= (x2 + y2 – xy) (x2 + y2 + xy) 39. a. a + b + c = 5, ab + bc + ca = 10 We know that: (a3 + b3 + c3 – 3abc)

= (a + b + c) (a2 + b2 + c2 – ab – bc – ca) = (a + b + c) [(a + b + c)2 – 3( ab + bc + ca)] = (5) Γ— [(5)2 – 3 Γ—10] = 5 Γ— (25 – 30)

16

MATHEMATICS - 9DDITION LA APR CTICEA

= 5 Γ— (– 5) = – 25 a3 + b3 + c3 – 3abc = – 25

Hence Proved.

b. Given: p + q + r = 16 Taking square on both sides we get

(p + q + r)2 = (16)2 = 256Using identity:

(a + b + c)2 = a2 + b2 + c2 + 2ab + 2bc + 2ca p2 + q2 + r2 + 2pq + 2qr = 256

⇒  p2 + q2 + r2 + 2(pq + qr + rp) = 256⇒  p2 + q2 + r2 + 2(pq + qr + rp) = 256⇒  p2 + q2 + r2 + 2 Γ— 25 = 256⇒  p2 + q2 + r2 = 256 – 50⇒  p2 + q2 + r2 = 206

17 17

MATHEMATICS - 9DDITION LA APR CTICEA

Worksheet-1

Section A 1. The distance of the point p(2, 1) from y-axis is 2. 2. The mirror image will be coordinates (2, –5). 3. The mirror image will be (–2, 5). 4. General form of any point of y-axis is (0, y). 5. The reflection of the point (–3, –2) in y-axis will be (3, –2).

Section B 6. i. Abscissa is – 4 and the ordinate is –2. β‡’ III Quadrant. ii. The ordinate is 3 and abscissa is 4. β‡’ I Quadrant. 7. i.

1 2 3 4 5–1–2–3–4

1

2

3

4

–1

–5 XXοΏ½

YοΏ½

Y

A(–5, 3) B(3, 3)

D(–5, 0) C(3, 0)

O

ii. ABCD is a rectangle. 8. Ordinates of the following are: (3, 4) ordinate is 4. (4, 0) ordinate is 0. (0, 4) ordinate is 4. (5, –3) ordinate is –3.

Coordinate Geometry03Chapter

18

MATHEMATICS - 9DDITION LA APR CTICEA

9.

No, the points (3, 2) and (2, 3) are not same because they have different abscissa and ordinate lie in the same Quadrant. 10. i.

ii. MN = 8 units.

19 19

03. Coord

inate G

eometry

MATHEMATICS - 9DDITION LA APR CTICEA

Section C 11.

12. Coordinates of points which is reflection of (3, 5) in y-axis are (–3, 5). Coordinates of points which is reflection of (3, 5) in x-axis are (3, –5).

20

MATHEMATICS - 9DDITION LA APR CTICEA

13.

Points Quadrants

(–2, 4) II

(3, –1) IV

(–1, 0) Negative x-axis

(1, 0) Positive x-axis

(1, 2) I

(–3, –5) III

(0, –1) Negative y-axis

14. If a point lie on x-axis at a distance of 9 units from y-axis then its coordinates can be (9, 0) or (–9, 0). If a point lie on y-axis at a distance of 9 units from x-axis then its coordinates can be (0, 9) or (0 – 9). 15.

1 2 3 4 5–1–2–3–4

1

2

3

4

–1

–2

–3

–5 XXοΏ½

YοΏ½

Y

5(2, 5)(–2, 5)

(4, –3)

(0, 2.5)

(–4, –3)

O 6

(6, –1)

16.

Fourth vertex i.e. D will be (3, 5) to form a rectangle. A(3, 2) and D(3, 5) lies in I-Quadrant. B(–4, 2) and C(–4, 5) lies in II-Quadrant.

21 21

03. Coord

inate G

eometry

MATHEMATICS - 9DDITION LA APR CTICEA

17.

Triangle is formed by joining the points A, B and C. Area of ABC = 12

Γ— Γ—b h = 12

8 4Γ— Γ— = 16 sq. unit 18.

A(–3, –4) β‡’ III C(–1, 4) β‡’ IIB(–2, 0) on negative x-axis.D(1, 0) on positive x-axis.

19.

Diagonals intersect at point O. Coordinates of point O are (4, 1).

22

MATHEMATICS - 9DDITION LA APR CTICEA

20. Coordinates of the given points are: P(1.5, 0.5) Q(–3, 0.5) R(–2, –2.5) S(3, 0.5) O(0, 0). T(4, –1.5)

Worksheet-II

Section A 1. (–, +). 2. III Quadrant. 3. Abscissa of P (–2, 3) is –2 Abscissa of Q (–3, 5) is –3. 4. Here y is 4, distance from y-axis is 4 unit. 5. (a) y-axis (b) x-axis.

Section B 6. (a) Yes, True.

(b) No, False, it will be 112

,βˆ’

.

7.

Trapezium is formed by joining the points P, Q, R and S. 8. Coordinates are (7, 0) or (–7, 0) coordinates will be (0, –7) or (0, 7).

23 23

03. Coord

inate G

eometry

MATHEMATICS - 9DDITION LA APR CTICEA

9.

Q and O lies on x-axis. 10. Abscissa of P (–5, 3) is –5 Abscissa of Q (7, 3) is 7 Ordinate of P (–5, 3) is 3

Ordinate of Q (7, 3) is 3.

Section C 11.

1 2 3 4 5–1–2–3–4

1

2

3

4

–1

–2

–3

–4

–5 XXοΏ½

YοΏ½

Y

D(5, –4)C(–2, –4)

B(–2, 3) A(5, 3)

6O

Coordinates of the vertex C(–2, –4).

24

MATHEMATICS - 9DDITION LA APR CTICEA

12.

i. Yes ii. No iii. Yes. 13. i. The origin (0, 0). ii. (0, –4). iii. (5, 0). 14.

25 25

03. Coord

inate G

eometry

MATHEMATICS - 9DDITION LA APR CTICEA

15.

Area of βˆ† ABC = 12

Γ— Base Γ— Height

= 12

Γ— 6 Γ— 6 = 18 sq. units.

Section D 16.

26

MATHEMATICS - 9DDITION LA APR CTICEA

17.

(a) Coordinates of the points between the points A and B are (2, 1), (2.5, 2), (3, 3), (3.5, 4) etc. (b) Ordinate of A (1, –1) is –1. Ordinate of B (4, 5) is 5. 18.

Therefore parallelogram ABCD is formed.

27 27

MATHEMATICS - 9DDITION LA APR CTICEA

Worksheet-1

Section A 1. x = 2, y = –1 is a solution of:

px + 3y = 15 p Γ— 2 + 3 Γ— –1 = 15 2p – 3 = 15 2p = 15 + 3 = 18

p = 182

p = 9.

2. 2x + 5 = 5y

2 5x yβˆ’ + 5 = 0 ...(i) ax + by + c = 0 ...(ii)Comparing eqn. (i), (ii), we get:

a = 2 , b = βˆ’ 5 , c = 5. 3. x – 3y = 4

x = 4 + 3y x = 4 + 3 Γ— 0 = 4 x = 4 + 3 Γ— 1 = 4 + 3 = 7

∴ Two solutions are (4, 0), (7, 1). 4. x = – 1, y = –1

Linear equation: 9kx + 12ky = 63 9k(–1) + 12k (–1) = 63 –9k – 12k = 63 –21k = 63

k = 6321βˆ’

k = –3. 5. Point (–1, –5) lies on the graph:

3x = ay + 7 3(–1) = a(–5) + 7 –3 = – 5a + 7 5a = 7 + 3 5a = 10 a = 2

6. LHS = x – 2y

= 2 – 2 ( )4 2

= 2 8 2βˆ’

= βˆ’7 2 β‰  4 = RHS

So, ( , )2 4 2 is not a solution of the equation.

Then, ( , )2 4 2 is not a solution of x – 2y = 4. 7. A linear equation has infinitely many solution. 8. The equation representing y-axis is x = 0.

9. 2x + 3y = 9 35.

2x + 3y – 9 35. = 0 ...(i) ax + by + c = 0 ...(ii)

x 4 7

y 0 1

Linear Equations in Two Variables04

Chapter

28

MATHEMATICS - 9DDITION LA APR CTICEA

Comparing eqns. (i), (ii), we get:

c = – 9 35. . 10. LHS = x – 3y

= 0 – 3(2) = – 6 β‰  4 = RHS

So (0, 2) is not a solution of the equation.

Section B 11. (a) 4x + 3y = 12

3y = 12 – 4x ...(i) 3y = 12 – 4 Γ— 0 = 12

y = 123 = 4

3y = 12 – 4 Γ— 1 = 12 – 4 = 8 y = 8/3

∴ Two solutions are (0, 4), (1, 8/3). (b) 2x + 5y = 0

2x = –5y 2x = –5 Γ— 0 x = 0 2x = –5y, y = 2 2x = –5 Γ— 2 = –10

x = –102

= –5

Therefore two solutions are (0, 0) and (–5, 2). 12. 4 solutions of 5x = – y,

Put y = 0, 5x = –0, x = 0y = –1, 5x = – (– 1) = 1, x = 1/5y = 2,

x = –25

y = 5,

x = βˆ’55

= – 1

Therefore 4 solutions are (0, 0), (1/5, –1) (–2/5, 2), (–1, 5).

13. For 2 kms, the fare is ` 19 and for subsequent distance it is ` 6.50 km.Total distance covered is = x

Total fare as = ` yfor first 2 kms fare = ` 19for next remaining (x – 2) km = 6.50 y = 19 + (x – 2)6.50 y = 19 + 6.50x – 13

y = 6 + 6.50x

14. x + 2y = 6 ...(i) x = 6 – 2yPut y = 0, x = 6 – 2 Γ— 0 = 6 y = 1, x = 6 – 2 Γ— 1 = 4 y = 2, x = 6 – 2 Γ— 2 = 6 – 4 = 2 y = 6, x = 6 – 2 Γ— 6 = 6 – 12 = – 6Therefore four solutions are (6, 0), (4, 1), (2, 2), (–6, 6).

x 0 1

y 4 8/3

x 0 – 5

y 0 2

x 0 1/5 – 2/5 – 1

y 0 – 1 2 5

x 6 4 2 – 6

y 0 1 2 6

29 29

04. Linear Eq

uation

s in Tw

o Variab

les

MATHEMATICS - 9DDITION LA APR CTICEA

15. i. Graphical representation in one variable: 5x + 15 = 0 5x = –15

x = βˆ’155

= –3

x = –3.

1 2 3–1–2–3 XXοΏ½ 0

x = 3

ii. In two variables: x = –3

β‡’ x + 0.y + 3 = 0 Solutions are: (–3, 0), (–3, 1), (–3, 2).

Section C 16. 3x + y = 6 y = 6 – 3x y = 6 – 3 Γ— 1 = 3 y = 6 – 3 Γ— 2 = 6 – 6 = 0.

The line meet x-axis at (2, 0) and y-axis at (0, 6)

x –3 –3 –3

y 0 1 2

x 1 2

y 3 0

30

MATHEMATICS - 9DDITION LA APR CTICEA

17. Let No. of hours = ` x and wages = ` y

y = 2x – 1 ...(1)x = 1, y = 2 Γ— 1 – 1 = 1x = 2, y = 2 Γ— 2 – 1 = 3x = 6, y = 2 Γ— 6 – 1 = 12 – 1 = 11

1 2 3 4–1

1

2

3

XXοΏ½YοΏ½

Y

O

4

5

6

5 6 7 8

7

8

9

10

11 C(6, 11)

y=

2x

–1

B

A

Labour works in 6 hours.We get (6, 11), so for 6 hours he get 11 rupees

18.

1 2 3 4 5–1–2–3–4

1

2

3

–1

–2

–3

–4

XXοΏ½

YοΏ½

Y

O 6 7 8

C(0, –3)

A(4, 0)

B(8, 3)

3x–

4y=

12

Line meet two axis at A(4. 0)

for X-axis and at (0, –3) Y-axis

Find three solutions of equation: 3x – 4y = 12 3x = 12 + 4y

x 1 2 6

y 1 3 11

31 31

04. Linear Eq

uation

s in Tw

o Variab

les

MATHEMATICS - 9DDITION LA APR CTICEA

Put, y = 0, 3x = 12 + 4 Γ— 0 = 12

x = 123

= 4

y = 3, 3x = 12 + 4 Γ— 3 = 12 + 12 = 24

x = 243

= 8

y = –3, 3x = 12 + 4 Γ— –3 = 12 – 12 = 0 x = 0Line 3x – 4y = 12 meet on x-axis at A(4, 0) and y-axis at B(0, – 3).

19. Draw the graph:

1 2 3 4 5–1–2

1

2

3

–1

–2

–3

–4

XXοΏ½

YοΏ½

Y

O 6 7 8

A(8, 3)

3x–

4y=

12

x = 4, y = 2 is not lying on graph

(4, 2) is not a solution of given equationοΏ½

9 10 11 12

4

5

6B(12, 6)

3x – 4y = 12 ...(i) 3x = 12 + 4y

x = 123

43

+ y

x = 4 + 43

y

y = 3, x = 4 + 43

3Γ— = 8

y = 6, x = 4 + 43

6Γ— = 4 + 8 = 12

Sincepoint (4, 2) does not lie on graph. So, x = 4, y = 2 is not a solution of equation 3x – 4y = 12. 20. 3x + 7y = 20 ...(i)

3x = 20 – 7yPut y = –1, 3x = 20 – 7 Γ— – 1 3x = 20 + 7

x = 273

= 9

y = +2, 3x = 20 – 7 Γ— +2 3x = 20 – 14 = 6

x = 63

= 2

x 4 8 0

y 0 3 –3

x 8 12

y 3 6

32

MATHEMATICS - 9DDITION LA APR CTICEA

Points are (9, –1) (2, 2) graph shown on graph paper.

The line meets x-axis at 203

0,

and y-axis at 0207

,

Line 3x + 7y = 20 meet on y-axis at (0, –2.5). 21.

Points are plot on above graph: A(4, 1), B(1, –2) Linear eqn. cut the x-axis at the point (3, 0). Linear eqn. cut the y-axis at the point (0, – 3).

Section D 22. Monthly expenditure on milk = ` 500

Extra quantity takes = x kgTotal expenditure = ` yLinear equation: ` y = 500 + 20x

x 10 20

y 700 900

33 33

04. Linear Eq

uation

s in Tw

o Variab

les

MATHEMATICS - 9DDITION LA APR CTICEA

23. Solve for x:

24 – 3(x – 2) = x + 18 24 – 3x + 6 = x + 18 24 – 3x – x + 6 – 18 = 0 30 – 18 – 4x = 0 12 – 4x = 0 –4x = –12

x = 124

x = 3. 24. x + y = 5 ...(i)

y = 5 – xx = 1, y = 5 – 1 = 4x = 2, y = 5 – 2 = 3 2x + 2y = 12 ...(ii) x + y = 6 y = 6 – x,x = 2, y = 6 – 2 = 4x = 4, y = 6 – 4 = 2

x 1 2

y 4 3

x 2 4

y 4 2

34

MATHEMATICS - 9DDITION LA APR CTICEA

Graph of above two lines show that the two lines are parallel lines. i.e. l || m 25. 2x – 3y = 12

–3y = 12 – 2x 3y = 2x – 12

y = 23

123

x βˆ’ = 23

x – 4

Let x = 3, y = 23

3Γ— – 4 = 2 – 4 = –2

x = 6, y = 23

6Γ— – 4 = 4 – 4 = 0

x = 0, y = 23

Γ— 0 – 4 = –4

The line meets x-axis at point (6, 0) and y-axis at point (0, – 4)

Worksheet-II

Section A 1. x = 2, y = 2

2x – ky + 7 = 8 2 Γ— 2 – k Γ— 2 + 7 = 8 4 + 7 – 2k = 8 11 – 2k = 8 – 2k = 8 – 11 = – 3 k = 3/2.

2. x = 2 is a line parallel to y-axis at a distance of 2 units. 3. y = 0, representing x-axis. 4. General form of a linear equation in two variables.

ax + by + c = 0.

Section B 5. x + 2y = 7

y = 0, x = 7 – 2y x = 7 – 2 Γ— 0 = 7y = 1, x = 7 – 2 Γ— 1 = 5y = 2, x = 7 – 2 Γ— 2 = 7 – 4 = 3y = 3, x = 7 – 2 Γ— 3 = 7 – 6 = 1∴ 4 solutions are: (7, 0), (5, 1), (3, 2) and (1, 3).

x 3 0 6

y –2 –4 0

x 7 5 3 1

y 0 1 2 3

35 35

04. Linear Eq

uation

s in Tw

o Variab

les

MATHEMATICS - 9DDITION LA APR CTICEA

6. P(x, y) x = abscissa, y = ordinate

y = 32

x

3x+ 4y = 18

3x + 432

Γ— x = 18

3x + 6x = 18 9x = 18

x = 189

= 2

Points (2, 3). 7. y = 3 i. In one variable:

ii. In two variables:

8. Find m: If x = –3, y = –1 is a solution of: mx – 3y = 15 m(–3) – 3(–1) = 15 –3m + 3 = 15 –3m = 15 – 3 = 12

m = 123βˆ’

m = –4. 9. Give two solutions of the equation: x + 3y = 8.

y = 1, x = 8 – 3y x = 8 – 3 Γ— 1 = 5

y = 0, x = 8 – 3 Γ— 0 = 8y = 3, x = 8 – 3 Γ— 3 = 8 – 9 = –1∴ Two solutions are (5, 1), (8, 0) and (–1, 3).

x 5 8 –1

y 1 0 3

36

MATHEMATICS - 9DDITION LA APR CTICEA

10. Draw graph:

11. y = 9x – 7For x = 1, y = 2

9x – 7 = 9 – 7 = 2 = y = RHSFor x = 0, y = –2

9x – 7 = 9(0) – 7 = –7 β‰  – 2So, (1, 2) is a solution of y = 9x – 7 but (0, –2) is not a solution of y = 9x – 7.

Section C 12. Let larger number = x and smaller number = y

According to Q: 5x = 2y + 9 ...(i) [∡ By Division Algorithm]y = 1, 5x = 2 Γ— 1 + 9 = 11

x = 115

y = 3, 5x = 2 Γ— 3 + 9 = 15 x = 3

Therefore two solutions are 1151,

and (3, 3).

13. C = 5 160

9F βˆ’

...(i)

a. T = 30Β°C

30 = 5 160

9F βˆ’

30 Γ— 9 = 5F – 160 270 + 160 = 5F 430 = 5F

F = 4305

= 86

Temperature is 86Β°F. b. Let x be the temperature which is numerically same in both fahrenheit and celcius.

So, x = 59 (x – 32)

9x = 5x – 32(5) 9x = 5x – 160 160 = 4x –40 = x

The temperature is numerically same at –40Β°. 14. 4x + 3y + 6 = 0

4x = –3y – 6 4x = –3(y + 2)

x 11/5 5

y 1 3

x 0 – 3 –6

y –2 2 6

37 37

04. Linear Eq

uation

s in Tw

o Variab

les

MATHEMATICS - 9DDITION LA APR CTICEA

y = –2 4x = –3(–2 + 2) = –3 Γ— 0 = 0 x = 0y = 2, 4x = –3(2 + 2) 4x = –3 Γ— 4 = –12 x = –3y = 6, 4x = –3(y + 2) 4x = –3(6 + 2) = –3(8) 4x = –24

x = βˆ’ 244

= –6

x = –6, y = 6When

y = 6, x = – 6So, coordinates are (– 6, 6)

15. 3x – 2y = 4 ...(i)

x = 4 23

+ y

y = 1, x = 4 2 1

3+ Γ—

= 63

= 2

y = 4, x = 4 2 4

3+ Γ—

= 123

= 4

P(2, 1), Q(4, 4) x + y – 3 = 0y = 1, x = 3 – y x = 3 – 1 = 2y = 3, x = 3 – 3 = 0(2, 1), (0, 3).

x 2 4

y 1 4

x 2 0

y 1 3

38

MATHEMATICS - 9DDITION LA APR CTICEA

16. 3x – 2y = 6 ...(i) 3x = 6 + 2y

x = 6 2

3+ Γ— y

y = 3, x = 63

23

+ y = 2 + 23

3Γ— = 4

y = 6, x = 2 + 23

6Γ— = 2 + 4 = 6

y = –6, x = 2 + 23

6Γ— βˆ’

x = 2 – 4 = – 2Point (–2, –6).

1 2 3 4–1–2

1

2

3

–1

–2

XXοΏ½

YοΏ½

Y

O

4

5

6

–3 5 6 7 8

–3

–4

–5

–6P(–2, –6)

(6, 6)

(4, 3)

3x

–2y

=6

–4

If y = –6

then x = –2

P(–2, –6)

x 4 6 – 2

y 3 6 – 6

39 39

04. Linear Eq

uation

s in Tw

o Variab

les

MATHEMATICS - 9DDITION LA APR CTICEA

17. P(1, –1) 3x + 2y – 1 = 0 ...(i) 3x + 2y = 1 3x = 1 – 2y1

x = 1 23

βˆ’ y

2x + y = 1 ...(ii) 2x = 1 – y

x = 12βˆ’ y

Section C 18. x = 2, ...(i)

x + 2 = 0 ...(ii)β‡’ x = –2 y + 3 = 0 y = –3 ...(iii)

1 2 3 4–1–2–3–4

1

2

–1

–2

–3

XXοΏ½

YοΏ½

Y

O 5

–4

x=

2

x=

–2

y = –3

Area enclosed

between 3 lines

ABCD = Γ— b

= 4 Γ— 3

= 12 sq. unit

l

CD

A B

x –1 3

y 2 –4

x –1 –2

y 3 5

40

MATHEMATICS - 9DDITION LA APR CTICEA

19. Let x = distance and y = fare` 50 is fixed fare.` 16 per km. 50 + 16x = yfare for 20 km = 50 + 16(20) = y = 50 + 320 = y y = ` 370∴ fare for 20 km = ` 370.

20. 3x + 5y = 15x = 0, 3(0) + 5y = 15y = 3y = 0, 3x + 5(0) = 15x = 5

21. 2x – y = 6Put x = 1 2 – y = 6 y = 4Put x = 0 y = 6Put x = –1 –2 – y = 6 y = 8 2x – y = 6 4x + y = 24Put x = 1 4 + y = 24 y = 20Put x = 0 y = 24Put x = –4 –16 + y = 24 y = 40 4x + y = 24.

x 0 5

y 3 0

x 1 0 –1

y 4 6 8

x 1 0 –4

y 20 24 40

41 41

MATHEMATICS - 9DDITION LA APR CTICEA

Worksheet-1

Section A 1. Things which are equal to the same things are equal to one

another. 2. We can draw infinite lines passing through the point P. 3. AB = x + 3

AC = 4x – 5 BC = 2xβ‡’ AC = AB + BC 4x – 5 = x + 3 + 2x 4x – 3x = 8

x = 8.

4. Concurrent lines: Three or more lines are said to be concurrent if there is a point which lies on all of them.

Section B 5. Given: C is the mid-point of AB, such that AC = BC.

To prove: AC = 12AB

Proof: We have AC = BC ...(1)We can write AB = AC + BC AB = AC + AC (From Eq. 1) AB = 2AC

β‡’ 12AB = AC

β‡’ AC = 12AB Hence proved.

6. Given: AB = CDTo prove: AC = BDProof: We have AB = CD ...(1)

Adding BC on both sides of Eq. (1), we get AB + BC = BC + CD AC = BDAxiom: If equals are added to equals, the wholes are equal.

7.

Introduction to Euclid’s Geometry05

Chapter

Given: BX = 12

AB, BY = 12

BC and AB = BC.

To prove: BX = BYProof: We have AB = BC

BX = 12

AB β‡’ 2BX = AB

and BY = 12

BC β‡’ 2BY = BC

⇒ AB = BC 2BX = 2BY⇒ BX = BY Hence proved.

8. Let D be another mid-point of AB.

∴ AD = DB ...(1)Let C is the mid-point of AB.∴ AC = CB ...(2)Subtracting (1) from (2), we get AC – AD = CB – DBβ‡’ DC = –DCβ‡’ 2DC = 0β‡’ DC = 0∴ C and D coincide.Thus, every line segment has one and only one mid-point.Hence proved.

9. If equals are subtracted from equals, the remainders are equal. 10. AB = BC

AX + XB = BY + YCAlso, BX = BYβ‡’ (AX + XB) – BX = (BY + CY) – BY [If equals are subtracted from equal the remainders are equal)β‡’ AX = CY

Section C 11. The terms need to be defined are β†’Polygon is a closed figure bounded by three or more line

segments. β†’Line segment is a part of line with two end points. β†’Line is undefined term. β†’Point is an undefined term. β†’Angle in a figure is formed by two rays with one common

initial point. β†’Acute angle is an angle whose measure is between 0 to 90Β°.

Here undefined terms are line and point. All the angles of equilateral triangle are 60Β° each (given)Two line segments are equal to the third one (given)Therefore, all three sides of an equilateral triangles are equal (according to Euclid’s axiom, things which are equal to the same thing are equal to one another)

12. Euclid’s fifth postulate states that β€˜given any two lines and a transversal, if the angles on the same side of transversal are less than 180 degrees they will meet on that side of transversal’.

42

MATHEMATICS - 9DDITION LA APR CTICEA

The statement given here is that two intersecting lines cannot be perpendicular to the same line. i.e., parallel. This statement is stating that the co-interior angles of intersecting lines cannot be 90 degrees in measure, and it is almost equivalent to Euclid postulates.

13. We know that if a transversal intersects two parallel lines, then each pair of corresponding angles are equal, which is a theorem. So, statement (i) is false and not an axiom.

Also, we know that, if a transversal intersect two parallel line then each pair of alternate interior angles are equal. It is also a theorem.

So, statement (ii) is true and an axiom. Thus, in given statements, (i) is false and (ii) is axiom. Hence, system of axiom is not consistent.

14. We know that if two lines intersect each other, then the vertically opposite angles are equal. It is a theorem, so given system (i) is false and not an axiom.

Also, we know that, if a ray stands on a line, then the sum of two adjacent angles so formed is equal to 180Β°. It is an axiom. So, given statement (ii) is true and an axiom.

Thus in given statements, (i) is false and (ii) is true. Hence given system of axioms is not consistent. 15. The given system of axioms are consistent.

Section D 16. OX = PX

⇒ 20X = 2PX⇒ XY = XZ

17. AB = BC

12

AB = 12

BC

AM = NCSince, M is midpoint of AB

AM = 12

AB

Also, N is midpoint of BC

CN = 12 BC

(ii) Since, M is midpoint of AB

BM = 12AB

Also, N is midpoint of BC

BN = 12BC

As, BM = BN

β‡’ 12AB =

12BC

β‡’ AB = BC

Worksheet-II

Section A 1. equal 2. A straight line may be drawn from one point to another point. 3. Two lines AB and CD lying the same plane are said to be

perpendicular, if they form a right angle. AB βŠ₯ CD. 4. For every line l and for every point p not lying on l, there exists a

unique line m passing through p and parallel to l.

Section B 5. Given: Two distinct lines l and m.

To prove: l ∩ m contains at the most one point.Proof: Let l ∩ m contains two distinct points A and B.Then l contains both points A and B.And m also contains both points A and B.But, there is one and only one line passing through two distinct points.∴ l = mThis contradicts the fact.Thus our assumption is wrong.Hence, two distinct lines cannot have more than one point in common.

6. A system of axioms is called consistent, when it is impossible to deduce any statement from these axioms, which contradicts any axiom or previously proved statement.

7. We have ∠ABC = ∠ACBand ∠3 = ∠4To prove: DB = DC

Proof: We have ∠ABC = ∠ACB ...(1)and ∠3 = ∠4 ...(2)Subtracting eq. (2) from (1), we get ∠ABC – ∠4 = ∠ACB – ∠3 ∠1 = ∠2β‡’ DC = BD

(sides opposite to equal angles are equals)Hence proved.

8. PQ, PR, PS, QR, RS, QS etc. 9. Q is the mid-point of AB.

A P Q B

⇒ AQ = QB⇒ AB = AQ + QB AB = QB + QB AB = 2QB ...(1)Also, P is the mid-point of AQ.⇒ AP = PQ AQ = PQ + APand AQ = PQ + PQ AQ = 2PQ ...(2)We can write AB = AQ + BQ

43 43

05. Introd

uction

to Euclid

’s Geom

etry

MATHEMATICS - 9DDITION LA APR CTICEA

AB = 2PQ + 12

AB

AB – 12

AB = 2PQ

2

2AB ABβˆ’

= 2PQ

AB = 4PQ

14

AB = PQ

Hence proved. 10. We can say that all four lines have only are common point A.

And we can not draw 4 different lines as they are parallel to each other.But if A, B, C, D and E are on same line than they can satisfied our given condition and as well as properties of parallel line.β‡’ A, B, C, D and E are collinear.

Section C 11.

AC = 12 cm AB + BC = 12 AB + 9 = 12 AB + 9 – 9 = 12 – 9 AB = 3

12. Angle is the figure formed by two rays called the sides of the angle sharing a common endpoint called the vertex of the angle.Two lines are said to be congruent if they have the same length.

13. Equilateral triangle is a triangle with all sides equal. We prove this by geometry.

β†’ Draw a line segment AB of any length. β†’ Take compass, put the pointy end at point A and pencil at

point B. β†’ Draw an arc. β†’ Here we draw an arc of radii AB. β†’ Now put the pointy end at B and pencil at A.

β†’ Draw another arc. Here are draw an arc of β†’ Here we draw an arc of radii BA. β†’ Mark the intersecting point as C. β†’ Join point A to point C by a straight line. β†’ Joint point B to point C by a straight line.

We need to prove AB = AC = BCNow AC = AB (radii of same circle)and BC = AB (radii of same circle)From Euclid’s axiom, things which are equal to the same thing are equal,So, AC = BCSo, we get AB = AC = BC,∴ ABC is an equilateral triangle.

14. AB = PQ, PQ = XYAs, we know that things which are equal to same things are equal to one another. So, AB = XY.

15. An axiom is a rule or statement that is accepted as true without proof.Postulate is a statement that is considered to be self-evident.

Section D 16. (i) Parallel lines: Two lines in a plane that do not intersect or

touch each other at any point are said to be parallel. (ii) A line meeting another line at a right angle or 90Β° is said to

be perpendicular to it. 17. Define lines are line segments and rays.

A line extends forever in both directions.A line segment is just part of a line. It has two endpoints.A ray starts at one point and continues on forever in one direction.

44 44

MATHEMATICS - 9DDITION LA APR CTICEA

Worksheet-1

Section A 1. Let angle be = x

So, x = 25 + (90 – x) 2x = 115 x = 57.5

2. Let an angle = xΒ°According to question x = 4(180Β° – x) x = 720 – 4x x + 4x = 720 5x = 720

x = 7205

x = 144Β° 180Β° – 144Β° = 36Β°.

3. Let the angles be x and y.According to question x – y = 40Β° ...(1)and x + y = 90Β° ...(2)Adding eqns. (1) and (2), we get: 2x = 130Β° x = 65Β°Put in eq. (1) 65Β° – y = 40Β° 65Β° – 40Β° = y y = 25 x = 65Β° and y = 25Β°.

4. Let the angles be x and y.We know that x + y = 180Β° ...(1)According to question x – y = 90Β° ...(2)Adding (1) and (2) 2x = 270Β° x = 135Β°Put in (1), 135Β° + y = 180Β° y = 45Β°

x = 135Β° and y = 45Β°. 5. a and b form a linear pair.

β‡’ a + b = 180Β° ...(1)and it is given that a – 2b = 30Β° a = 30Β° + 2b ...(2)Putting this value of a in Eq. (1), we get 30Β° + 2b + b = 180Β° 3b = 150Β°

b = 50Β°

Lines and Angles06Chapter

Putting this value of b in Eq. (2) a = 30Β° + 2 Γ— 50Β° a = 30Β° + 100Β°

a = 130Β°

Section B 6. Since BO and CO are bisectors of ∠DBC and ∠ECB.

β‡’ ∠DBO = ∠CBOand ∠ECO = ∠BCO

In DABC, ∠ABC + ∠BAC + ∠ACB = 180Β° (A.S.P.) 40Β° + 70Β° + ∠ACB = 180Β°β‡’ ∠ACB = 180Β° – 110Β°β‡’ ∠ACB = 70Β°Now ∠ABC + ∠DBO + ∠CBO = 180Β° 40Β° + 2 ∠CBO = 180Β° 2 ∠CBO = 140Β° ∠CBO = 70Β°Similarly, ∠ACB + ∠BCO + ∠ECO = 180Β° 70Β° + 2∠BCO = 180Β° 2∠BCO = 110Β° ∠BCO = 55Β°In DBOC, ∠BOC + ∠CBO + ∠BCO = 180Β° (A.S.P.) ∠BOC + 70Β° + 55Β° = 180Β° ∠BOC = 180Β° – 125Β°

∠BOC = 55°

7. We have a – b = 80Β° ...(1)We know that a + b = 180Β° (C.P.)Adding eqns. (1) and (2), we get; 2a = 260Β°

a = 130Β°

ab

A BO

C

45 45

06. Lines an

d A

ng

les

MATHEMATICS - 9DDITION LA APR CTICEA

Put in (1), 130Β° – b = 80Β° b = 130Β° – 80Β°

b = 50Β°

8. Since POQ is a straight line.

So, 5y + 5y + 2y = 180Β° (L.P.) 12y = 180Β°

y = 18012

Β°

y = 15Β°

9. Let the angles be 2x and 3x.Sum of supplementary angles = 180Β°A.T.Q. 2x + 3x = 180Β° 5x = 180Β°

x = 1805

Β°

x = 36Β° 2x = 2 Γ— 36Β° = 72Β° 3x = 3 Γ— 36Β° = 108Β°.

10. x + y = w + zSince the sum of all the angles round a pair is equal to 360Β°.

(∠BOC + ∠COA) + (∠BOD + ∠AOD) = 360Β°β‡’ (x + y) + (z + w) = 360Β°But x + y = w + z (x + y) + (x + y) = 360Β° 2(x + y) = 360Β° x + y = 180Β°Thus, ∠BOC and ∠COA as well as ∠BOD and ∠AOD form linear pairs.Consequently, OA and OB are two opposite rays.Therefore, AOB is a straight line.

Section C 11. Given: Two lines AB and CD intersect at O.

To prove: ∠AOC = ∠BODand ∠AOD = ∠BOCProof: Since a ray OC stands on the line AB, ∠AOC + ∠COB = 180Β° (L.P.) ...(1)Since ray OA stands on the line CD, ∠AOC + ∠AOD = 180Β° (L.P.) ...(2)From eqns (1) and (2), we have ∠AOC + ∠COB = ∠AOC + ∠AODβ‡’ ∠COB = ∠AODSimilarly, ∠AOC = ∠BOD.Hence proved.

12. In DABC,

∠BAC + ∠ABC + ∠BCA = 180Β° (A.S.P.) x + x + x + 13 = 180Β° 3x = 180Β° – 13Β° 3x = 167Β° x = 55.7 ∠CAB = 55.7and ∠BAD = 55.7 Γ— 2 = 111.3 ∠BAC = 55.7 ∠ABC = 55.7 ∠ACB = 55.7 + 13 = 68.7

13. QT βŠ₯ PR,

∠TQR = 40Β° and ∠SPR = 30Β°In DTQR, ∠TQR + ∠QTR + ∠QRT = 180Β° (A.S.P.) 40Β° + 90Β° + ∠QRT = 180Β° ∠QRT = 50Β° x = 50Β°In DPSR, ∠RPS + x + z = 180Β° (A.S.P.) 30Β° + 50Β° + z = 180Β° z = 180Β° – 80Β°

z = 100Β°

y + z = 180Β° (L.P.) y + 100Β° = 180Β°

y = 80Β°

x = 50Β°

46

MATHEMATICS - 9DDITION LA APR CTICEA

14. AB || CD, from the given figure:

z130Β°

C D

A B

F

E G

yx

EF βŠ₯ CD, ∠GFC = 130Β° 130Β° + ∠GFD = 180Β° (L.P.) ∠GFD = 50Β°β‡’ z + 50Β° = 90Β°

z = 40Β°

In DEFG, 90Β° + y + z = 180Β° (A.S.P.) 90Β° + y + 40Β° = 180Β°

y = 50Β°

x + y = 180Β° (L.P.) x + 50Β° = 180Β°

x = 130Β°

15. AB || CD, x = ?

Draw EH || CD || ABAB || EH and GE is a transversal. ∠1 + ∠BGE = 180° (Consecutive interior angles) ∠1 + 135° = 180° ∠1 = 45°Again EH || CD, EF is a transversal. ∠2 + ∠DFE = 180° ∠2 + 125° = 180° ∠2 = 55°But x = ∠1 + ∠2 = 45° + 55° = 100°.

Section D 16.

∠POR : ∠ROQ = 5 : 7

Let ∠POR = 5x and ∠ROQ = 7xand ∠POR + ∠ROQ = 180° (L.P.) c + d = 180° 5x + 7x = 180° 12x = 180°

x = 18012

Β° x = 15Β°

β‡’ c = 7x = 7 Γ— 15 = 105Β° d = 5x = 5 Γ— 15 = 75Β°β‡’ a = c and b = d [Vertically opposite angles]β‡’ a = c = 105Β° d = b = 75Β°

17. Given: A triangle ABC.To prove: ∠A + ∠B + ∠C = 180°i.e. ∠1 + ∠2 + ∠3 = 180°Construction: Through A, draw a line l parallel to BC.Proof:

Since l || BC∴ ∠2 = ∠4 and ∠3 = ∠5 [Alternate interior angles]We know that sum of angles at a point is 180Β°.β‡’ ∠1 + ∠4 + ∠5 = 180Β°β‡’ ∠1 + ∠2 + ∠3 = 180Β°β‡’ ∠A + ∠B + ∠C = 180Β°Hence proved. ∠A + ∠B = 120Β°and ∠B + ∠C = 100Β° ∠A = 120Β° – ∠B ∠C = 100Β° – ∠B ∠A + ∠B + ∠C = 180Β° 120Β° – ∠B + ∠B + 100Β° – ∠B = 180Β° 220Β° – 180Β° = ∠B

∠B = 40°

18. TQ is a bisector of ∠PQR.

So, ∠PQT = ∠TQR = 12∠PQR.

Also, TR is bisector of ∠PRS.

So, ∠PRT = ∠TRS = 12∠PRS

47 47

06. Lines an

d A

ng

les

MATHEMATICS - 9DDITION LA APR CTICEA

In DPQR, ∠PRS is an exterior angle ∠PRS = ∠QPR + ∠PQR ...(1)

(Exterior angle property)In DTQR, ∠TRS is exterior angle ∠TRS = ∠TQR + ∠QTR ...(2)

(Exterior angle property)

Putting, ∠TRS = 12∠PRS and ∠TQR =

12∠PQR

12∠PRS =

12∠PQR + ∠QTR

12∠PRS = 1

2∠PQR + ∠QTR

Putting, ∠PRS = ∠QPR + ∠PQR [From (1)]

12

∠ + ∠( )QPR PQR = 12∠PQR + ∠QTR

12

12

∠ + ∠QPR PQR = 12∠PQR + ∠QTR

12∠QPR = ∠QTR

β‡’ ∠QTR = 12∠QPR

Hence proved.

19. Given: EF || GH.To prove: AB || CD

Proof: Since, EF || GH and intersected by a transversal PQ at F and H.∴ ∠EFP = ∠GHF 2 ∠EFP = 2 ∠GHFβ‡’ ∠AFP = ∠CHFBut these are corresponding angles formed by two lines AB and CD when transversal PQ intersects them.β‡’ AB || CDHence proved.

20. ∠ACD = 100° and ∠QAP = 35°Since ∠QAP = ∠BAC [Vertically opp. angles]

β‡’ ∠BAC = 35Β°

and ∠ACB + ∠ACD = 180Β° (L.P.) ∠ACB + 100Β° = 180Β° ∠ACB = 180Β° – 100Β°

β‡’ ∠ACB = 80Β°

In DABC, ∠ABC + ∠BAC + ∠ACB = 180Β° (A.S.P.) ∠ABC + 35Β° + 80Β° = 180Β° ∠ABC = 180Β° – 115Β°

β‡’ ∠ABC = 65Β°

Worksheet-II

Section A 1. By exterior angle property, exterior angle of a triangle is equal to

the sum of its two opposite interior angles.

β‡’ ∠ACD = ∠ABC + ∠BAC 100Β° = 60Β° + ∠BAC 100Β° – 60Β° = x

x = 40Β°

2. Let x = 1aand y = 4a

xy

m

l

Now, x + y = 180Β° (L.P.) 1a + 4a = 180Β° 5a = 180Β°

a = 1805

Β°

a = 36Β° x = 36Β° and y = 4 Γ— 36Β° = 144Β°.

3. Sum of all angles at a point = 180Β° (x + 3) + (x + 20Β°) + (x + 7) = 180Β° 3x + 30Β° = 180Β° 3x = 180Β° – 30Β° = 150Β° x = 50Β°.

48

MATHEMATICS - 9DDITION LA APR CTICEA

4. Since ABCD is a rectangle,

∠B = 90Β°Since BP bisects ∠Bβ‡’ ∠ABP = ∠PBC = 45Β° ∠APB + ∠BPC = 180Β° (L.P.) 100Β° + ∠BPC = 180Β° ∠BPC = 80Β° ∠BPC + ∠PCB + ∠PBC = 180Β° (A.S.P.) 80Β° + x + 45Β° = 180Β° x = 180Β° – 125Β°

x = 55Β°

5. PQ || RS, ∠ACS = 127°

and ∠PAB = 50Β°Since, ∠ACB + ∠ACS = 180Β° (L.P.) ∠ACB + 127Β° = 180Β° ∠ACB = 180Β° – 127Β° ∠ACB = 53Β°Since PQ || RS and AC is a transversal. ∠ACB = ∠QAC = 53Β°Now ∠PAB + ∠BAC + ∠QAC = 180Β°

(Sum of all angles at a point is 180Β°) 50Β° + ∠BAC + 53Β° = 180Β° ∠BAC = 180Β° – 103Β°

∠BAC = 77°

Section B 6. We know that:

∠ABD + ∠ABC = 180° (L.P.) 100° + ∠ABC = 180° ∠ABC = 80°Similarly, ∠ACE + ∠ACB = 180° (L.P.) 120° + ∠ACB = 180° ∠ACB = 60°In DABC, ∠ABC + ∠BAC + ∠ACB = 180° (A.S.P.)

80Β° + ∠BAC + 60Β° = 180Β° ∠BAC = 180Β° – 140Β° ∠BAC = 40Β°.

7. We know that: ∠AOC + ∠BOE = 70°Also, ∠AOC + ∠COE + ∠BOE = 180°

(Sum of all angles at a point is 180°) ∠AOC + ∠BOE + ∠COE = 180° 70° + ∠COE = 180°

∠COE = 110°

Now, ∠COE + ∠BOE + ∠BOD = 180Β° 110Β° + (70Β° – ∠AOC) + 40Β° = 180Β° 70Β° – ∠AOE = 180Β° – 150Β° 70Β° – ∠AOE = 30Β° 70°– 30Β° = ∠AOC ∠AOC = 40Β° ∠BOE = 70Β° – ∠AOC ∠BOE = 70Β° – 40Β°

∠BOE = 30°

8. PQ || RS and AC is a transversal. ∠ACS + ∠ACB = 180Β° (L.P.) 100Β° + ∠ACB = 180Β°β‡’ ∠ACB = 80Β°Also, ∠PAB = ∠ABC = 70Β° (Alternate angles)Now, in DABC ∠ABC + ∠BAC + ∠ACB = 180Β° (A.S.P.) 70Β° + ∠BAC + 80Β° = 180Β° ∠BAC = 180Β° – 150Β°

∠BAC = 30° and ∠ABC = 70°

9. ∠APQ = 60Β° and ∠PRD = 137Β°Since AB || CD and PQ is a transversal.β‡’ ∠APQ = ∠PQR (alternate angles)β‡’ x = 60Β°Now ∠PRQ + ∠PRD = 180Β° (L.P.) ∠PRQ + 137Β° = 180Β° ∠PRQ = 43Β°In DPQR,β‡’ ∠PQR + ∠QPR + ∠PRQ = 180Β° (A.S.P.) x + y + 43Β° = 180Β° 60Β° + y + 43Β° = 180Β° y + 103Β° = 180Β° y = 180Β° – 103Β° y = 77Β° x = 60Β° and y = 77Β°.

10. We have, ∠AOC + ∠BOD = 70°

Also, we know that sum of all angles at a point is 180Β°.β‡’ ∠AOC + ∠BOD + ∠COD = 180Β°β‡’ 70Β° + ∠COD = 180Β°β‡’ ∠COD = 180Β° – 70Β°

β‡’ ∠COD = 110Β°

49 49

06. Lines an

d A

ng

les

MATHEMATICS - 9DDITION LA APR CTICEA

Section C 11. ∠PRT = 40° and ∠RPT = 95°

In DPRT, ∠PRT + ∠RPT + ∠PTR = 180Β° (A.S.P.) 40Β° + 95Β° + ∠PTR = 180Β° ∠PTR = 180Β° – 135Β° ∠PTR = 45Β°Now, ∠PTR = ∠QTS = 45Β° (V.O.A.)In DTQS, ∠TQS + ∠TSQ + ∠QTS = 180Β° (A.S.P.) ∠SQT + 45Β° + 75Β° = 180Β° ∠SQT = 180Β° – 120Β°

∠SQT = 60°

12. An exterior angle of a triangle is equal to the sum of two interior opposite angles of the triangle.Let the other interior angle = xAccording to question x + 30Β° = 110Β° x = 110Β° – 30Β° x = 80Β°.

13. Since OR βŠ₯ PQ,Hence ∠ROP = 90Β° and ∠ROQ = 90Β°

We can say that ∠ROP = ∠ROQ ∠POS + ∠ROS = ∠ROQ ∠POS + ∠ROS = ∠QOS – ∠ROS ∠SOR + ∠ROS = ∠QOS – ∠POS 2 ∠ROS = ∠QOS – ∠POS

∠ROS = 12

(∠QOS – ∠POS)

Hence proved.

14.

A

B

C D

2

4

1

P

x y

3

Given: AB || CD and P is any point between AB and CD.To prove: ∠ABP + ∠CDP = ∠DPBConstruction: Draw a line xy || AB and xy || CD.Proof: Since AB || xy ∠1 = ∠2 (Alternate int. angles) ...(1)Also xy || CD ∠3 = ∠4 (Alternate int. angles) ...(2)Adding Eqns. (1) and (2), we get ∠1 + ∠3 = ∠2 + ∠4 ∠ABP + ∠CDP = ∠DPBHence proved.

15. AD βŠ₯ AB and AD || BC. ∠BDC = 30Β° and x : y = 11 : 19Let x = 11a and y = 19aIn DABD, ∠ABD + ∠BAD + ∠ADB = 180Β° (A.S.P.) y + 90Β° + x = 180Β° 19a + 90Β° + 11a = 180Β° 30a = 90Β° a = 3∴ x = 11 Γ— 3 = 33Β° y = 19 Γ— 3 = 57Β°Now, ∠ADC = ∠DCE

(AD || BC and DC is a transversal, so alt. angles are equal)Now, ∠ADC = x + 30° = 33° + 30° = 63°

β‡’ ∠DCE = 63Β°

Section D 16. We know that sum of three angles of a triangle is 180Β°.

So, (2x – 7)Β° + (x + 25)Β° + (3x + 12)Β° = 180Β° 6x + 30Β° = 180Β° 6x = 150Β° x = 25Β°Angles are ∠A = 2x – 7 = 2 Γ— 25 – 7 = 50Β° – 7 = 43Β° ∠B = x + 25Β° = 25Β° + 25Β° = 50Β° ∠C = 3x + 12Β° = 3 Γ— 25 + 12Β° = 75 + 12Β° = 87Β°.

17. Since POQ is a straight line.OS bisects ∠POR and OT bisects ∠QOR.

To prove: ∠SOT is a right angle.Proof: We know that ∠POR + ∠QOR = 180ΒΊ (L.P.)Since OS bisects ∠PORβ‡’ ∠POS = ∠ROSAlso, ∠POS + ∠ROS = ∠PORβ‡’ 2 ∠ROS = ∠POR ...(1)Similarly OT bisects QORβ‡’ ∠QOT = ∠ROTAlso, ∠QOT + ∠ROT = ∠QORβ‡’ 2 ∠ROT = ∠QOR ...(2)Now, ∠POR + ∠QOR = 180ΒΊFrom eqns. (1) and (2), we get; 2 ∠ROS + 2 ∠ROT = 180Β° 2 (∠ROS + ∠ROT) = 180Β°

∠ROS + ∠ROT = 1802

Β°

β‡’ ∠ROS + ∠ROT = 90Β°and ∠SOT = ∠ROS + ∠ROT = 90Β°β‡’ ∠SOT = 90Β°β‡’ ∠SOT is a right angle.Hence proved.

50

MATHEMATICS - 9DDITION LA APR CTICEA

18. Given: Mirror PQ || Mirror RS. An incident ray AB strikes the mirror RQ at B, the reflected

ray moves along BC and strikes the mirror RS at C and again reflects back along CD.

To prove: AB || CD

Construction: Draw BN βŠ₯ PQ and CM βŠ₯ RS.Since BN || CMβ‡’ ∠3 = ∠2β‡’ 2∠3 = 2∠2β‡’ ∠3 + ∠3 = ∠2 + ∠2β‡’ ∠3 + ∠4 = ∠2 + ∠1 ∠ABC = ∠BCD AB || CD(As corresponding angles are equal, lines are paralleland ∠3 = ∠4 and ∠1 = ∠2as angle of incidence equal to angle of reflection.)

51 51

MATHEMATICS - 9DDITION LA APR CTICEA

Worksheet-1

Section A 1. AB = AC, ∠B = ∠C, x = 50°

We know that: ∠A + ∠B + ∠C = 180Β° 50Β° + 50Β°+ ∠A = 180Β° 100Β° + ∠A = 180Β° ∠A = 180Β° – 100Β° = 80Β° = yΒ°\ x = 50Β°, y = 80Β°

2. ∠ABD : ∠ACDAs AB = AC\ ∠ABC = ∠ACB

A

B C

D

then 12

(∠ABC) = 12

(∠ACB)

β‡’ ∠ABD = ∠ACD i.e. 1 : 1See the figure.

3. In DABD, we get: ∠BAD + ∠ABD + ∠ADB = 180Β° ∠BAD + 35Β° + 90Β° = 180Β° ∠BAD + 125Β° = 180Β° ∠BAD = 180Β° – 125Β° ∠BAD = 55Β°

4.

Triangles07Chapter

From DABC and DDEF, DABC β‰… DDEFBecause AB = DE BC = EF AC = DF From, S.S.S. congruent rule. DABC β‰… DDEF

5.

∠B = 180 – x ∠C = 180 – yNow, ∠B > ∠Cβ‡’ 180 – x > 180 – yβ‡’ –x > – yβ‡’ x < y

Section B 6.

Since AB = AC ∠B = ∠CNow in DABC ∠A + ∠B + ∠C = 180 60 + ∠B + ∠C = 180 ∠B + ∠C = 120 ∠B + ∠B = 120 2∠B = 120 ∠B = 60 ∠C = ∠B = 60Since each of the angle of DABC is of 60° so it is an equilateral triangle.

7. If the sum of 2 angles of a DABC is equal to its third angle. Find the third angle.

52

MATHEMATICS - 9DDITION LA APR CTICEA

Given x + y = zIn DABC, x + y + z = 180Β° z + z = 180Β° 2z = 180Β°\ Third angle is z = 90Β°

8. x > y β‡’ – x < – yβ‡’ (180Β° – x) < (180Β° – y)β‡’ ∠ABC < ∠ACBβ‡’ ∠ACB > ∠ABCβ‡’ AB > AC [ Side opposite to larger angle is larger.]

Hence, AB > AC 9. Consider DABC and DABD

AC = AD (Given) ∠CAB = ∠DAB (Since AB bisects ∠A) AB = AB (common)\ DABC β‰… DABD (SAS)β‡’ BC = BD (CPCT)Hence Proved.

10.

In DPQR PQ + QR > PR ...(1)In DRQS SR + RQ > SQ ...(2)In DPSR, SR + PS > RP ...(3)[Since in sum of two sides is greater than the third side]In DPSQ, PS + PQ > SQ ...(4)Add (1), (2), (3), (4)PQ + QR + SR + RQ + SR + PS + PS + PQ > PR + SQ + RP + SQ⇒ 2PQ + 2QR + 2SR + 2PS > 2SQ + 2PR⇒ 2 (PQ + QR + SR + PS) > 2 (PR + SQ)⇒ PQ + QR + SR + PS > PR + SQHence Proved.

Section C 11. From the given figure: a. In DAPB and DCQD

∠CDQ = ∠ABQ [Alternate angles] ∠P = ∠Q AB = CD

\ DAPB β‰… DCQD [ AAS congruent rule]b. AP = CQ [C.P.C.T.]

12. From the given figure:

In DADB and DADC, we get: BD = DC [AD is the median of a triangle] AD = AD [Common line segment] ∠ADB = ∠ADC\ DABD β‰… DADC [By SAS congruent rule]Hence Area (DABD) = Area (DADC)Hence Proved.

13. ∠DCA = ∠ECBβ‡’ ∠DCA + ∠DCE = ∠ECB + ∠DCEβ‡’ ∠ACE = ∠DCBAlso, ∠A = ∠B, AC = BCβ‡’ DDBC β‰… DEAC (ASA) DC = EC (CPCT)Hence Proved.

14. ∠PQR = ∠S + ∠SPQSince ∠PQR = ∠Rβ‡’ ∠R = ∠S + ∠SPQβ‡’ ∠R > ∠S PS > PRHence Proved.

15. From given figure:

AB = ACβ‡’ ∠ACB = ∠ABC

β‡’ 12∠ACB =

12∠ABC

β‡’ ∠OCB = ∠OBC In DBOC, we have

∠OBC + ∠OCB + ∠BOC = 180Β°β‡’ 2∠OBC + ∠BOC = 180Β°β‡’ ∠ABC + ∠BOC = 180Β°β‡’ 180Β° – ∠DBA + ∠BOC = 180Β°β‡’ ∠DBA = ∠BOC

53 53

07. Triang

les

MATHEMATICS - 9DDITION LA APR CTICEA

Section D 16. Construction: Join PR

To prove: ∠R > ∠PProof: Since PQ is the longest sideβ‡’ PQ > QRβ‡’ ∠3 > ∠1 ...(1)

Also, Since SR is the shortest sideβ‡’ SR < PSβ‡’ PS > SRβ‡’ ∠4 > ∠2 ...(2)Add (1) and (2) ∠3 + ∠4 > ∠1 + ∠2 ∠R > ∠PSimilarly ∠S > ∠Q

17. Given: DABC in which AD is median.

A

B CD

E

To prove: AB + AC > 2AD Construction: Produce AD to E Such that AD = DE. Join EC Proof: In D ADB and DEDC, we get;

AD = DE [By construction] ∠ADB = ∠EDC [vert. opp. angles] BD = CD [ D is the mid-point of BC]\ DADB β‰… DEDC [By SAS criteria]\ AB = EC [C.P.C.T.] EC + AC > AE [Sum of 2 sides of a D is greater

than the III side] AB + AC > 2AD [EC = AB, AE = 2AD]Hence AB + AC > 2AD

18. Given: DABC in which AB = AC

To Prove: ∠B = ∠C Construction: Draw AD, the bisector of ∠A, to meet BC in

D.

Proof: In DABD and DACD, we get AB = AC (given) AD = AD (Common line segment) ∠BAD = ∠CAD (By Construction)\ DABD β‰… DACD [ SAS criteria]

Hence, ∠B = ∠C (C.P.C.T.)Hence Proved.

To Prove: ∠ABD = ∠ACDIn DABC, AB = AC β‡’ ∠ABC = ∠ACB ...(1)In DDBC, DB = DC β‡’ ∠DBC = ∠DCB ...(2)Add (1), (2) β‡’ ∠ABC + ∠DBC = ∠ACB + ∠DCB β‡’ ∠ABD = ∠ACD

Hence Proved. 19. ...(1)

Since AB = ACβ‡’ ∠1 = ∠2Also, AB = AD ...(2)From (1), (2)β‡’ AC = ADβ‡’ ∠3 = ∠4Now in DBCD ∠B + ∠C + ∠D = 180Β° (Angle Sum property) ∠1 + ∠2 + ∠3 + ∠4 = 180Β° 2∠2 + 2∠3 = 180Β° 2(∠2 + ∠3) = 180Β° ∠2 + ∠3 = 90Β° ∠C = 90Β°β‡’ ∠BCD is a right angleHence Proved.

20. In right DABC, ∠B = 90°

Then ∠A + ∠C = 90Β°\ ∠B > ∠A and ∠B > ∠Cβ‡’ AC > BC and AC > AB [side opposite to larger angle is

longer]\ AC is the longest sideHence, in a right triangle, the hypotenuse is the longest side.Hence Proved.

54

MATHEMATICS - 9DDITION LA APR CTICEA

Worksheet-II

Section A 1. DABC isosceles right angled triangle in which

∠A = 90Β° ∠A + ∠B + ∠C = 180Β° 90Β° + ∠B + ∠B = 180Β° 2∠B = 180Β° – 90Β°= 90Β° ∠B = 45Β°, ∠C = 45Β°

2.

To find: AD

Solution AD = DB = 12

(12) AD = 6 cmIn DPDA AP2 = PD2 + AD2

(10)2 = PD2 + (6)2

100 = PD2 + 36 PD2 = 100 – 36 = 64 PD = 8 cm

3. Yes, Sides 8 cm, 7 cm, 4 cm DABC is constructed because the sum of two sides is greater

than third 7 + 4 > 8 11 cm > 8 cm

4. In ∠ABC,

∠A = ∠C [AB = BC]In DABC, ∠A + ∠B + ∠C = 180Β° ∠A + 80Β° + ∠A = 180Β° 2∠A = 180 – 80 = 100 ∠A = 50Β°, ∠C = 50Β°

Section B 5. Since AB = AD β‡’ ∠ABD = ∠ADB

Also BC = CD β‡’ ∠CBD = ∠CDB [Angle opposite to equal sides are equal]So, ∠ABD + ∠CBD = ∠ADB + ∠CDBβ‡’ ∠B = ∠Dβ‡’ ∠ABC = ∠ADC

6. From the given figure: ∠ACD = 120Β° AB = AC [given] ∠B = ∠C ∠ABC = ∠ACB = 60Β°In DABC, ∠BAC + ∠ABC + ∠ACB = 180Β° ∠A + 60Β° + 60Β° = 180Β° ∠A +120Β° = 180Β° ∠A = 180Β° – 120Β° = 60Β° ∠A = 60Β°

7.

Since BP bisects ∠Bβ‡’ ∠1 = ∠2Also, ∠2 = ∠3 (Alternate interior angles)\ ∠1 = ∠3β‡’ BQ = PQ (Sides opposite to equal angles)β‡’ DBQP is isosceles triangle.

8. Prove that: PS > PQ

When altitude is equal to one of its side then opposite side of an altitude is greater.Given PQ = PR

where PQ is a altitude.\ PQ < PSHence Proved.

Section C 9.

55 55

07. Triang

les

MATHEMATICS - 9DDITION LA APR CTICEA

Since AD = AEβ‡’ ∠ADE = ∠AED [Angle opposite equal sides]β‡’ 180 – ∠ADB = 180 – ∠AECβ‡’ ∠ADB = ∠AECAlso, AD = AE, BD = CEβ‡’ DABD β‰…DACE [SAS]

10. Consider DDAB and DCBA AD = BC (Given) BD = AC (Given) AB = AB (Common)β‡’ DDAB β‰… DCBA (SSS)β‡’ ∠DAB = ∠CBA (CPCT)

11. AD = AC (Given)β‡’ ∠ACD = ∠ADC (Angles opposite to equal sides)Now, ∠ADC is exterior angle for DABD ∠ADC = ∠ABD + ∠BADβ‡’ ∠ADC > ∠ABD ∠ACD > ∠ABD AB > AC\ AB > AD (As AC = AD)

12. Consider DAEB and DAFC

∠A = ∠A (Common) ∠AEB = ∠AFC = 90Β° BE = CF (Given)β‡’ DAEB β‰… DAFC (AAS)β‡’ AB = AC ...(1) (CPCT)Similarly DAFC β‰… DBECβ‡’ AC = BC ...(2) (CPCT)By (1), (2) AB = BC = ACβ‡’ DABC is equilateral.Hence Proved.

Section D 13. Since ∠DBC = ∠DCB

β‡’ DB = DC (Sides opposite to equal angles) AD = AD (Common) AB = AC (Given)β‡’ DADB β‰… DADC (SSS)β‡’ ∠DAB = ∠DAC (CPCT)β‡’ AD bisects ∠BAC of ∠ABC

14. Given: ∠C = 90Β°, from given figure M is the mid-point of AB. PM = CMProve that: (a) DAMC β‰… DBMD (b) DDBC β‰… DACBProof:

(a) In D AMC and DBMD, we get: ∠DMB = ∠AMC [Vertically opp. angles] BM = MA [Given] MC = DM [Given]

\ DAMC β‰… DBMD [  SAS congruent criteria] (b) Since DAMC β‰… DBMD

β‡’ ∠MAC = ∠MBD [CPCT]β‡’ DB || AC [Since alternate angles are equal]β‡’ ∠B + ∠C = 180β‡’ ∠B + 90 = 180

β‡’ ∠B = 90Β°Consider DDBC and DACB DB = AC [Since DAMC β‰… DBMD So, DB = AC by CPCT] ∠B = ∠C = 90Β° BC = BC [Common]

β‡’ DDBC β‰… DACB [SAS]Hence Proved.

15. (a) Consider DABM and DPQN AB = PQ [Given] AM = PN [Given]

BC = QR β‡’12

BC = 12

QR

[Since AM and PN are Medians]β‡’ BM = QN

\ DABM β‰… DPQN [SSS]β‡’ ∠B = ∠Q [CPCT](b) Consider DABC and DPQR AB = PQ [Given] ∠B = ∠Q [Proved above] BC = QR [Given]β‡’ DABC β‰… DPQR (SAS)

56 56

MATHEMATICS - 9DDITION LA APR CTICEA

Worksheet-1

Section A 1.

∠D = ∠B = 110°[Since opposite angles of parallelogram are equal].

2. 110 + 85 + 45 + 65 = 305 But sum of angles of a quadrilateral is 360Β°. So, these can not be the angles of a quadrilateral. 3. Yes, if three angles of quadrilaterals are equal then the obtained

quadrilateral is parallelogram. 4.

∠K = ∠M (Opposite angles of parallelogram) ∠K + ∠L = 180Β° (Co-interior angles) 45Β° + ∠L = 180Β° ∠L = 180Β° – 45Β° = 135Β° ∠N = ∠L (Opposite angles of parallelogram) ∠N = 135Β°.

5. Perimeter = 4 + 6 + 4 + 6 = 20 cm∴ Perimeter of a kite is 20 cm.

Section B 6.

Given: Let ABCD is parallelogram where AC = BD.To prove: ABCD is rectangle.Proof: Rectangle is a parallelogram with one angle is 90Β°, we have to prove one of its interior angle is 90Β°.In βˆ†ABC and βˆ†DCB, we get: AB = DC (Opp. side of parallelogram) BC = BC (Common)

Quadrilaterals08Chapter

AC = DB (Given) βˆ†ADC β‰… βˆ†DCB (SSS congruence rules) ∠ABC = ∠DCB (C.P.C.T.) ...(2)Now AB || DC

(Opp. side of parallelogram are parallel)and BC is transversal line ∠B + ∠C = 180°(Interior angle on same side of transversal are supplementary) ∠B + ∠B = 180° [from (1)]

∠B = 1802

Β° = 90Β°

ABCD is parallelogram with one angle 90°.∴ ABCD is rectangle.

7. Let a = 3x, b = 5x, c = 9x, d = 13xSum of angle of quadrilateral is 360Β°. a + b + c + d = 360Β° 3x + 5x + 9x + 13x = 360Β° 30x = 360Β° x = 12Β° a = 3 Γ— x = 3 Γ— 12 = 36Β° b = 5 Γ— x = 5 Γ— 12 = 60Β° c = 9 Γ— x = 9 Γ— 12 = 108Β° d = 13 Γ— x = 13 Γ— 12 = 156°∴ Angles of quadrilateral are 36Β°, 60Β°, 105Β°, 156Β°.

8. ∠C and ∠D are adjacent angle of parallelogram. From given figure, ∠C + ∠D = 180° x + 75° = 180° x = 105° y = x [Alt. int. angle] x + y = 105° + 105° = 210°.

9. Sum 4x + 7x + 15x + 10x = 360Β° 36x = 360Β°

x = 36036

= 10

x = 10Β°Smallest angle = 4 Γ— x = 40Β°Largest angle = 15 Γ— x = 150Β°.

Section C 10.

Given: ABCD is quadrilateral P, Q, R, S are mid-point of side AB, BC, CD, DA.To prove: PR and SQ bisect each otherConstruction: Join A and C.Proof: In βˆ†ADC,

57 57

08. Qu

adrilaterals

MATHEMATICS - 9DDITION LA APR CTICEA

S is mid-point of AD and R is mid-point of CD.Line segment joining the mid-point of two sides AD and CD is parallel to 3rd side and half of it.

∴ SR || AC and SR = 12

AC ...(1)In βˆ†ABCP is mid-point of AB and Q is mid-point of BC.Line segment joining the mid-point of two sides AB and BC is parallel to 3rd side and half of it.

∴ PQ || AC and PQ = 12

AC ...(2)

From (1) and (2)

PQ = SR and PQ || SRIn PQRSOne pair of opp. side is parallel and equal.Hence PQRS is parallelogram.Hence proved.

11. Given: ABC is triangle D, E, F are mid-points of side AB, BC and CA.To prove: βˆ†ABC is divided into 4 congruent triangles.Proof. D and F are mid-points of side. AB and AC of βˆ†ABC. DF || BC

(Line segment joining the mid-point of two sides of triangle is parallel to 3rd side)

Similarly we can write: DE || AC and EF || ABNow in DBEF DF || BE

[Since DF || BC parts of parallel lines are parallel] DB || EF

[Since EF || AB parts of parallel lines are parallel]Since both pair of opp. side are parallelDBEF is parallelogram.DBEF is a parallelogram and DE is a diagonal.∴ βˆ†DBE β‰… βˆ†DFE[Diagonal of parallelogram divide to two congruent triangles]

...(1)Similarly, DFCE is parallelogram βˆ†DFE β‰… βˆ†CEF ...(2)Similarly, βˆ†ADF β‰… βˆ†DFE ...(3)From (1), (2), (3), we get: βˆ†DBE β‰… βˆ†DFE β‰… βˆ†CEF β‰… βˆ†ADFAll 4 triangles are congruent.

12.

Given: ABCD is rhombus.To prove: i. AC bisect ∠A and ∠C.i.e. ∠1 = ∠2and ∠3 = ∠4ii. BD bisects ∠B and ∠D.Proof: In βˆ†ADC AB = BC (Side of rhombus are equal)So, ∠4 = ∠2

(Angle opp. to equal sides are equal) ...(1)

Now, AD || BC(Opp. side of rhombus are parallel and transversal AC)

∠1 = ∠4 (Alt. angle) ...(2)Now AB || DC (Opposite sides of rhombus are parallel)and transverse AC, ∠2 = ∠3 (alternate angle) ...(3)From (1) and (3), we get: ∠3 = ∠4AC bisect ∠CFrom (1) and (2), we get: ∠1 = ∠2AC bisect ∠A.Hence, AC bisect ∠C and ∠ASimilarly we can prove that BD bisect ∠B and ∠D.Hence proved.

13.

Given: ABCD is parallelogram with AC as its diagonal.To prove: βˆ†ABC β‰… βˆ†ADC.Proof: Opp. sides of parallelogram is parallel.So, AB || DC and AD || BCSince AB || DC and AC is transversal. ∠BAC β‰… ∠DCA (Alt. angle) ...(1)Since AD || BC and AC is transversal. ∠DAC β‰… ∠BCA (Alt. angle) ...(2)In βˆ†ADC and βˆ†ABC ∠BAC = ∠DCA From (1) AC = AC (Common) ∠DAC = ∠BCA From (2)∴ βˆ†ABC β‰… βˆ†ADC (ASA) Congruency)Hence proved.

14.

Given: ABCD is parallelogram.To prove: ∠AOD = 90°Proof: ∠DAB + ∠ADC = 180°

(Angles on same side of transversal are supplementary)

12

12

∠ + ∠DAB ADC = 90°

[(∠DAB + ∠ADC)] = 90°

∠1 + ∠2 = 90Β° ∠OAD + ∠ODA = 90Β°In βˆ†ODA, we get: ∠1 + ∠2 + ∠AOD = 180Β° 90Β° + ∠AOD = 180Β° ∠AOD = 90Β°.

58

MATHEMATICS - 9DDITION LA APR CTICEA

15.

AD = BC = 2x CD = AB = 2x + 2 [Opposite sides of parallelogram are equal]Now, perimeter = 40 cm AB + BC + CD + AD = 402x + 2 + 2x + 2x + 2 + 2x = 40 8x + 4 = 40 8x = 36

x = 368 =

92

x = 92

= 4.5

Section D 16. From mid-point theorem:

The line segment joining the mid-points. of two sides of βˆ† is parallel to third side.Given: ABC is βˆ†E and F are mid-points of AB and AC.To prove: EF || BC.Construction: Through C draw a line segment parallel to AB and extend EF to meet this line at D.Proof: Since AB || CD (As AB || CD) with transversal ED. ∠BAC= ∠ACD (Alt. angle) ...(1)In βˆ†AEF and βˆ†CDF, we get: ∠BAC = ∠ACD From (1) AF = CF (F is mid-point AC) ∠AFE = ∠CFD (Vertically opposite angles)∴ βˆ†AEF β‰…βˆ†CDF (AAS)So, EA = DC (CPCT)But EA = EB (E is mid-point of AB)Hence, EB = DCNow in quadrilateral EBCD EB || CD and EB = DCThus, one pair of opp. sides is equal and parallel. Hence EBCD is parallelogram. Since opp. sides of parallelogram are parallel ED || BC.∴ EF || BCHence proved.

17. (a) MD ^ AC As MD || BC and AC is transversal

∠MDC + ∠BCD = 180° (Int. angle on same side of transversal are supplementary) ∠MDC + 90° = 180° ∠MDC = 90° MD ^ AC.

(b) D is mid-point of AC.

Given: ABC is a triangle right angled at C. ∠C = 90Β°.M is mid-point of AB MD || BCTo prove: D is mid-point of AC,Proof: In βˆ†ABC,M is mid-point of AB

MD || BC.D is mid-point of AC.

(Line through mid-point of one side of triangle, parallel to another

side bisect the third side)

(c) CM = MA = 12

AB

Joints MC, in βˆ†AMD and βˆ†CMD

AD = CD(Proved in part (1) D is mid-point of AC)

∠ADM = ∠CDM (Both 90Β° MD ^ DC) DM = DM (Common) βˆ†AMD β‰… βˆ†CMD (SAS) AM = CM (C.P.C.T.) ...(1)

AM = 12

AB [Since M is midpoint of AB] ...(2)

CM = AM = 12

AB. [From (1) and (2)]

18. From the given figure:

A B

CD

P

QS

R

Given: ABCD is parallelogram, AP, BP, DR and CR are angle bisectors of ∠A, ∠B, ∠C, ∠D.To prove: PQRS is rectangle.Proof: A rectangle is parallelogram with one angle 90° first we will prove PQRS is parallelogram.Now, AB || CD

(Opp. side of parallelogram are parallel) ∠A + ∠D = 180°(Int. angle one same side of transversal are supplementary).Multiply by half:

12

12

∠ + ∠A D = 90°

∠DAS + ∠ADS = 90°

(DR bisect ∠D and AP bisect ∠A) ...(2)

Now, in βˆ†ADS: ∠DAS + ∠ADS + ∠DSA = 180Β°

59 59

08. Qu

adrilaterals

MATHEMATICS - 9DDITION LA APR CTICEA

90° + ∠DSA = 180° ∠DSA = 90°Also, line AP and DR intersect.So, ∠PSR = ∠DSA (Vertically opp. angle)∴ ∠PSR = 90°Similarly, we can prove that ∠SPQ = ∠PQR = ∠SRQ = 90°So, ∠PSR = ∠PQR and ∠SPQ = ∠SRQBoth pair of opp. angle of PQRS are equal.So, PQRS is parallelogram ∠PRS = ∠PQR = ∠SPQ = ∠SPQ = 90°PQRS is parallelogram in which one angle 90°.PQRS is rectangle.

19.

Consider βˆ†AOD and βˆ†COB OA = OC OD = OB ∠AOD = ∠BOC (vertically opposite angle)β‡’ βˆ†AOD β‰… βˆ†COB (SAS)β‡’ AD = BC (CPCT) ...(1)Similarly βˆ†AOB β‰…βˆ†CODβ‡’ AB = CD (CPCT) ...(2)By (1), (2) we get ABCD is a parallelogram [Since opposite sides are equal]In βˆ†AOB and βˆ†COB AO = OC, ∠AOB = ∠COB, OB = OBβ‡’ βˆ†AOB β‰… βˆ†COB (SAS)β‡’ AB = BC ...(3)By (1), (2), (3) AB = BC = CD = ADIn βˆ†DAB and βˆ†CBA, AD = BC By (1) AC = BD (Given) AB = AB (common) βˆ†DAB β‰… βˆ†CBAβ‡’ ∠A = ∠B = 90 (CPCT)

20.

Given: ABCD is rectangle.AC bisect ∠A ∴ ∠1 = ∠2AC bisect ∠C ∴∠3 = ∠4To prove: ABCD is square.Proof: A square is rectangle when all sides are equal.Now, AD || BC

(Opp. side of parallelogram are parallel and AC is transversal)

∠1 = ∠4 (Alt. angle)Also, ∠1 = ∠2 (Given)

Hence ∠4 = ∠2In βˆ†ABC, ∠2 = ∠4 BC = AB

(Side opp. to equal angle are equal) ...(1)But BC = AD and AB = DC

(Opp. side of rectangle) ...(2)From (1) and (2), we get: AB = BC = CD = DASo, ABCD is square.Hence proved.

Worksheet-II

Section A 1. ∠P – ∠R = 0 because two opp. angle of parallelogram are same. 2. No, all the angles of quadrilateral can’t be obtuse. Obtuse angle

is greater than 90Β° and sum of all angles are 360Β°. 3. 2x + 16 = 96 – x

[Since opposite angles of parallelogram are equal) 2x + x = 96 – 16 3x = 80

x = 803

Section B2 : 4 : 5 : 7 angles of quadrilateral

4. 2x + 4x + 5x + 7x = 360Β° 18x = 360Β°

x = 20Β°

2 Γ— 20 = 40Β° 7 Γ— 20 = 140Β° 5 Γ— 20 = 100Β°. 4x = 4 Γ— 20 = 80Β°

5. 4x + 5x = 180Β° 9x = 180Β°

x = 20Β°

∠A = 4 Γ— 20 = 80Β° ∠B = 5 Γ— 20 = 100Β°.

6. Let length of short side = x cm P = 382(Sum of adjacent side) = 38 2(long + short) = 38 2(11 + x) = 38

11 + x = 382

= 19

x = 19 – 11 x = 8∴ Length of shorter side is 8 cm.

Section C 7.

60

MATHEMATICS - 9DDITION LA APR CTICEA

Given: ABCD is a parallelogram and AC and BD are diagonal intersecting at O.Hence OA = OC and OB = OD

(Since diagonal bisect each other)Consider βˆ†AOL and βˆ†COM: ∠AOL = ∠COM (Vertically opp. angle) OA = OC (Given) ∠OAL = ∠OCM (Alt. angle) βˆ†AOL β‰… COM (By ASA)Hence OL = OM (C.P.C.T.)

8.

A B

CD

P

Q

Given: ABCD is parallelogram with AP ^ BD and CQ ^ BDTo Prove: AP = CQProof: Now AB || DC (Opp. side of parallelogram

are parallel and transversal BD) ∠ABP = ∠CDQ (Alt angles) ...(1)In βˆ†APB and βˆ†CQD, we get; ∠APB = ∠CQD (Both 90Β°) ∠ABP = ∠CDQ (From (1)) AB = CD (Opp. side of parallelogram

are equal)∴ βˆ†APB = βˆ†CQD (AAS congruence rule) AP = CQ (C.P.C.T.)

Hence proved. 9.

Given: ABCD is a square. Diagonals intersect at O.To prove: AC = BDand AC is perpendicular to BDProof: βˆ†ABC and βˆ†DCB AB = DC (Sides of square are equal) ∠ABC = ∠DCB (Both 90Β° as all angles of

square are 90Β°) BC = BC (Common) βˆ†ABC β‰… βˆ†DCB (SAS)β‡’ AC = DB (C.P.C.T.) ...(1)Hence diagonal of square are equal in length.To prove that diagonal bisect each other AO = CO, BO = DOIn βˆ†AOB and βˆ†COD ∠AOB = ∠COD (Vertically opp. angle) ∠ABO = ∠CDO (AB || CD and BD as transvers all alternate angles equal) AB = CD βˆ†AOB β‰… βˆ†COD (AAS) ...(2) AO = CO and OB = OD (CPCT)Hence diagonals bisect each other.

In βˆ†AOB and βˆ†COB OA = OC (from (2)) AB = BC (Side of square are equal) BO = BO (Common) βˆ†AOB β‰… βˆ†COB (SSS)∴ ∠AOB = ∠COB (C.P.C.T.) ...(3)Now, ∠AOB + ∠COB = 180Β° (Linear pair) ∠AOB + ∠AO = 180Β° (from (3)) 2∠AOB = 180Β° ∠AOB = 90Β°Hence AC and BD bisect each other at right angle.

10.

Considerβˆ†FEB and βˆ†DEC BE = CE (Given) ∠BFE = ∠CDE (Alternate angles) ∠BEF = ∠CED (Vertically opposite angles) βˆ†FEB β‰… βˆ†DEC (AAS) CD = FB (CPCT)But CD = AB (Opposite sides of FB = AB parallelogram)

AB = 12

AF (1)

In βˆ†AFD, the angle bisector AE is also the median [FE = ED by CPCT]β‡’ βˆ†AFD is isosceles(A triangle in which angle bisector is also a median is isosceles triangle) AF = AD ...(2)

By (1), (2) AB = 12AF =

12AD

AB = 12AD

Section D 11.

Given: ABCD is parallelogram where E and F are the mid-points of sides AB and CD respectively.To prove: AF and EC trisect BD BQ = QP = DPProof: ABCD is a parallelogram AB || CD (Opp. sides of parallelogram

are parallel) AE || CF (Part of parallel lines are

parallel)

61 61

08. Qu

adrilaterals

MATHEMATICS - 9DDITION LA APR CTICEA

and AB = CD (Opp. sides of parallelogram are equal)

12

AB = 12

CD (Given F is mid-point of CD and E is mid-point of AB)

AE = CFIn AECF, AE || CF and AE = CFOne pair of opp. sides is equal and parallel. AECF is a parallelogram.β‡’ AF || CE (Opp. side of parallelogram are parallel)∴ PF || CQ and AP || EQ

(Part of parallel lines are parallel)In βˆ†DQC, F is mid-point of DC and PF || CQ

(Line drawn through mid-point of one side of triangle, parallel to another side bisect third side. P is mid-point of DQ)

PQ = DP ...(1)In βˆ†ABP, E is mid-point of AB and AP || EQ

(Line drawn through mid-point of one side of triangle, parallel to another side bisect third side.)

Q is mid-point of BP⇒ PQ = QB ...(2)From (1) and (2) DP = PQ = DQHence line segment AF and EC trisect the diagonal BD.

12.

Since ABCD is a parallelogram. ∠A = ∠C (opposite angles of parallelogram)

12∠A =

12 ∠C

∠1 = ∠2 = ∠3 = ∠4 ...(1)[Since AP is bisector of ∠A and CQ is bisector of ∠C]Consider βˆ†ADP and βˆ†CBQ AD = BC (opposite sides of parallelogram) ∠1 = ∠4 (Proved in (1)) ∠B = ∠D (opposite angles of parallelogram)β‡’ βˆ†ADP β‰… βˆ†CBQ (ASA) DP = BQP and Q are midpoints of sides CD and AB respectively.So, we get AQ || PC [opposite sides of parallelogram]

and AQ = PC as AB CD

AB= CD

=

β‡’

12

12AQCP is a parallelogram.

13.

Construction: Join BDNow, AB = BC = CD = AD [As all sides of square are equal]Also, AE = BF = CG = DH (given)E, F, G, H are midpoints of AB, BC, CD, AD respectively.In βˆ†ABD, E and H are midpoints.

So, by midpoint theorem, HE || DB and HE = 12DB ...(i)

Also, In βˆ†BCD, F and G are midpoints of sides BC and CD.So, by midpoint theorem, FG || BD

and FG = 12BD ...(ii)

By (i), (ii), we get HE = GF and HE || GFβ‡’ EFGH is a parallelogramβ‡’ EH = FG and EF = HG ...(iii) [since opposite sides of parallelogram are equal]Consider βˆ†AEH and βˆ†BEF AE = BE [since E is midpoint of AB] ∠A = ∠B = 90Β°

AH = BF

Since AD BC

AD BC

AH BF

=

β‡’ =

β‡’ =

12

12

β‡’ βˆ†AEH β‰… βˆ†BEF (SAS)β‡’ HE = EF ...(iv)By (iii), (iv), EF = FG = GH = EHIn βˆ†AEH, ∠1 + ∠3 = 90Β°In βˆ†BFE, ∠4 + ∠5 = 90 ∠1 + ∠3 + ∠4 + ∠5 = 180 2∠3 + 2∠4 = 180 ∠3 + ∠4 = 90Now, ∠3 +∠HEF + ∠4 = 180 90 + ∠HEF = 180 ∠HEF = 90As AH = AEβ‡’ ∠1 = ∠3Also, as BF = BEβ‡’ ∠5 = ∠4So, EFGH is a parallelogram in which all sides are equal and one angle is 90Β°.β‡’ EFGH is a square.

62 62

MATHEMATICS - 9DDITION LA APR CTICEA

Worksheet-1

Section A 1. Area = 10 cm2

D1 = 4 cm

Area of rhombus = 12

Γ— D1 Γ— D2

10 = 12

Γ— 4 Γ— D2

D2 = 105

D2 = 5 cm.

2. Area of trapezium = 12

Γ— (Sum of its parallel sides)

Γ— (distance between the parallel sides)

β‡’ 12

Γ— (AB + DC) Γ— DE.

3. Area of βˆ†ABC = 12

ar(parallelogram ABCD)

Area of βˆ†ABC = 12

Γ— 24

Area of βˆ†ABC = 12 cm2. 4. Area of triangle is equal to half the area of parallelogram.

arβˆ†ABD = 12

ar(ABCD)

ar ABDar ABCD

βˆ†( )

= 12

.

5. ar(βˆ†APB) = 20 cm2

Area of parallelogram ABCD = ?

Area of Parallelograms and Triangles09

Chapter

We know that

Area of βˆ†APB = 14

ar(parallelogram ABCD)

20 = 14

ar(parallelogram ABCD)

20 Γ— 4 = ar(parallelogram ABCD) 80 = ar (parallelogram ABCD)β‡’ ar(parallelogram ABCD) = 40 cm2.

Section B 6.

area of parallelogram PQRS (with SP are base and QN as altitude) = SP Γ— QN = SP Γ— 8area of parallelogram PQRS (with SM as altitude and PQ as base) = SM Γ— PQ = 6 Γ— 10So, SP Γ— 8 = 6 Γ— 10

SP = 6 108

608

152

Γ— = =

= 7.5 cm 7. Let ABC be any triangle and AD be one of its medians.

To prove: ar(βˆ†ABD) = ar(βˆ†ACD)From A, draw AN βŠ₯ BC

ar(βˆ†ABD) = 12

Γ— b Γ— h

= 12

Γ— BD Γ— AN

= 12

Γ— DC Γ— AN

[ D is the mid-point of BC, BD = DC] = ar(βˆ†ACD).Hence, median of a triangle divides it into two triangles of equal areas.

63 63

09. Area of P

arallelogram

s and

Triang

les

MATHEMATICS - 9DDITION LA APR CTICEA

8. ar (βˆ†ABC) = 10 cm2

ar(βˆ†ABC) = 12

Γ— b Γ— h

ar(βˆ†EBD) = 12

Γ— ED Γ— BD

ar(βˆ†ABC) = 12

BC Γ— AD

10 = 12

Γ— BC Γ— AD

20 = BC Γ— AD

ar(βˆ†EBD) = 12

12

12

BC ADΓ—

= 12

12

12

Γ— Γ— (BC Γ— AD)

= 18

Γ— 20 = 2.5 cm2

9. Perimeter of rhombus = 4 Γ— side AC = 48 cm and BD = 20 cmIn βˆ†AOB,

OA = 24 cm and OB = 10 cmAccording to Pythagoras Theorem, AB2 = OA2 + OB2

= (24)2 + (10)2

AB2 = 576 + 100 = 676

AB = 676 AB = 26 cm Perimeter = 4 Γ— 26 = 104 cm.

Section C 10. AD is the median of βˆ†ABC.

\ It will divide βˆ†ABC into two triangles of equal area.

\ ar(βˆ†ABD) = ar(βˆ†ACD)

\ ar(βˆ†ABD) = 12

ar(βˆ†ABC) ...(1)

In βˆ†ABD, E is the mid-point of AD.\ BE is the median.

\ ar(βˆ†BED) = ar(βˆ†ABE)

ar(βˆ†BED) = 12

12

Γ— ar(βˆ†ABC) [From eq. (1)]

ar(βˆ†BED) = 14

ar(βˆ†ABC)Hence proved.

11. Given: AP || BQ || CR. To prove: ar(AQC) = ar(βˆ†PBR) Proof: As triangles ABQ and PBQ have the same base BQ and

are between the same parallel AP || BQ.

\ ar(βˆ†ABQ) = ar(PBQ) ...(1)Also, as triangles BQC and BQR have the same base BQ and are between the same parallels BQ and CR.\ ar(βˆ†BQC) = ar(βˆ†BQR) ...(2)

Adding (1) and (2), we get; ar(βˆ†ABQ) + ar(βˆ†BQC) = ar(βˆ†PBQ) + ar(βˆ†BQR)β‡’ ar(βˆ†AQC) = ar(βˆ†PBR)Hence proved.

12. E and F are the mid-points of AB and BC.

As βˆ†ADF and parallelogram ABCD are on the same base and between the same parallels,

ar(βˆ†ADF) = 12

ar(parallelogram ABCD) ...(1)

Also, (βˆ†DCE) and parallelogram ABCD are on the same base and between the same parallels.

\ ar(βˆ†DCE) = 12

ar(parallelogram ABCD) ...(2)

From Eq. (1) and (2) ar(βˆ†ADF) = ar(βˆ†DCE)Hence proved.

13. Join BD and AC

To prove: ar βˆ†AXY = 38

ar(parallelogram ABCD)

In βˆ†ACD, AY is the median.

β‡’ ar βˆ†ACY = 12

ar βˆ†ACD ...(1)

64

MATHEMATICS - 9DDITION LA APR CTICEA

In βˆ†ABC, AX is the median

β‡’ ar βˆ†ACX = 12

ar βˆ†ABC ...(2)

β‡’ ar βˆ†ACY + ar βˆ†ACX [Add (1) and (2)]

= 12

ar βˆ†ACD + 12

ar βˆ†ABC

β‡’ ar AXCY = 12

ar ABCD ...(3)

In βˆ†BCD, since X and Y are midpoints of BC and CD respectively

ar βˆ†CXY = 14

ar βˆ†BCD ...(4)

Now, ar βˆ†AXY = ar AXCY – ar βˆ†CYX

= 12 ar ABCD –

14βˆ†BCD

= 12

ar ABCD – 18

ar ABCD

= 12

18

βˆ’

ar ABCD

ar BCD ar ABCD=

12

= 38

ar ABCD

14. To prove: ar(βˆ†ADX) = ar(βˆ†ACY)

A X

CD

B

Y

Proof: Join CX, DX and AYTriangles ADX and ACX are on the same base AX and between the same parallel AX and DC.\ ar(βˆ†ADX) = ar(βˆ†ACX) ...(1)Also, βˆ†ACX and βˆ†ACY are on the same base AC and between the same parallels AC and XY.\ ar(βˆ†ACX) = ar(βˆ†ACY) ...(2)From Eq. (1) and (2), we get ar(βˆ†ADX) = ar(βˆ†ACY).Hence proved.

15. In βˆ†ABD, S and P are midpoints

β‡’ ar βˆ†ASP = 14

ar βˆ†ABD

= 14

ar βˆ†AOB + 14

ar βˆ†AOD

In βˆ†ABC, P and Q are midpoints

β‡’ ar βˆ†PBQ = 14

ar βˆ†ABC

= 14

ar βˆ†AOB +14

ar βˆ†BOC

Similarly in βˆ†BCD and βˆ†ACD

ar βˆ†RCQ = 14

ar βˆ†BOC + 14

ar βˆ†COD

ar βˆ†DSR = 14

ar βˆ†COD + 14

ar βˆ†AOD

Now, ar PQRS

= ar ABCD – 12

ar βˆ†ASP – 12

a βˆ†PBQ – 12

ar βˆ†RCQ

– 12

ar βˆ†DSR

= ar βˆ†AOB + ar βˆ†BOC + ar βˆ†COD + ar βˆ†AOD

– 12

ar βˆ†ASP – 12

ar βˆ†PBQ – 12

ar βˆ†RCQ

– 12

ar βˆ†DSR

ar PQRS = 12

ar βˆ†AOB +12

ar βˆ†BOC + 12

ar βˆ†COD

+ 12

or βˆ†DOA

= 12

[ar βˆ†AOB+ ar βˆ†BOC + ar βˆ†COD + ar βˆ†DOA]

= 12

ar ABCD

Hence proved.

Section D 16. To prove: ABCD is a parallelogram.

Let’s draw DP βŠ₯ AC and BQ βŠ₯ AC.

Proof: In βˆ†DOP and βˆ†BOQ, ∠DPO = ∠BQO (90Β° each) ∠DOP = ∠BOQ (V.O.A) OD = OB (given)\ βˆ†DOP β‰… βˆ†BOQ (by AAS)\ DP = BQ (by CPCT) ...(1)and ar(βˆ†DOP) = ar (BOQ) ...(2) [Area of congruent triangles are equal]In βˆ†CDP and βˆ†ABQ, ∠CPD = ∠AQB (90Β° each) CD = AB (given) DP = BQ [From Eq. (1)]\ βˆ†CPD β‰… βˆ†ABQ (by RHS)β‡’ ar(βˆ†CPD) = ar(ABQ) ...(3)Adding Eq. (2) and (3), we get; ar(DOC) = ar(AOB) ...(4)Adding ar(OCB) on both sides of Eq. (4) ar(DOC) + ar(OCB) = ar(AOB) + ar(OCB)β‡’ ar(DCB) = ar(ACB)We know that two triangles having same base and equal areas, lie between same parallels.Here βˆ†DBC and βˆ†ABC are on the same base BC and are equal in area.So DA ||CBNow, In βˆ†DOA and βˆ†BOC, ∠DOA = ∠BOC (V.O.A.) ∠DAO = ∠BCO (Alternate angles) OD = OB (Given)

65 65

09. Area of P

arallelogram

s and

Triang

les

MATHEMATICS - 9DDITION LA APR CTICEA

\ βˆ†DOA β‰… βˆ†BOC (by AAS)β‡’ DA = BC (by CPCT)So, In ABCD, since DA || CB and DA = CBOne pair of opposite sides of quadrilateral ABCD is equal and parallel.\ ABCD is a parallelogram.Hence proved.

17. Since D and E are the mid-points of sides BC and AC.

\ DE || BA β‡’ DE ||BF Similarly, FE ||BD. So, BDEF is a parallelogram.

Similarly, DCEF and AFDE are parallelograms.Now, DF is a diagonal of parallelogram BDEF\ ar(βˆ†BDF) = ar(βˆ†DEF) ...(1)DE is diagonal of parallelogram DCEF\ ar(βˆ†DCE) = ar(βˆ†DEF) ...(2)FE is a diagonal of parallelogram AFDE\ ar(βˆ†AFE) = ar(DEF) ...(3) From Eq. (1), (2) and (3), we get; ar(BDF) + ar(DCE) + ar(AFE) + 3 ar(βˆ†DEF)But ar(BDF) + ar(DCE) + ar(AFE) + ar(DEF) = ar(ABC)\ 4 ar(βˆ†DEF) = ar(βˆ†ABC)

β‡’ ar(βˆ†DEF) = 14

ar(βˆ†ABC)

Now, ar(parallelogram BDEF) = 2 ar(βˆ†DEF)

β‡’ ar(parallelogram BDEF) = 2 Γ— 14

ar(βˆ†ABC)

β‡’ ar(parallelogram BDEF) = 12

ar(βˆ†ABC)

Hence proved. 18. Since AD and AE are medians of βˆ†ABC and βˆ†ABD respectively.

\ ar(βˆ†ABD) = 12

ar(βˆ†ABC) ....(1)

and ar(βˆ†ABE) = 12

ar(βˆ†ABD) ...(2)

OB is a median of βˆ†ABE,

\ ar(βˆ†BOE) = 12

ar(ABE) ...(3) From Eq. (1), (2) and (3), we get;

ar(βˆ†BOE) = 12

ar(ABE)

= 12

12ar ABD

= 14

ar βˆ†ABD

= 14

12

Γ— ar βˆ†ABC = 18

ar βˆ†ABC

Hence proved.

Worksheet-II

Section A 1. ar(parallelogram ABCD) = 56 cm2

ar(βˆ†ABC) = 12

ar(parallelogram ABCD)

= 12

Γ— 56 = 28 cm2

2. Area of rhombus = 12

Γ— D1 Γ— D2

= 12

Γ— 48 Γ— 30

= 24 Γ— 30 = 720 cm2

3.

In ||ABCD, AD ||BCβ‡’ ∠2 = ∠4 [Corresponding angles]In parallelogram ABEF, AF ||BEβ‡’ ∠1 = ∠3Also AF = BESo, βˆ†ADF β‰… βˆ†BEC

β‡’ ar βˆ†ADF = ar βˆ†BEC [AAS]Add ar (ABED) on both sides [Congruent triangle have equal area]β‡’ ar βˆ†ADF + βˆ†ABED = ar βˆ†BEC + ar ABEDβ‡’ ar ABEF = ar ABCDHence proved. 1

4. ar(βˆ†ABD) = 12 cm2

Since AD is a median. A median divides a triangle into two triangles of equal areas.

β‡’ ar(βˆ†ABD) = ar(βˆ†ACD)β‡’ ar(βˆ†ACD) = 12 cm2

But ar(βˆ†ABC) = ar(βˆ†ABD) + ar(βˆ†ACD) = (12 + 12) cm2 = 24 cm2

Section B 5. Area of parallelogram = base Γ— height

A = b Γ— h h = 2b (According to question) 288 = b Γ— 2b 288 = 2b2

b2 = 2882

b = 144 b = 12 cm h = 2b h = 2 Γ— 12 = 24 cm

66

MATHEMATICS - 9DDITION LA APR CTICEA

6. D1 = 24 cm and side of rhombus = 20 cm BD = 24 cm, AB = 20 cm

Using Pythagoras Theorem, In βˆ†AOB, AB2 = OA2 + OB2

(20)2 = OA2 + (12)2

400 = OA2 + 144 400 – 144 = OA2

OA2 = 256

OA = 256 OA = 16 AC = 32 cm

Area of rhombus = 12

Γ— D1 Γ— D2

= 12

Γ— 32 Γ— 24

= 32 Γ— 12 = 384 cm2

7. ar(βˆ†ABC) = 48 cm2

ar(βˆ†BDE) = ?

Since AD is the median of βˆ†ABC.β‡’ ar(βˆ†ABD) = ar(βˆ†ADC)But ar(βˆ†ABC) = ar(βˆ†ABD) + ar(βˆ†ADC)β‡’ 48 = 2 ar(βˆ†ABD)β‡’ 24 = ar(βˆ†ABD)Now, BE is the median of βˆ†ABD.β‡’ ar(βˆ†BDE) = arβˆ†ABE)But ar(βˆ†ABD) = ar(βˆ†BDE) + ar(βˆ†ABE)β‡’ 24 = 2 ar(βˆ†BDE)β‡’ ar(βˆ†BDE) = 12 cm2

8. Area of trapezium ABCD = ?AB = 5 cm, DE = 5 cm, BE = 12 cm and BC = 13 cm

In βˆ†BEC, BC2 = BE2 + EC2

(13)2 = (12)2 + EC2

169 – 144 = EC2

EC = 25

EC = 5 cm

Area of trapezium ABCD = 12

Γ— (AB + CD) Γ— BE

= 12

Γ— (5 + 10) Γ— 12

= 15 Γ— 6 = 90 cm2

Section C 9. Given: ar(βˆ†DQC) = ar(βˆ†DPC)

ar(βˆ†BDP) = ar(βˆ†AQC)To prove: ABCD and DCPQ are trapeziums.

Proof: ar(βˆ†AQC) = ar(βˆ†BDP) ...(1) ar(βˆ†DQC) = ar(βˆ†DPC) ...(2)Subtracting Eq. (2) from Eq. (1) ar(βˆ†AQC) – ar(βˆ†DQC) = ar(βˆ†BDP) – ar(βˆ†DPC)β‡’ ar(βˆ†ADC) = ar(βˆ†BDC)Now, triangles ADC and BDC have equal areas and are on the same base DC, therefore, AB || DCβ‡’ ABCD is a trapezium.Also, triangle DQC and DPC have equal area and are on the same base DC therefore, DC || QPβ‡’ DCPQ is a trapezium.Hence proved.

10. Given: AC and BD are diagonals of trapezium ABCD and AB || CD.To prove: ar(βˆ†AOD) = ar(βˆ†BOC)

Proof: As the triangle ABC and ABD are on the same base AB and between the same parallels AB || DC,\ ar(βˆ†ABC) = ar(βˆ†ABD)β‡’ ar(βˆ†ABC) – ar(βˆ†AOB) = ar(βˆ†ABD) – ar(βˆ†AOB) [Subtracting equal area from both sides]β‡’ ar(βˆ†BOC) = ar(βˆ†AOD)Hence proved.

11. As βˆ†ABE and parallelogram EBCY are on the same base BE and between the same parallels BE || CA.

\ ar(βˆ†ABE) = 12

ar(parallelogram EBCY) ...(1)

As βˆ†ACF and parallelogram XBCF have same base CF and are between the same parallels FC || AB

\ ar(βˆ†ACF) = 12

ar(parallelogram XBCF) ...(2)

But parallelogram EBCY and parallelogram XBCF have same base BC and between the same parallel EF || BC,

67 67

09. Area of P

arallelogram

s and

Triang

les

MATHEMATICS - 9DDITION LA APR CTICEA

B C

E F

A

X Y

\ ar(parallelogram EBCY) = ar(parallelogram XBCF)

β‡’ 12

ar(parallelogram EBCY) = 12

ar(parallelogram XBCF)

β‡’ ar(βˆ†ABE) = ar(βˆ†ACF) [From Eq. (1) and (2)]Hence proved.

12. Given: S divides QR of βˆ†PQR in the ratio l : mTo prove: ar(βˆ†PQS) : ar(βˆ†PRS) = l : mProof: Draw PN βŠ₯QR.

ar(βˆ†PQS) = 12

Γ— b Γ— h

= 12

Γ— QS Γ— PN

ar(βˆ†PRS) = 12

Γ— SR Γ— PN

ar PQSar PRS( )( )βˆ†βˆ†

=

1212

Γ— Γ—

Γ— Γ—

QS PN

SR PN

ar PQSar PRS( )( )βˆ†βˆ†

= QSSR

= lm

(given)

β‡’ ar(βˆ†PQS) : ar(βˆ†PRS) = l : mHence proved.

Section D 13. Draw BF || AE

To prove: ar βˆ†ABE = 12

ar ABCD

Proof: In ABFE, AB || EF [Given] AE || BF [By construction]β‡’ ABFE is a parallelogram.In parallelogram ABFE, BE is a diagonal and diagonal divides parallelogram in two triangle of equal area

β‡’ ar βˆ†ABE = 12

ar ABFE ...(1)

Now parallelogram ABFE and parallelogram ABCD lie on same base AB and between same parallel AB || CE

β‡’ ar ABFE = ar ABCD ...(2)

By (1), (2), ar βˆ†ABE = 12

ar ABCD.

Hence proved. 14. Construction: Through P, draw a line parallel to AB to meet AD

at E and BC at F.

Then AEFB is a parallelogram.(a) As βˆ†APB and parallelogram are on the same base AB and

between the same parallels AB and EF.

\ ar(βˆ†APB) = 12

ar(parallelogram AEFB) ...(1)

Since EF || AB and AB || DC, so EF || DCβ‡’ EDCF is a parallelogram.As βˆ†PDC and parallelogram EDCF are on the same base DC and between the same parallels DC and EF,

\ ar (βˆ†PDC) = 12

ar(parallelogram EDCF) ...(2)

From (1) and (2), we get

ar(βˆ†APB) + ar(βˆ†PDC) = 12

[ar(parallelogram AEFB)

+ ar(parallelogram EDCF)]

= 12

ar(parallelogram ABCD).

(b) Similarly, by drawing a line through P and parallel to AD.

ar(βˆ†APD) + ar(βˆ†PBC) = 12

ar(parallelogram ABCD)

= ar(βˆ†APB) + ar(βˆ†PDC)Hence proved.

15. Given: ABCD is a parallelogram.

To prove: ar βˆ†AOB = ar βˆ†BOC = ar βˆ†COD = ar βˆ†AODProof: AS we know diagonals of parallelogram bisect each otherβ‡’ AC and BD bisects each other at O.β‡’ O is Midpoint of AC and BD.In βˆ†ABD, AO is the Medianβ‡’ ar βˆ†AOB = ar βˆ†AOD ...(1)[Since in a triangle median divides it into two triangle of equal area]In βˆ†ABC, BO is the Medianβ‡’ ar βˆ†AOB = ar βˆ†BOC ...(2)In βˆ†BCD, CO is the Medianβ‡’ ar βˆ†BOC = ar βˆ†COD ...(3)In βˆ†ACD, DO is the Medianβ‡’ ar βˆ†COD = ar βˆ†AOD ...(4)From (1), (2), (3), (4)ar βˆ†AOB = ar βˆ†COD = ar βˆ†BOC = ar βˆ†AOD.Hence proved.

68 68

MATHEMATICS - 9DDITION LA APR CTICEA

Worksheet-1

Section A 1. PQRS is a cyclic quadrilateral.

∠QRS = 110° ∠SPQ = ?We know that sum of opposite angles of a cyclic quadrilateral is 180°.

β‡’ ∠QRS + ∠SPQ = 180Β°β‡’ 110Β° + ∠SPQ = 180Β°β‡’ ∠SPQ = 180Β° – 110Β°β‡’ ∠SPQ = 70Β°

2. ∠BAC = 65°, ∠BDC = ?

∠BAC = 65Β°β‡’ ∠BDC = 65Β° (Angles in the same segment of a

circle are equal) 3. O is the centre of the circle.

∠ACB = 30°, ∠ABC = ? We know that angle in a semicircle is a right angle

β‡’ ∠BAC = 90Β°Now, in DABC, ∠ABC + ∠BAC + ∠ACB = 180Β° (A.S.P.) ∠ABC + 90Β° + 30Β° = 180Β° ∠ABC = 180Β° – 120Β° ∠ABC = 60Β°

Circles10Chapter

4. ∠ABC = 69° and ∠ACB = 31°

∠BDC = ?In DABC, ∠ABC + ∠BAC + ∠ACB = 180Β° (A.S.P) 69Β° + ∠BAC +31Β° = 180Β° ∠BAC = 180Β° –100Β° ∠BAC = 80Β°Since ∠BAC = 80Β° β‡’ ∠BDC = 80Β° (Angles in a same segment of a circle are equal)

5. ∠ACB = 65°

OA = OB = Radii ∠OAB = ∠OBA = x (Angles opp. to equal sides are equal)β‡’ ∠AOB = 2∠ACB (Angle subtended by an arc at centre is

twice the angle subtended by it on remaining part of circle)β‡’ ∠AOB = 2 Γ— 65Β°β‡’ ∠AOB = 130Β° β‡’ y = 130Β°Now in DOAB, x + y + ∠OBA = 180Β° (A.S.P.) x + 130Β° + x = 180Β° 2x = 180Β° – 130Β° 2x = 50Β°

x = 25Β° and y = 130Β°

6. In DOPA and DOPB

OA = OB OP = OP (common) ∠OPA = ∠OPB = 90°

69 69

10. Circles

MATHEMATICS - 9DDITION LA APR CTICEA

β‡’ DOPA β‰… DOPB (RHS)β‡’ AP = PB (CPCT)In DOPA, OA2 = OP2 + AP2

132 = 122 + AP2

169 – 144 = AP2

25 = AP2

5 = AP⇒ AB = 2AP = 10 cm

7. Let ABCD be a cyclic quadrilateral and let O be the centre of the circle. Then each of AB, BC, CD and DA being a chord of the circle, its right bisector must pass through O.

Hence, the right bisectors of AB, BC, CD and DA pass through O.

So we can say that perpendicular bisectors of the sides of a cyclic quadrilateral are concurrent.

Section B 8. Given: AB is a diameter of circle

To Prove: AB is greatest chord of a circle. Proof: AB is nearest to centre than CD \ AB > CD (Q Any two chords of a circle, the one which is

nearer to centre is larger) Hence, AB is larger than every other chord. 9. ∠ABC = 70°, ∠BCA = 60°

In DABC, ∠ABC + ∠BCA + ∠CAB = 180Β° (A.S.P.) 70Β°+ 60Β°+ ∠CAB = 180Β° ∠CAB = 180Β° – 130Β° ∠CAB = 50Β°Also, ∠BDC = 50Β° (Angle in the same

segment of a circle are equal)

\ ∠CDB = 50°

10. ∠ACB = 35Β°, ∠OAB = ? OA = OB = Radiiβ‡’ ∠OAB = ∠OBA (Angle opposite to equal sides

of a triangle are equal)

Let ∠OAB = ∠OBA = x ∠AOB = 2 ∠ACB (Angle subtended by an arc) ∠AOB = 2 Γ— 35Β° ∠AOB = 70Β°In DAOB, x + ∠AOB + x = 180Β° (A.S.P) 2x + 70Β° = 180Β° 2x = 180Β° – 70Β° = 110Β° x = 55Β°

β‡’ ∠OAB = 55Β°

11. OA = 3 cm, OP = 5 cm

Find PQ. We know that the perpendicular from centre to a chord bisect

the chord.

\ PA = 12

PQ

β‡’ 2PA = PQ\ PQ = 2PAIn DOAP, OP2 = OA2 + PA2

(5)2 = (3)2 + PA2

25 = 9 + (PA)2

PA2 = 25 – 9 = 16 = (4)2

PA = 4 cm\ PQ = 2PA = 2 Γ— 4 = 8 cm

Section C 12. Given: ABCD is a cyclic quadrilateral

To prove: ∠A + ∠C = 180Β° and ∠B + ∠D = 180Β°Construction: Join AC and BD.Proof: Let AB be the chord of circle.\ ∠ACB = ∠ADB ...(1) [Angle in the same segment are equal] ∠BAC = ∠BDC ...(2)Adding eq. (1) and (2), we get;β‡’ ∠ACB + ∠BAC = ∠ADB + ∠BDC

70

MATHEMATICS - 9DDITION LA APR CTICEA

β‡’ ∠ACB + ∠BAC = ∠ADCβ‡’ ∠ABC + ∠ACB + ∠BAC = ∠ABC + ∠ADC (Adding ∠ABC on both sides)β‡’ 180Β° = ∠ABC + ∠ADC ∠B + ∠D = 180Β°But ∠A + ∠B + ∠C + ∠D = 360Β°\ ∠A + ∠B + ∠C + ∠D = 360Β° ∠A + ∠C + 180Β° = 360Β° ∠A + ∠C = 360Β° – 180Β° ∠A + ∠C = 180Β° and ∠B + ∠D = 180Β°Hence proved.

13. Let O and Oβ€² be centres of circles with radii 5 cm, 3 cm, and AB be the common chord.

\ OOβ€² βŠ₯ AB and M is the mid-point of AB.Let AM = y and OM = x cmAs OOβ€² = 4 cm (given)\ MOβ€² = (4 – x) cm In DOMA, by Pythagoras Theorem, OA2 = OM2 + AM2

β‡’ (5)2 = x2 + y2

β‡’ x2 + y2 = 25 ...(1)In DOβ€²MA, by Pythagoras Theorem, Oβ€²A2 = OMβ€²2 + AM2

β‡’ (3)2 = (4 – x)2 + y2 ...(2)Subtracting eq. (2) from (1), we get 25 – 9 = x2 – (4 – x)2

16 = 8x – 16 32 = 8x x = 4Putting this value of x in eq. (1), we get (4)2 + y2 = 25 y2 = 25 – 16 y2 = 9 y = 3Length of common chord, AB = 2AM = 2 Γ— 3 = 6 cm

14. ∠DAC = ∠DBC

(Angle in the same segment of a circle are equal)β‡’ ∠DAC = 40Β°\ ∠DAB = 40Β° + 45Β° = 85Β°As ABCD is a cyclic quadrilateral. ∠BCD + ∠DAB = 180Β° ∠BCD + 85Β° = 180Β° ∠BCD = 95Β°

In DABC, AB = BC β‡’ ∠BCA = ∠BACbut ∠BAC = 45Β°β‡’ ∠BCA = 45Β°\ ∠ECD = ∠BCD – ∠BCA = 95Β° – 45Β° = 50Β°

β‡’ ∠ECD = 50Β°

and ∠BCD = 95°

Section D 15. Given ABCD is a cyclic Quadrilateral and its diagonals are

diameters of circle.

Now OA = OC [Radii of same circle]and OB = ODSo, the diagonals AC and BD of the Quadrilateral bisect each other.Therefore, ABCD is a Parallelogram.Also, AC is diameter,\ ABCD = 90Β° (Angle in a semicircle is a right angle) Thus, ABCD is a Parallelogram in which one angle is 90Β°\ ABCD is Rectangle.Hence proved.

16. Let A, B and C represent the three girls Reshma, Salma and Mandeep.Then AB = 6 m and BC = 6 m

If O is the centre of the circle,then OA = radius = 5 m

From B, draw BM βŠ₯ AC.Since ABC is an isosceles triangle with AB = BC, M is mid-point of AC.\ BM is βŠ₯lar bisector of AC, hence it passes through the centre of the circle.Let AM = y and OM = x,then BM = (5 – x)In DOAM, by Pythagoras Theorem, OA2 = OM2 + AM2

β‡’ 52 = x2 + y2

β‡’ x2 + y2 = 25 ...(1)In DAMB, by Pythagoras Theorem, AB2 = BM2 + AM2

β‡’ 62 = (5 – x)2 + y2

β‡’ (5 – x)2 + y2 = 36 ...(2)

71 71

10. Circles

MATHEMATICS - 9DDITION LA APR CTICEA

Subtracting eq. (2) from eq. (1), we get 36 – 25 = (5 – x)2 – x2

β‡’ 11 = 25 – 10x

β‡’ 10x = 14 β‡’ x = 75

Substituting this value of x in eq. (1), we get

y2 + 4925

= 25 β‡’ 2 = 25 – 4925

625 4925

= βˆ’

β‡’ y2 = 57625

β‡’ y = 245

\ AC = 2AM = 2 Γ— y = 2 Γ— 245

485

= = 9.6 m

Hence, the distance between Reshma and Mandip is 9.6 m. 17. Given: Radius = 5 cm, AB = 6 cm and CD = 8 cm. OP βŠ₯ AB, OQ βŠ₯ CD and AB ||CD.

To find: PQ Construction: Join OA and OC. Sol.

AP = 12

AB β‡’ AP = 12

6Γ— β‡’ AP = 3 cm

CQ = 12

CD β‡’ CQ = 12

8Γ— β‡’ CQ = 4 cm

In DOPA, by Pythagoras Theorem, OA2 = OP2 + AP2

(5)2 = OP2 + (3)2

OP2 = 25 – 9 OP2 = 16 OP = 4In DOQC, by Pythagoras Theorem, OC2 = OQ2 + CQ2

(5)2 = OQ2 + (4)2

OQ2 = 25 – 16 OQ2 = 9 = (3)2

OQ = 3\ PQ = OP – OQβ‡’ PQ = 4 – 3

β‡’ PQ = 1 cm

Worksheet-II

Section A 1. AB = 10 cm

OA = OB = Radii of circle = 13 cm

AM = MB = 12 (AB)

= 12

(10)

= 5 cmIn DOAM, OA2 = AM2 + OM2

(13)2 = (5)2 + OM2

169 – 25 = OM2

OM2 = 144 OM = 12 cm\ Distance of chord from centre = 12 cm.

2. ∠AOC = 130°

Construction: Take a point P, anywhere on circle. Join AP and CP.

Sol. ABC subtends ∠AOC at centre and ∠APC at remaining part of circle

β‡’ ∠APC = 12∠AOC =

12

Γ— 130Β°

∠APC = 65Β° ∠ABC + ∠APC = 180Β° (opposite angles of cyclic Quadrilateral) 65Β° +∠ABC = 180 Β° ∠ABC = 180Β° – 65Β°

∠ABC = 115°

3. OP = radius = 13 cm

OR = 5 cm In DOPR,

OP2 = OR2 + PR2

(13)2 = (5)2 + PR2

169 – 25 = PR2

PR2 = 144 PR = 12 PQ = 2PR PQ = 2 Γ— 12 PQ = 24 cm.

4. ∠PQR = 120° OP = OR = Radii of the circle.

72

MATHEMATICS - 9DDITION LA APR CTICEA

Reflex ∠POR = 2 ∠PQR = 2 Γ— 120Β° = 240Β°\ ∠POR = 360Β° – 240Β° = 120Β°In DOPR, OP = OR (Radii of same circle)\ ∠OPR = ∠ORP ∠OPR + ∠ORP + ∠POR = 180Β° (A.S.P.) 2∠OPR + 120Β° = 180Β° 2∠OPR = 60Β°

∠OPR = 30°

Section B 5. ∠OAB = 45°

In DOAB, OA = OB (Radii of circle)β‡’ ∠OAB = ∠OBA = 45Β° (A.S.P.) ∠OAB + ∠OBA + ∠AOB = 180Β° 45Β° + 45Β° + ∠AOB = 180Β° 90Β° + ∠AOB = 180 ∠AOB = 90Β°

∠AOB = 2 ∠ACB 90° = 2 ∠ACB

∠ACB = 902

Β°

β‡’ ∠ACB = 45Β°

6. OA = 10 cm and AB = 12 cm

OD βŠ₯ AB AB = 12 cm AC = BCAnd AB = AC + BC 12 = 2 AC AC = 6 cmIn DAOC, by Pythagoras Theorem OA2 = OC2 + AC2

(10)2 = OC2 + (6)2

100 – 36 = OC2

OC2 = 64 OC = 8 cmβ‡’ OD = OA = 10 (radii of same circle)β‡’ CD = OD – OC = 10 – 8 = 2 cm

7. We have ∠ABC = 55°and ∠BCA = 65°

In DABC,∠ABC + ∠BCA + ∠BAC = 180° (A.S.P.)

55Β° + 65Β° + ∠BAC = 180Β° 120Β° + ∠BAC = 180Β° ∠BAC = 180Β° – 120Β° ∠BAC = 60Β°β‡’ ∠BDC = 60Β° (Angles in the same segment

of a circle are equal) 8. The circumcentre of a triangle ABC is O. Prove that ∠OBC +

∠BAC = 90°

Sol. O is the circumcentre of DABC. Join OB and OC.

In DOBC, OB = OC (Radii of same circle)\ ∠OBC = ∠OCB (Q angles opposite

to equal sides are equal) ∠BOC + ∠OBC + ∠OCB = 180Β°β‡’ ∠BOC + 2 ∠OBC = 180Β°

β‡’ 12∠BOC + ∠OBC = 90Β° ...(1)

Now, ∠BAC = 12∠BOC

β‡’ ∠BAC = 90Β° – ∠OBCβ‡’ ∠OBC + ∠BAC = 90Β°Hence proved.

Section C 9. Since OA, OB and OC are radii of the circle.

In DOAB, OA = OB ∠OAB = ∠OBA ∠OAB + ∠OBA + ∠AOB = 180Β° (A.S.P.) 2 ∠OAB + 120Β° = 180Β° 2 ∠OAB = 60Β° ∠OAB = 30Β°In DAOC, we get; OA = OCβ‡’ ∠OCA = ∠OAC

73 73

10. Circles

MATHEMATICS - 9DDITION LA APR CTICEA

∠OCA + ∠OAC + ∠AOC = 180° (A.S.P.) 2 ∠OAC + 80° = 180 ° 2∠OAC = 100° ∠OAC = 50° ∠BAC = ∠OAB + ∠OAC ∠BAC = 30° + 50° ∠BAC = 80°

10. In DAOB, OA = OBβ‡’ ∠ABO = 30Β° β‡’ In DBOC, OB = OC β‡’ ∠OBC = 40Β°\ ∠ABC = ∠ABO + ∠OBC ∠ABC = 30Β° + 40Β° = 70Β° ∠AOC = 2 Γ— ∠ABC ∠AOC = 2 Γ— 70Β°

\ ∠AOC = 140° 11. Given: A circle such that Chord PQ = Chord RS

To prove: ∠POQ = ∠ROS Proof: In DPOQ and DROS,

PQ = RS (Given) OP = OR = radius OQ = OS = radius\ DPOQ = DROS (by SSS)\ ∠POQ = ∠ROS (by CPCT)β‡’ Angles subtended by equal Chords are equal.

12. PQRS is a cyclic quadrilateral and sum of opposite angles of cyclic quadrilateral is 180Β°.

β‡’ ∠P + ∠R = 180Β°β‡’ 5y + y = 180Β°β‡’ 6y = 180Β°

β‡’ y = 1806

Β°

y = 30Β°

∠P = 5y = 5 Γ— 30Β° =150Β° ∠R = 30Β°β‡’ ∠Q + ∠S = 180Β°β‡’ 3x + x = 180Β° 4x = 180Β°

x = 1804

Β°

x = 45Β° ∠Q = 45Β° ∠S = 3 Γ— 45Β° ∠S = 135Β°

Section D 13. Given: An arc PQ of a circle C (O, r) and a point R on the

remaining part of the circle i.e., arc QP . To prove: ∠POQ = 2 ∠PRQ. Construction: Join RO and produce it to a point M outside the

circle.

Proof: When PQ is a minor arc In DPOR, ∠POM is the exterior angle.

\ ∠POM = ∠OPR + ∠ORP [Q Exterior angle property]

β‡’ ∠POM = ∠ORP + ∠ORP ...(1) (Q OP = OR = r \ ∠OPR = ∠ORP)β‡’ ∠POM = 2∠ORPIn DQOR,\ ∠QOM = ∠OQR + ∠ORQβ‡’ ∠QOM = ∠ORQ + ∠ORQβ‡’ ∠QOM = 2 ∠ORQ ...(2)Adding (1) and (2), we get

∠POM + ∠QOM = 2 ∠ORP + 2 ∠ORQ\ ∠POM + ∠QOM = 2 (∠ORP + ∠ORQ)β‡’ ∠POQ = 2∠PRQHence proved.

14. Given: AB = 8 cm and CD = 6 cm Radius = 5 cm To find: PQ Construction: Join OA and OC

Sol. AB = 8 cm AP = PB = 4 cm CD = 6 cm [Q from the centre of circle on any chord

of circle bisect it] CQ = QD = 3 cm OA = OC = radii = 5 cmIn right DOAP,By Pythagoras Theorem, OP2 = OA2 – AP2

= (5)2 – (4)2

= 25 – 16 = 9 OP = 3In DOCQ, By Pythagoras Theorem, OQ2 = OC2 – CQ2

= (5)2 – (3)2

74

MATHEMATICS - 9DDITION LA APR CTICEA

= 25 – 9 = 16 OQ = 4β‡’ PQ = PO + OQ = 3 + 4 = 7 cmβ‡’ PQ = 7 cm

15. Given: Two circles C (o, r) and c(oβ€², r) intersect at the points A and B.

To prove: OOβ€² is perpendicular to AB Construction: Join OA and OB, Oβ€²A and Oβ€²B

Proof: In DOAOβ€²and OBOβ€² OA = OB [Radii of circles] Oβ€²A = Oβ€²B

OOβ€² = OOβ€² (Common)\ DOAOβ€² β‰… DOBOβ€² (by SSS)β‡’ ∠AOOβ€² = ∠BOOβ€² (by CPCT)In DAOM and DBOM, OA = OB (radii) ∠AOOβ€² = ∠BOOβ€² (Proved above) OM = OM (Common)\ DAOM β‰… DBOM (by SAS) AM = BM (by CPCT) ∠OMA = ∠OMB (by CPCT)\ ∠OMA + ∠OMB = 180Β° (L.P) ∠OMA + ∠OMA = 180Β°β‡’ 2∠OMA = 180Β°β‡’ ∠OMA = 90Β°Hence, OOβ€² is the Perpendicular Bisector.

75 75

MATHEMATICS - 9DDITION LA APR CTICEA

Worksheet-1

Section A 1. Yes, we can construct an angle of 67.5Β°. We draw an angle of

60Β° then add 12 of 15Β° i.e., 7.5Β°. So with the help of compass. We

construct an angle of 67.5Β°. 2. Yes, we can construct an angle of 52.5Β°. First we can draw an

angle of 45Β° with the help of compass. Similarly, add 7.5Β° i.e., 12

of 15Β° with the help of compass. So we can construct an angle of 52.5Β°.

3. In DABC, BC = 5 cm, ∠B = 45Β°. AB – AC = 5.2 cm. Difference between two sides is less than 5.0 cm but not more than 5.0 cm. So the construction of DABC is impossible.

4. An angle of measurement 41Β° can not constructed by using a compass. It is false statement. 41Β° angle is constructed by protractor only.

Section B 5.

<CAB = 75Β°

6. Draw an angle of angle 105Β°.

7.

DFF

CB

A

GeometricalConstructions11

Chapter

This figure is not constructed.The given statement is false. Because ∠B = 105° which is obtuse angle and perimeter of DABC = 10 cm. One angle must be acute angle but not obtuse angle.

Section C 8.

9.

G

EF

A BD

C

10.

1. Draw a line segment AB = 5 cm.2. Take centre A with length 5 cm.3. Similarly with centre B.4. Both arc intersect at C.5. Therefore DABC an equilateral triangle with side 5.0 cm.

76

MATHEMATICS - 9DDITION LA APR CTICEA

Section D 11.

Steps of construction:1. Draw a line segment AB = 11 cm.2. Draw ∠A = 90° and ∠B = 30° with compass and scale.3. Bisect ∠DAB and ∠ABC.4. Draw perpendicular bisector of AZ and BZ.5. Therefore join XZ and YZ.6. Then DXYZ is required triangle.

12.

Worksheet-I1

Section A 1. Because it can’t be bisected. 2. No because it can’t be bisected. 3. Because it can’t be bisected. 4. When 2 lines meet and make ∠ of 90Β°.

Section B 5. Because it can be bisected. 6. No, because ∠40Β° can’t be made by construction. 7. Yes, because we can construct an angle with the one angle

known and difference of 2 sides. 8. Yes, reason mentioned above.

Section C 9. (a)

(b)

10.

Square of side 6.2 cm.ABCD is a square.Side AB = BC = CD = DA = 6.2 cm.

11.

A triangle with 4.3 cm altitude. 12.

77 77

011. Geom

etrical Con

struction

s

MATHEMATICS - 9DDITION LA APR CTICEA

Section D 13.

A triangle with base 5.2 cm ∠C = 60° ∠B = 60°DABC is required triangle.

14.

YX

D E

G FA

B 5 cm C

45Β°60Β°

15.

78 78

MATHEMATICS - 9DDITION LA APR CTICEA

Worksheet-1

Section A 1. Area of an equilateral triangle = 16 3 m2.

34

2a = 16 3

a2 = 16 3 4

3Γ—

a2 = 64 a = 8 m Perimeter = 3a = 3 Γ— 8 = 24 m

2. Side of equilateral triangle = 2a

Area = 34

2a

= 34

2 2Γ— ( )a

= 34

4 2Γ— a = 3 2a

3. Perimeter of triangle = 32 cm

\ Semi-perimeter ( ) = 322

s = a b c+ +

2

322

= 8 11

2+ + c

; 32 = 19 + c

c = 32 – 19 c = 13

4. Area of isosceles triangle = 32 cm2

12

2a = 32

a2 = 32 Γ— 2 a2 = 64 a = 8

Hypotenuse = 8 2 , which is given.So, True

5. Area of an equilateral triangle = 4 3 cm2

34

2a = 4 3

a2 = 4 4 3

3Γ—

a2 = 16 a = 4 cm

Heron’s Formula12Chapter

Perimeter = 3a = 3 Γ— 4 = 12 cm

Half of perimeter = 122 = 6 cm.

Section B 6. Base = 80 cm and area = 360 cm2

Area of triangle = 12Γ— Γ—base height

360 = 12

80Γ— Γ— h

h = 36040

β‡’ h = 9 cm

In βˆ†ABD, By Pythagoras theorem,

AB2 = AD2 + BD2

AB2 = (40)2 + (9)2

= 1600 + 81 = 1681 AB = 41 cm Perimeter = 2a + b = 2 Γ— 41 + 80 = 82 + 80 = 162 cm.

7. Height of equilateral triangle = 9 cmArea of an equilateral triangle:

= h2

3= ( )9

3

2

= 813

= 46.767 cm2.

8. Perimeter of plot = 210 mLet breadth of rectangular plot = x\  length of rectangular plot = 3x Perimeter = 2(L + B)

2102

= x + 3x

105 = 4x

x = 1054

x = 26.25 Length = 3 Γ— 26.25 = 78.75 Breadth = 26.25 Area = L Γ— B = 78.75 Γ— 26.25 = 2067.19 m2.

9. Area of parallelogram = 392 m2

Let base = x and height = 3x Area = b Γ— h

79 79

12. Heron’s Form

ula

MATHEMATICS - 9DDITION LA APR CTICEA

392 = x Γ— 3x 392 = 3x2

x2 = 3923

= 130.67

x2 = 131

x = 131 x = 11.4 Base = 11.4 m\ Altitude = 3 Γ— 11.4 = 34.2 m.

10. Let AB = 18 cm BC = 24 cm AC = 30 cm

s = a b c+ +

2

s = 18 24 302

+ + = 722

= 36 cm

Area of βˆ†ABC = s s a s b s c( )( )( )βˆ’ βˆ’ βˆ’

= 36 36 18 36 24 36 30( )( )( )βˆ’ βˆ’ βˆ’

= 36 18 12 6Γ— Γ— Γ—

= 9 4 6 3 6 2 2 3Γ— Γ— Γ— Γ— Γ— Γ— Γ— = 3 Γ— 2 Γ— 6 Γ— 3 Γ— 2 = 9 Γ— 4 Γ— 6 = 36 Γ— 6 = 216 cm2.

Section C 11. Perimeter = 324 m

Let AB = 85 m AC = 154 mPerimeter = a + b + c 324 = 85 + 154 + c c = 324 – 239 c = 85 m

Area = s s a s b s c( )( )( )βˆ’ βˆ’ βˆ’

S = 3242

= 162 m

(a) Area = 162 162 85 162 154 162 85( )( )( )βˆ’ βˆ’ βˆ’

= 162 77 8 77Γ— Γ— Γ—

= 77 81 2 8Γ— Γ—

= 77 9 9 16Γ— Γ— = 77 Γ— 9 Γ— 4 = 2772 m2.

(b) To find AM

In βˆ†AMB and βˆ†CMB BM = BM [Common] AB = BC = 85 m  ∠AMB = ∠CBM = 90Β°β‡’ βˆ†AMB β‰… βˆ†CMB [RHS]β‡’ AM = MC [CPCT]

β‡’ AM = MC = 12AC =

12(154) = 77 m

In βˆ†AMB, by pythagoras theorem AB2 = AM2 + BM2

(85)2 = (77)2 + BM2

BM2 = (85)2 – (77)2

= 7225 – 5929 = 1296β‡’ BM = 36 m

12. Area of rhombus = 480 cm2

One of diagonal = 48 cm

(a) Area of rhombus = 12 1 2Γ— Γ—D D

480 = 12

48 2Γ— Γ—D

D2 = 480 248Γ—

D2 = 20 cm.

Now, AC = 48 cmand BD = 20 cm OA = 24 cm, OB = 10 cm

(b) In right βˆ†AOB, AB2 = OA2 + OB2

AB2 = (24)2 + (10)2

AB2 = 576 + 100

AB = 676 AB = 26 cm.(c) Perimeter = 4 Γ— Side = 4 Γ— 26 = 104 cm.

13. Diagonal AC = 42 cm

80

MATHEMATICS - 9DDITION LA APR CTICEA

In βˆ†ABC, we get; a = 34 cm b = 20 cm c = 42 cm

s = a b c+ +

2 =

34 42 202

+ + = 48 cm

Area of βˆ†ABC = s s a s b s c( )( )( )βˆ’ βˆ’ βˆ’

= 48 48 34 48 42 48 20( )( )( )βˆ’ βˆ’ βˆ’

= 48 14 6 28Γ— Γ— Γ— = 336 cm2

Area of parallelogram ABCD = 2 Γ— Area of βˆ†ABC = 2 Γ— 336 = 672 cm2.

14. Area of Rhombus = 12 1 2Γ— Γ—D D

Length of each side = 10 cmOne of its diagonal = 10 cm AC = 10 cmIn right βˆ†AOB, AB2 = OA2 + OB2

(10)2 = (5)2 +OB2

100 = 25 + OB2

OB2 = 100 – 25

OB = 75

OB = 8.6 cm BD = 8.6 + 8.6 BD = 17.2 cm.

Area of rhombus = 12

Γ— Γ—AC BD

= 12

10 17 2Γ— Γ— . = 17.2 Γ— 5 = 86 cm2.

15. In βˆ†ABD, AB = 13, AD = 5 and BD = 12

s = a b c+ +

2

s = 13 12 52

+ + = 302

= 15

Area = s s a s b s c( )( ) ( )βˆ’ βˆ’ βˆ’

= 15 15 13 15 12 15 5( )( )( )βˆ’ βˆ’ βˆ’

= 15 2 3 10Γ— Γ— Γ—

= 3 5 2 3 5 2Γ— Γ— Γ— Γ— Γ—

= 3 Γ— 5 Γ— 2 = 30 cm2

In βˆ†BCD, BC = 16, CD = 20 and BD = 12

s = a b c+ +2

= 16 20 12

2+ +

= 482

= 24

Area = 24 24 16 24 20 24 12( )( )( )βˆ’ βˆ’ βˆ’

= 24 8 4 12Γ— Γ— Γ—

= 12 2 4 2 4 12Γ— Γ— Γ— Γ— Γ—

= 12 Γ— 2 Γ— 4 = 48 Γ— 2 = 96 cm2

Area of Quadrilateral ABCD = Area of βˆ†ABD + Area of βˆ†BCD = 30 + 96 = 126 cm2

Section D 16. Area of trapezium =

12

Γ— (Sum of parallel sides) Γ— Height

= 12

25 10Γ— +( ) Γ— Height.

CE || DA, so ADCE is a parallelogram having AD || CE and CD || AE such that AD = 13 cm and CD = 10 m AE = 10 m CE = AD = 13 m BE = AB – AE = 25 – 10 = 15 mIn βˆ†BCE,

s = a b c+ +2

= 14 13 15

2+ +

= 422

= 21 m

Area of βˆ†BCE = s s a s b s c( )( )( )βˆ’ βˆ’ βˆ’

= 2121 14 21 13 21 15( )( )( )βˆ’ βˆ’ βˆ’

= 21 7 8 6Γ— Γ— Γ—

= 7 3 7 4 2 2 3Γ— Γ— Γ— Γ— Γ— Γ—

= 7 Γ— 2 Γ— 3 Γ— 2 = 42 Γ— 2 = 84 m2

Area of βˆ†BCE = 12

Γ— Γ—BE CL

84 = 12

15Γ— Γ—CL

CL = 16815

= 565

m

Height of trapezium = 565

m

∴ Area of trapezium

= 12

25 10565

Γ— + Γ—( )

= 12

35565

Γ— Γ—

= 7 Γ— 28 = 196 m2. 17. Perimeter of quadrilateral = 590 m Let a = 5x, b = 12x, c = 17x and d = 25x

Perimeter = a + b + c + d 590 = 59x + 12x + 17x + 25x 590 = 59x

81 81

12. Heron’s Form

ula

MATHEMATICS - 9DDITION LA APR CTICEA

x = 59059

β‡’ x = 10

a = 5 Γ— 10 = 50 b = 12 Γ— 10 = 120 c = 17 Γ— 10 = 170 d = 25 Γ— 10 = 250

s = a b c d+ + +

2 =

50 120 170 2502

+ + +

s = 5902 = 295 m

Area = ( )( )( ) ( )s a s b s c s dβˆ’ βˆ’ βˆ’ βˆ’

= ( )( )( )( )295 50 295 120 295 170 295 250βˆ’ βˆ’ βˆ’ βˆ’

= 245 175 125 45Γ— Γ— Γ—

= 7 35 25 7 5 25 5 9Γ— Γ— Γ— Γ— Γ— Γ— Γ—

= 7 Γ— 5 Γ— 5 Γ— 5 Γ— 3 35 = 2525 35 m2. 18. Base = 8 cm and height = 5.5 cm

Area of triangle = 12Γ— Γ—B H

= 12

8 5 5Γ— Γ— . = 22 cm2.

Worksheet-1I

Section A 1. AB = 4 cm and AC = 4 cm

Area of triangle = 12Γ— Γ—b h

= 12

4 4Γ— Γ— = 8 cm2.

2. Side of equilateral triangle = 4 3 cm

Area of equilateral triangle = 34

(Side)2

= 34

4 3 4 3Γ— Γ—

= 4 Γ— 3 3

= 12 3 cm2.

3. Area of equilateral triangle = 20 3 cm2

34

2a = 20 3

a2 = 20 Γ— 4

a = 80

a = 2 20

and its given that side is 8 cm, which is false.

4. s = AB+BC+AC

2 = 8 cm

Area = s s a s s c( ) ( ) ( )βˆ’ βˆ’ βˆ’6

= 8 8 6 8 6 8 4( )( ) ( )βˆ’ βˆ’ βˆ’

= 8 2 2 4Γ— Γ— Γ—

= 128 cm2

= 8 2 cm2

5. Area of equilateral triangle = 16 3 m2

34

2a = 16 3

a2 = 16 Γ— 4

a2 = 64 a = 8 m Perimeter = 3a = 3 Γ— 8 = 24 m.

Section B 6. Area of isosceles right-angled triangle = 200 cm2.

12

2a = 200

a2 = 200 Γ— 2 a2 = 400 a = 20 cm

Perimeter = 2a + 2 a

= 2 20 2Γ— + (20)

= 40 20 2+ 7. a = 70 cm, b = 80 cm, c = 90 cm

s = a b c+ +

2 =

70 80 902

+ + =

2402

= 120

Area = s s a s b s c( ) ( )( )βˆ’ βˆ’ βˆ’

= 120 120 70 120 80 120 90( )( )( )βˆ’ βˆ’ βˆ’

= 120 50 40 30Γ— Γ— Γ—

= 12 10 5 10 4 10 3 10Γ— Γ— Γ— Γ— Γ— Γ— Γ—

= 12 Γ— 10 Γ— 10 5 = 1200 5 = 1200 Γ— 2.23 = 2676 cm2.

8. Area of rhombus = 12 1 2Γ— Γ—D D

= 12

300 160Γ— Γ— = 24,000 m2

82

MATHEMATICS - 9DDITION LA APR CTICEA

Now, we have; OB = 150 m and OA = 80 mIn βˆ†AOB, by Pythagoras theorem, OA2 + OB2 = AB2

(150)2 + (80)2 = AB2

AC2 = 6400 + 22500 AC2 = 28900 AC = 170 m.

9. By Heron’s formula,

Area of triangle = s s a s b s c( )( )( )βˆ’ βˆ’ βˆ’

s = a b c+ +

2 =

41 40 92

+ + =

902

= 45

Area = 45 45 41 45 40 45 9( )( )( )βˆ’ βˆ’ βˆ’

= 45 4 5 36Γ— Γ— Γ— = 180 m2

= 1800000 cm2

Number of flower beds = 1800000900

cm2 = 2000.

10. Perimeter of isosceles triangle = 32 cm 2a + h = 32Let a = 3x and h = 2x 3x + 3x + 2x = 32 8x = 32 x = 4 a = 12 cm h = 8 cmSides of triangle are = 12 cm, 12 cm, and 8 cm

s = a b c+ +

2 =

12 12 82

+ + =

322

= 16 cm

Area of triangle = s s a s b s c( )( )( )βˆ’ βˆ’ βˆ’

= 16 16 12 16 12 16 8( )( )( )βˆ’ βˆ’ βˆ’

= 16 4 4 8Γ— Γ— Γ—

= 4 Γ— 4 Γ— 2 2 = 32 2 cm2.

11. Given sides are 6 cm, 8 cm and 10 cm

s = a b c+ +

2 =

6 8 102

+ + =

242

= 12

Area = s s a s b s c( )( )( )βˆ’ βˆ’ βˆ’

= 12 12 6 12 8 12 10( )( )( )βˆ’ βˆ’ βˆ’

= 12 6 4 2Γ— Γ— Γ— = 2 6 6 4 2Γ— Γ— Γ— Γ—

= 2 Γ— 6 Γ— 2 = 24 cm2

Cost of painting = 0.09 Γ— 24 = ` 216

12. Perimeter = 11 cm and Base = 5 cm

Perimeter = a + b + c a = b 11 = a + a + c 11 = 2a + 5 11 – 5 = 2a 6 = 2a

a = 3β‡’ a = b = 3

s = a b c+ + =

2112

Area = s s a s b s c( ) ( ) ( )βˆ’ βˆ’ βˆ’

= 112

112

3112

3112

5βˆ’

βˆ’

βˆ’

= 112

52

52

12

Γ— Γ— Γ— = 54

11 cm2

Section C 13. In βˆ†ABD,

a = 90, b = 50, c = 70

s = a b c+ +

2= 90 50 70

2+ +

s = 2102

= 105

Area = s s a s b s c( ) ( )( )βˆ’ βˆ’ βˆ’

= 105 105 90 105 50 105 70( )( )( )βˆ’ βˆ’ βˆ’

= 105 15 55 35Γ— Γ— Γ—

= 21 5 15 5 11 5 7Γ— Γ— Γ— Γ— Γ— Γ—

= 3 7 5 5 3 5 11 5 7Γ— Γ— Γ— Γ— Γ— Γ— Γ— Γ—

= 3 5 5 7 11Γ— Γ— Γ— β‡’ 525 11 cm2

In βˆ†BCD, a = 30, b = 60, c = 70

s = a b c+ +

2 =

30 60 702

+ + =

1602

= 80

Area = 80 80 30 80 60 80 70( )( )( )βˆ’ βˆ’ βˆ’

= 80 50 20 10Γ— Γ— Γ—

= 8 10 5 10 2 10 10Γ— Γ— Γ— Γ— Γ— Γ—

= 10 10 4 5Γ— Γ— = 400 5

Area of quadrilateral ABCD = Area of βˆ†ABD + Area of βˆ†BDC

= 525 11 + 400 5 cm2. 14. AB = 8 cm, BC = 6 cm

In βˆ†ABC,

By Pythagoras theorem, AC2 = AB2 + BC2

83 83

12. Heron’s Form

ula

MATHEMATICS - 9DDITION LA APR CTICEA

AC2 = (8)2 + (6)2

AC2 = 64 + 36

AC = 100

AC = 10 cmβ‡’ OA = 5 cmSimilarly, BD = 10 cm and OB = 5 cmNow in βˆ†OAB, OA = OB = 5 cm and AB = 8 cm

s = a b c+ +

2 =

5 5 82

+ + =

182

= 9

Area of triangle = s s a s b s c( )( )( )βˆ’ βˆ’ βˆ’

= 9 9 5 9 5 9 8( )( )( )βˆ’ βˆ’ βˆ’ = 9 4 4 1Γ— Γ— Γ—

= 3 Γ— 2 Γ— 2 = 12 cm2. 15. Area to be planted = Area of βˆ†ABC.

Here, a = 50 m, b = 80 m and c = 120 m

s = a b c+ +

2 =

50 80 1202

+ +

s = 2502 = 125 m

Area = s s a s b s c( )( )( )βˆ’ βˆ’ βˆ’

= 125 125 80 125 50 180 120( )( )( )βˆ’ βˆ’ βˆ’

= 125 75 45 60Γ— Γ— Γ—

= 25 5 25 3 5 9 4 3 5Γ— Γ— Γ— Γ— Γ— Γ— Γ— Γ—

= 25 Γ— 5 Γ— 3 Γ— 3 Γ— 2 5

= 2250 5 m2

Number of meters to be fenced = (50 + 80 + 120) – 3 =250 – 3 = 247 m Cost of fencing = ` 20 per metre = ` 20 Γ— 247 = ` 4940.

16. In βˆ†AOD,

By Pythagoras Theorem AD2 = OD2 + OA2

AD2 – AO2 = OD2

(30)2 – (24)2 = OA2

OA2 = 900 – 576 = 324

OA = 54 6Γ—

OA = 18 m

Area of βˆ†AOD = 12

24 18Γ— Γ—

= 12 Γ— 18 = 216 m2

∴ Area of rhombus = 4 Γ— βˆ†AOD = 4 Γ— 216 = 864 m2

∴ Grass area for 18 cows = 864 m2

Grass area for 1 cow = 86418

2

m = 48 m2.

17. Perimeter of rhombus = 4 Γ— side 32 = 4 Γ— side Side = 8 m

Diagonal of βˆ† = 10 m

Area = 12 1 2Γ— Γ—D D ;

D D12

22

2 2

+

= (Side)2

102 2

22

2

+

D = (8)2

1004

22

+D4

= 64

100 + D22 = 64 Γ— 4

D22 = 256 – 100

D22

= 156

D2 = 156 β‡’ D2 = 2 39 m

Area = 12

10 2 39Γ— Γ—

= 10 39 m2

= 10 Γ— 6.245 m2 = 62.45 m2

Area of both sides = 2 Γ— 62.45 m2 = 124.90 m2

Cost of painting = ` 124.90 Γ— 5 = ` 624.50.

Section D 18. Let a, b, c be the sides of given triangle and s be its perimeter.

∴ s = a b c+ +2

…(1)

The sides of new triangle are 2a, 2b, 2cLet sβ€² be its semi-perimeter

sβ€² = 2 2 2

2a b c+ +

= a + b + c = 2s [from equation (i)]

Area of triangle = s s a s b s c( )( )( )βˆ’ βˆ’ βˆ’ …(2)

Area of new βˆ†β€²β€ƒ= β€² β€² βˆ’ β€² βˆ’ β€² βˆ’s s a s b s c( )( )( )2 2 2

= 2 2 2 2 2 2 2s s a s b s c( )( )( )βˆ’ βˆ’ βˆ’ [using equation (1)]

= βˆ’ βˆ’ βˆ’16s s a s b s c( )( )( ) = 4βˆ† ∴ Increase in Area of βˆ†β€ƒ=β€ƒβˆ†β€²β€ƒβ€“β€ƒβˆ†β€ƒ= 4 βˆ†β€ƒβ€“ βˆ†β€ƒ= 3βˆ†

Percentage increase in Area =3

100βˆ†βˆ†Γ—

% = 300%

19. Let sides of triangular plot = 3x, 5x and 7x Perimeer = 300 m 3x + 5x + 7x = 300

84

MATHEMATICS - 9DDITION LA APR CTICEA

15x = 300 x = 20 m∴ sides are : a = 3x = 3 Γ— 20 = 60 m b = 5x = 5 Γ— 20 = 100 m c = 7x = 7 Γ— 20 = 140 m

s = a b c+ +

2=

60 100 1402

3002

+ + =

= 150 m

\ Area of triangle = s s a s b s c( )( )( )βˆ’ βˆ’ βˆ’

= 150 150 60 150 100 150 140( )( )( )βˆ’ βˆ’ βˆ’

= 150 90 50 10Γ— Γ— Γ—

= 1500 3 m2. 20. Perimeter of triangle = 50 cm

Let smaller side = x One side = x + 4 Third side = 2x – 6

Perimeter = a + b + c 50 = x + x + 4 + 2x – 6 50 + 2 = 4x 52 = 4x x = 13 Smaller side = 13 One side = 17\ Third side = 20

s = a b c+ +

2=13 17 20

2+ +

s = 502

= 25

\ Area = s s a s b s c( )( )( )βˆ’ βˆ’ βˆ’

= 25 25 13 25 17 25 20( )( )( )βˆ’ βˆ’ βˆ’

= 25 12 8 5Γ— Γ— Γ—

= 5 5 4 3 4 2 5Γ— Γ— Γ— Γ— Γ— Γ—

Area = 5 4 30Γ— = 20 30 cm2.

85 85

MATHEMATICS - 9DDITION LA APR CTICEA

Worksheet-1

Section A 1. Volume of cone =

13

2Ο€r h

r = 7 cm and l = 25 cm l2 = h2 + r2

h2 = l2 – r2

h2 = (25)2 – (7)2

h2 = 625 49βˆ’

h = 576 h = 24 cm

Volume = 13

227

7 7 24Γ— Γ— Γ— Γ—

V = 154 Γ— 8 = 1232 cm3. 2. Surface area of sphere = 154 cm2

4pr2 = 154

r2 = 154 722 4

Γ—Γ—

r2 = 154 722 4

Γ—Γ—

r2 = 494

r = 494

r = 72

cm.

3. T.S.A. of cube = 216 cm2

6a2 = 216

a2 = 2166

a = 36 a = 6 cm Volume = a3

= 6 Γ— 6 Γ— 6 = 216 cm3. 4. Height of cone = ?

Diameter = 30 m

Radius = D2

= 302

= 15 m

Slant height (l) = 25 m l2 = h2 + r2

h2 = l2 – r2

= (25)2 – (15)2 = 625 – 225 = 400

h = 400

Height = 20 m. 5. Volume of cube = 27a3

(side)3 = 27a3

side = 27 33 a

Surface Areas and Volumes13

Chapter

side = 3a T.S.A. of cube = 6a2

= 6(3a)2

= 6 Γ— 9a2 = 54a2. 6. Radius of cylinder = 14 cm

Height = 14 cm Volume of cylinder = pr2h

= 227

14 14 14Γ— Γ— Γ—

= 44 Γ— 196 = 8624 cm3. 7. Diameter of cone = 7 cm

Radius of cone = 72

cm

Height of cone = 12 cm

Volume of cone = 13

2Ο€r h

= 13

227

72

72

12Γ— Γ— Γ— Γ—

= 77 Γ— 2 = 154 cm3. 8. Radius of sphere = 5 cm

Volume of sphere = 43

3Ο€r

= 43

227

5 5 5Γ— Γ— Γ— Γ—

= 8821

125Γ— = 1100021

= 262 (approx.) cm3. 9. Base area of cylinder = 154 cm2, Height = 5 cm

pr2 = 154

r2 = 154722

Γ—

r = 7 cm Volume of cylinder = pr2h

= 227

7 7 5Γ— Γ— Γ—

= 154 Γ— 5 = 770 cm3

10. Edge of a cube = 11 cmS.A. of a cube = 4a2 = 4 Γ— 11 Γ— 11 = 4 Γ— 121 = 484 cm2

Section B 11. R = radius of balloon = 7 cm r = radius of pumped balloon = 14 cm

β‡’ S.A.of spherebefore inflated

S.A.of sphericalballoonafter inflatted

= 4

4

2

2ππR

r =

7 714 14

Γ—Γ—

= 14

Their ratio will be 1 : 4. 12. Volume of planks = L Γ— B Γ— H

= 200 Γ— 3 Γ— 4 = 2400 cm3

86

MATHEMATICS - 9DDITION LA APR CTICEA

Volume of wooden blocks = L Γ— B Γ— H = 600 Γ— 18 Γ— 99 = 10,69,200 cm3

Number of planks = Volume of woodenblocksVolume of eachplanks

= 10692002400

= 445.5 = 446 (approx.)

13. Circumference of cylindrical base = 2pr

132 = 2227

Γ— Γ— r

132 72 22

Γ—Γ—

= r

r = 21 cm Height = 25 cmVolume of cylinder = pr2h

= 227

21 21 25Γ— Γ— Γ—

= 66 Γ— 21 Γ— 25 = 34,650 cm3. 14. Height of cone = 15 cm

Volume = 1570 cm3

Volume of cone = 13

2Ο€r h

1570 = 13

3 14 152Γ— Γ— Γ—. r (Using p = 3.14)

r2 = 1570 33 14 15

Γ—Γ—.

r = 100 β‡’ r = 10 cm

Diameter = 2r = 2 Γ— 10 = 20 cm

15. Length = 15 cm, Breadth = 10 cm and Height = 20 cm Surface area = 2h(l + b) = 2 Γ— 20(15 + 10) = 40 Γ— 25 = 1000 cm2.

16. C.S.A. of cylinder = 4400 cm2

Circumference of base = 110 cm 2pr = 110

r = 110 722 2

Γ—Γ—

= 352

cm

C.S.A. = 4400 2prh = 4400

2227

352

Γ— Γ— Γ— h = 4400

Height = 4400 7 22 22 35

Γ— Γ—Γ— Γ—

Height = 40 cm. 17. Length = 12 cm, Breadth = 12 cm, Height = 12 cm

Volume of wooden block = L Γ— B Γ— H = 12 Γ— 12 Γ— 12 = 1728 cm3

Volume of cubes of side 3 cm = (Side)3

= 3 Γ— 3 Γ— 3 = 27 cm3

Number of cubes formed = Volume of woodencuboid

Volume of cubes

= 172827

= 64

18. Radius of shot-put = 4.9 cm

Volume of shot-put = 43

3Ο€r

= 43

227

4 9 4 9 4 9Γ— Γ— Γ— Γ—. . .

= 493 cm3.

19. Radius of conical tin = 30 cm Slant height = 50 cm l2 = h2 + r2

(50)2 = h2 + (30)2

2500 – 900 = h2

1600 = h2

h = 40 cm

Volume of conical tin = 13

2Ο€r h

= 13

3 14 30 30 40Γ— Γ— Γ— Γ—.

= 314 Γ— 120 = 37680 cm3. 20. Length = 2.5 m

Height = 10 mCapacity of tank = 50,000 lVolume of cuboid = 50,000 l L Γ— B Γ— H = 50,000 l

2.5 Γ— B Γ— 10 = 50 0001

1000, Γ— m3

B = 50000

1000 2 5 10Γ— Γ—.

Breadth (B) = 2 m.

Section C 21. Number of people = 50000

75 L water requires for 1 people.75 Γ— 5000 L water requires for 5000 people375000 L water requires for 5000 people.Volume of water in overhead tank = 40 Γ— 25 Γ— 15 m3

= 40 Γ— 25 Γ— 15 Γ— 1000 l

So number of days = 40 25 15 1000

375000Γ— Γ— Γ—

= 40 days.

22. Volume of cone = 100 cm3

Height = 12 cm

Volume of cone = 13

2Ο€r h

100 = 13

227

122Γ— Γ— Γ—r

r2 = 100 722 4

Γ—Γ—

= 70088

r = 70088

= 2.8 cm

C.S.A. = 227

2 8Γ— Γ—. l = prl

l = h r2 2+ l =

( ) ( . )12 2 82 2+ l = 144 7 84+ .

l = 151 84.

= 12.32 cm

= 12 cm (approx)

C.S.A = prl

= 227

2 8 12Γ— Γ—. = 105.6 cm2

87 87

13. Su

rface Areas an

d V

olum

es

MATHEMATICS - 9DDITION LA APR CTICEA

23. Inner radius (r1) of cylindrical pipe = 242

= 12 cm

Outer radius (r2) of cylindrical pipe = 282

= 14 cm

Height of pipe = Length of pipe = 35 cm

Volume of pipe = Ο€( )r r h22

12βˆ’

= 22714 12 352 2( )βˆ’ Γ—

= 110 Γ— 52 = 5720 cm3

Mass of 1 cm3 wood = 0.6 gmMass of 5720 cm3 wood = (5720 Γ— 0.6)g = 3432 g = 3.432 kg

24. Here, cube of side 12 cm is divided into 8 cubes of side a cm.Given that their volumes are equal.Volume of big cube side 12 cm = Volume of 8 cubes of side a cm (Side of big cube)3 = 8 Γ— (Side of small cube)3

(12)3 = 8 Γ— a3

12 12 128

Γ— Γ— = a3

a3 = 12 12 122 2 2Γ— Γ—Γ— Γ—

a3 = 6 Γ— 6 Γ— 6 a = 6 cm∴ Side of small cube = 6 cm

Ratio of their S.A. = S.A.of cube of side12 cmS.A.of cube of side 6 cm

= 6(Side of big cube)

6(Side of small cube)

2

2

= 6 12 126 6 6Γ— Γ—Γ— Γ—

= 41

Ratio of S.A. = 4 : 1.

25. Number of tiles required = S.A. for tiles to coverArea of each tile

We put tiles on 5 faces Area of 1 face = (Side)2

Area of 5 faces = 5 Γ— Area of 1 face = 5 Γ— (Side)2

= 5 Γ— (1.5)2

= 5 Γ— (1.5 Γ— 100 cm)2

= 5 Γ— (150)2

= 5 Γ— 150 Γ— 150Now, we find area of tile Area of 1 tile = Side Γ— Side = 25 Γ— 25 cm2

Number of tiles required = Area of 5 facesArea of 1tile

= 5 150 15025 25Γ— Γ—

Γ—

= 30 Γ— 6 = 180Given that cost of 1 dozen tiles = ` 360∴ Cost of 12 tiles = ` 360

Cost of 1 tile = ` 36012

Cost of 180 tiles = ` 36012

180Γ—

= ` 30 Γ— 180 = ` 5400. 26. Given r = 7 m

h = 24 m

∴ l = r h2 2+

= 7 242 2+ = 49 576+

= 625 = 25 mC.S.A. of cone = prl

= 227

7 25Γ— Γ— = 22 Γ— 25 = 550 m2

Area of cloth used = 550 m2

Given, width of cloth = 5 m

Length of the cloth used = AreaWidth

= 5505

= 110 m.

27. Cost of painting the four walls = 15000Cost of painting per square meter = ` 10

Area of four walls painted = 1500010

= 1500 m2

Area of four walls painted = 2h( l + b) = 250 h 1500 m2 = 250 h

β‡’ 1500250

= h

β‡’ h = 6 m. 28. Three cubes are joined to end to end then cuboid is formed

Its length will form is 12 + 12 + 12 = 36 cm Breadth = 12 cm, height = 12 cm Volume of cuboid = l Γ— b Γ— h = 36 Γ— 12 Γ— 12 = 5184 cm3.

29. Height of roller = 1.5 m

Diameter of roller = 84 cm⇒ Radius of roller = 42 cm = 0.42 m CSA of roller = 2prh

= 2227

42100

1 5Γ— Γ— Γ— .

= 3.96 m2

β‡’ Area covered in one revolution = 3.96 m2

β‡’ Area covered in 100 revolutions = 100 Γ— 3.96 = 396 m2

Cost of leveling the ground = ` 39650100

Γ— = ` 198

30. Diameter of ball = 4.2 cm⇒ Radius of ball (r) = 2.1 cm

Volume of ball = 43

3Ο€r

= 43

227

2 1 2 12 1Γ— Γ— Γ—. . .

= 38.81 cm3

Section D 31. L = 20 m, B = 15 m, H = 8 m

C.S.A. of hall = 2H (L + B) = 2 Γ— 8 (15 + 20) = 16 Γ— 35 = 560 m2

Cost of painting = 15 Γ— 560 = ` 8400 32. Water flowing in river in 1 hour = 2 km

Water flowing in river in 1 hour = 2000 mWater flowing in river in 60 minute = 2000 m

Water flowing in river in 1 minute = 200060

m = 1003

mNow, river is in shape of cuboid

Length = 1003

m

Breadth = 40 m

88

MATHEMATICS - 9DDITION LA APR CTICEA

Height = 3 m∴ Volume of water falling into sea = Volume of cuboid = Length Γ— Breadth Γ— Height

= 1003

40 3Γ— Γ—

m3 = 4000 m3

33. Inner radius = r1 = 42

= 2 cm

Height = 77 cm C.S.A. of pipe = 2pr1h

= 2227

2 77Γ— Γ— Γ— = 968 cm2

Outer radius = r2 = 4 42.

= 2.2

C.S.A. of pipe = 2pr2h

= 2227

2 2 77Γ— Γ— Γ—. = 1064.8 cm2

T.S.A. = C.S.A. of inner cylinder + C.S.A. of outer cylinder + 2 Γ— Area of base Area of base = pr2

2 – pr12

= 227

2 2 22 2[( . ) ( ) ]βˆ’

= 2274 84 4( . )βˆ’ =

227

0 84Γ— . = 2.64 cm2

T.S.A = 968 + 1064.8 + 2 Γ— 2.64 = 2032.8 + 5.28 = 2038.08 cm2.

34.

Circumference (c) = 17.6 cm 2pr = 17.6

r = 17 6 72 22. Γ—Γ— =

1232440

r = 1232440

= 2.8 m

r = 280 cmSurface area of hemispherical dome = 2pr2

= 2 Γ— 227

Γ— 280 Γ— 280

= 44 Γ— 123207

= 1760 cm2

Cost of painting = 5 1760100Γ—

= ` 88

35. T.S.A. of cylinder = 6512 cm2

Circumference of base = 88 cm Circumference = 2pr

88 = 2227

Γ— Γ— r

r = 88 72 22

Γ—Γ—

r = 14 cm T.S.A. = 2pr (h + r)

6512 = 2227

14 14Γ— Γ— +( )h

6512 = 1232 + 88 h 6512 – 1232 = 88 h 5280 = 88 h

h = 528088

h = 60 cm Volume of cylinder = pr2h

= 227

14 14 60Γ— Γ— Γ— = 44 Γ— 14 Γ— 60

= 36960 cm2.

36. Volume of hollow hemisphere = 23

3 3Ο€( )R rβˆ’A.T.Q.,

Volume of bowl = 23

4 5 43 3Ο€ [( . ) ( ) ]βˆ’

= 23

4 5 4 4 5 4 5 4 42 2Ο€ [( . )[( . ) ( . ) ( ) ]βˆ’ + Γ— +

= 23

0 5 54 25Ο€ [( . . )]Γ—

= 23

3 14 27 125Γ— Γ—. . = 56.78 cm3

37. Given height = 1 mand capacity of cylindrical vessel = Volume of cylinder = 15.4 l

= 15 41

10003. Γ— m

= 1541

100003Γ— m =

15410000

3m

Volume of cylinder = pr2h

15410000

= 227

12Γ— Γ—r

r2 = 15410000

722

Γ—

r2 = 49

10000

r = 4910000

r = 7100

m

r = 0.07 m T.S.A. = 2pr (r + h)

= 2227

7100

Γ— Γ—7100

1+

= 2227

7100

Γ— Γ—107100

= 441100

107100

Γ— Γ— = 0.4708 m2.

Worksheet-II

Section A 1. Radius =

r2

, Slant height = 2 l

T.S.A. of cone = pr (l + r)

= 227 2

22

Γ— +

rl

r=

117

22

r lr+

= 117

42

rl r+

= 1114

4r l r( )+

89 89

13. Su

rface Areas an

d V

olum

es

MATHEMATICS - 9DDITION LA APR CTICEA

2. Volume of cylinder = pr2hLet original radius = rand height = hWhen radius is doubled then it will be 2r. Volume of cylinder = p(2r)2h = p(4r2)h = 4pr2h.Volume becomes 4 times

3. Volume of cube = a3

Volume of sphere = 43

3Ο€r

According to the question, a = 2r

Volume of sphere : Volume of cube

β‡’ 43

3 3Ο€r a:

β‡’ 43

23 3Ο€r r: ( )

β‡’ 43

83 3Ο€r r:

β‡’ 43

227

8Γ— :

β‡’ 8821

8:

β‡’ 1121

1:

β‡’ 11 : 21. 4. Radius is doubled i.e., 2r

S.A. of sphere = 4pr2 = 4p(2r)2= 4p(4r2) = 16pr2

5. Volume of sphere = 43

3Ο€r

Volume of hemisphere = 23

3Ο€r

If their radius are same.Volume of sphere : Volume of hemisphere

43

23

3 3Ο€ Ο€r r:

43

23:

4 : 2 2 : 1

6. Diameter of cone = 30 mRadius of cone = 15 mSlant height = 25 m l2 = h2 + r2

h2 = l2 + r2

h2 = (25)2 – (15)2

h = 625 225βˆ’

h = 400 h = 20 m.

7. Volume of cone = 48p cm3

Height of cone = 9 cm

Volume = 13

2Ο€r h

48p Γ— 3 = pr2(9)

48 39Γ—

= r2

r = 16 r = 4

Diameter = 2r = 2 Γ— 4 = 8 cm.

8. Diameter of sphere is equal to length of cube, 6 cmRadius of sphere = 3 cm

Volume of sphere = 43

3Ο€r

= 43

227

3 3 3Γ— Γ— Γ— Γ—

= 36 22

7Γ—

= 7927

= 113.14 cm3

9. Sum of all edges of cube = 36 cm

Length of one edge = 3612

= 3 cm

Volume of a cube = a3 = (3)3

= 27 cm3.

10. Radius of a sphere = 13

r

Volume of sphere = 43

3Ο€r = 43

13

3

Ο€ r

= 43

227

13

13

13

Γ— Γ— Γ— Γ—r r r

= 88567

3r

Section B 11. Side of each cube = 5 cm

Length of cuboid = 10 cmBreadth of cuboid = 5 cmHeight of cuboid = 5 cm T.S.A. = 2(lb + bh + hl) = 2(10 Γ— 5 + 5 Γ— 5 + 5 Γ— 10) = 2(50 + 25 + 50) = 2 Γ— 125 = 250 cm2

12. T.S.A. of cylinder = 1628 cm2

r + h = 37 cm T.S.A. = 2pr(r + h)

1628 = 2 Γ— 227

Γ— r (37)

1628 744 37

Γ—Γ—

= r β‡’ r = 7

r + h = 37 7 + h = 37 h = 30 cm

13. Volume of cone = 9856 cm2

Radius = 14 cm

Volume = 13pr2h

9856 Γ— 3 = 227

Γ— 14 Γ— 14 Γ— h

9856 322 2 14

Γ—Γ— Γ—

= h

h = 16 Γ— 3 h = 48 cm

90

MATHEMATICS - 9DDITION LA APR CTICEA

14. Height of conical pit = 12 m

Radius = 3 52.

= 1.75 m

Capacity of pit = Volume of cone

= 13pr2h

= 13

227

Γ— Γ— (1.75)2 Γ— 12

(1 m3 = 100 l, 1 m3 = 1 Kl)

= 38.5 kilolitres 15. Radius of sphere = 0.63 m

Volume of sphere = 43

pr3

= 43

227

Γ— Γ— 0.63 Γ— 0.63 Γ— 0.63

= 1.0478 m3

Section C 16. Length of a cylindrical roller = 2.5 m

Radius of a cylindrical roller = 1.75 mTotal area rolled by it = 5500 m2

\ Area in one revolution = C.S.A. of cylinder = 2prh

= 2 Γ— 227

Γ— 1.75 Γ— 2.5

= 27.5 m2

No. of revolutions = 550027 5.

= 200

17. Diameter, d = 7 cm

Radius, r = 72

cm

Height, h = 12 cm\ V = pr2h

= 227

72

72

12Γ— Γ— Γ—

= 77 Γ— 6 = 462 cm3

Total milk for 1600 students = 462 Γ— 1600 = 739200 cm2

= 7392001000

litres

= 739.2 litres

18. Diameter = 9 m, Radius = 92

= 4.5 m, Height = 3.5 m

Volume of cone = 13pr2h

= 13

227

Γ— Γ— 4.5 Γ— 4.5 Γ— 3.5 = 74.25 m3

l = h r2 2+

= ( . ) ( . )4 5 3 52 2+

= 20 25 12 25. .+

= 32 5. = 5.7 m C.S.A. of cone = prl

= 227

Γ— 4.5 Γ— 5.7 = 80.614 m2

Area of canvas required = 80.625 m2

19. Height of solid = 8 cmRadius of solid = 6 cm

Volume of solid = 13pr2h

= 13

227

6 6 8Γ— Γ— Γ— Γ— = 301.71 cm3

20. Diameter = 8.4 cm

Radius = 8 42.

= 4.2 cm

Volume of sphere = 43

3Ο€r

= 43

227

4 2 4 2 4 2Γ— Γ— Γ— Γ—. . .

= 310.464 cm3

Section D 21. Let radius and height of cone be 3x and 9x.

Volume of cone = 8748p cm3

Volume = 13pr2h

8748 = 13p (3x)2(9x)

8748 39 9

Γ—Γ—

= x3

x3 = 108 Γ— 3 x3 = 324

x = 2 2 3 3 3 33 Γ— Γ— Γ— Γ— Γ—

x = 3 123

Radius = 3x

= 3 Γ— 3 123 = 9 123

Height = 9x

= 9 Γ— 3 123 = 27 123

22. Thickness = 2 cm Outer diameter = 16 cm Outer radius (R) = 8 cm As outer radius = Inner radius + thickness\ Inner radius = Outer radius – thickness = 8 – 2 = 6 cm\ Volume of iron used = Volume of outer cylinder – Volume of inner cylinder = pR2h – pr2h

= 227

Γ— 100 (R2 – r2)

= 227

Γ— 100 (64 – 36)

= 227

Γ— 100 Γ— 28 = 8800 cm3

23. Length = 2b, Height = 3 m Area of four walls = 108 m2

β‡’ 2h(l + b) = 108β‡’ 2(2b + b) Γ— 3 = 108β‡’ 18b = 108

91 91

13. Su

rface Areas an

d V

olum

es

MATHEMATICS - 9DDITION LA APR CTICEA

⇒ b = 6 m\ l = 2b⇒ l = 2 × 6 = 12 m\ Volume of cold storage = l × b × h = 12 × 6 × 3 = 216 m3

24. Radius = r, Surface Area = S

Volume of 27 sphere of radius r = Volume of big sphere of radius r1

2743

3Ο€r

=

43p(r1)3

27r3 = (rβ€²)3

(rβ€²)3 = (3r)3

rβ€² = 3r

SSβ€²

= S.A.of sphere withradiusS.A.of sphere withradius

rrβ€²

= 44

2

2

ππ

rr( )β€²

= rr

2

23( ) =

rr

2

29

= 19

β‡’ 1 : 9

25. Water used by total population in a day = 3200 Γ— 125 l = 4,00,000 lWater in the tank = 30 m Γ— 10 m Γ— 8 m = 2400 m3

= 2400 Γ— 1000 l = 24,00,000 l

No. of days = 24 00 0004 00 000, ,, ,

No. of days = 244

= 6.

92 92

MATHEMATICS - 9DDITION LA APR CTICEA

Worksheet-1

Section A 1. Maximum value = 70

Range = Maximum value

2702

= = 35

Range = Maximum value – Minimum value 35 = 70 – Minimum value Minimum value = 35.

2. Let upper limit = x and lower limit = y

Now, x y+

2 = 42

β‡’ x + y = 84 ...(1)Class size = 10β‡’ x – y = 10 ...(2)Adding eq. (1) and (2) 2x = 94 x = 47 and y = 37 Lower limit = 37 and upper limit = 47.

3. Mean = 17According to question,

10 12 16 20 26

6+ + + + +P

= 17

84 + P = 17 Γ— 6 P = 102 – 84 P = 18

4. Class mark = Upper limit Lower limit+

2

= 100 120

2+

= 2202

= 110

5. First five natural numbers are 1, 2, 3, 4 and 5.

Mean = Sumof all observationsTotalno.of observations

= 1 2 3 4 5

5+ + + +

= 155

= 3.

Mean = 3. 6. Given data: 5, 8, 4, 5, 5, 8, 4, 7, 8, x

Mode of data = 5 Mode = The value in a set of data that appears most often i.e. = 5 Set of data = 5, 8, 4, 5, 5, 8, 4, 7, 8, x5 and 8 both are repeating thrice.But it is given that mode = 5β‡’ x = 5

Statistics14Chapter

93 93

14. Statistics

MATHEMATICS - 9DDITION LA APR CTICEA

Section B 7. Mean of 10 numbers = 20

The sum of 10 numbers = 10 Γ— 20 = 200If 5 is subtracted from every number = 200 – (10 Γ— 5) = 200 – 50 = 150

So, the new mean = 15010

= 15.

8. Class size = upper limit – lower limit = 10i.e., lower class limit = 104upper class limit = 114

9. The frequency distribution table for the data given is:

No. of Heads No. of Frequency

0123

6994

Total 28

10. Given points scored by a team:17, 2, 7, 27, 25, 5, 14, 18, 10, 24, 48, 10, 8, 7, 10, 28.Arrange in ascending order2, 5, 7, 7, 8, 10, 10, 10, 14, 17, 18, 24, 25, 27, 28, 48. Value of n = even No. of terms = 16

β‡’ Value of 12

n n2 2

1

+ +

th th

term term will be the median.

= 12

162

162

1th th

term term+ +

= 12

[8th term + 9th term]

= 1210 14[ ]+

= 12

24Γ— = 12

Median = 12.Mode is the maximum no. of repeating terms in a data, and 10 is repeating thrice. Mode = 10.

Section C 11. Given data: 8, 15, 10, 12, 20, 13, 7, 25, 15, 20, 20, 9, 20, 25, 15

Mean = Sumof all observationsTotal no.of observations

= 8 15 10 12 20 13 7 25 15 20 20 9 20 25 15

15+ + + + + + + + + + + + + +

= 23415

= 15.6

Mean = 15.6Arrange in ascending order7, 8, 9, 10, 12, 13, 15, 15, 15, 20, 20, 20, 20, 25, 25.Since n = 15

So median = Value of n +

1

2

th

term

= 15 12+

th

term

94

MATHEMATICS - 9DDITION LA APR CTICEA

= 162

th

term

= 8th term = 15 Median = 15Mode is the maxium no. of repeating term, i.e., 20 is repeating four times. Mode = 20

12.

First we draw a histogram from the given data and then join the mid-points of the top of the rectangles. Complete the polygon by joining the mid-points of first and last class intervals to the mid-points.

13. Mean of following distribution is 50.

x y xy

1030507090

175a + 3

327a – 11

19

17030(5a + 3)

1600(7a – 11) 70

1710

Mean = Sumof all observationsNo.of observations

Mean = ΣΣx ffi i

i

50 = 170 1600 1710 150 90 490 770

60 12+ + + + + βˆ’

+a aa

50 (60 + 12a) = 640a + 2800 3000 + 600a = 640a + 2800 3000 – 2800 = 640a – 600a 200 = 40a

a = 20040 β‡’ a = 5

Frequency of 30 = 5(5) + 3 = 25 + 3 = 28Frequency of 70 = 7(5) – 11 = 35 – 11 = 24.

14. No. of students = 100 Initial mean = 40

Mean = Sumof all observationsNo.of observations

95 95

14. Statistics

MATHEMATICS - 9DDITION LA APR CTICEA

Sum of observations = Mean Γ— No. of students = 40 Γ— 100 = 4000Total mistake done = (88 + 92 + 81) – (08 + 29 + 18) = 261 – 35 = 206∴ Correct sum of observations = 4000 + 206 = 4206

∴ Correct mean = 4206100

= 42.06

15. Median of following arranged observation = 241.4, 18, x + 2, x + 4, 30, 34Since n = 6, i.e., even

Median = 12 2 2

1n n

+ +

th th

term

Median = 12

62

62

1

+ +

th th

term

24 = 12

[3rd term + 4th term]

24 Γ— 2 = (x + 2 + x + 4] 48 = 2x + 6 2x = 48 – 6 2x = 42 x = 21 x + 2 = 21 + 2 = 23 x + 4 = 21 + 4 = 25Following data will be14, 18, 23, 25, 30, 34

Mean = Sumofall observationsNo.of observations

= 14 18 23 25 30 34

6+ + + + +

= 1446

= 24

Section D 16. Let the total mark of boys = b

Let the total mark of girls = g Total no. of boys = x Total no, of girls = y

Mean marks of boys = bx

= 70

Mean marks of girls = gx

= 73

Mean marks of total students = ( )( )b gx y

++

= 71

b + g = 71 (x + y) b + g = 71x + 71yNow b = 70x and g = 73yβ‡’ (70x + 73y) = 71x + 71yβ‡’ – x = –2yβ‡’ x = –2y

β‡’ xy

= 21

β‡’ Ratio will be 2 : 1

96

MATHEMATICS - 9DDITION LA APR CTICEA

17. Draw a histogram for the given table of marks scored by 70 students of class IX.

5

10

15

20

XXοΏ½YοΏ½

Y

O

25

30 40 50 60 70 80 90

No

.o

fstu

den

ts

Marks obtained

30

35

40

45

50

55

60

100

65

18. Mean of five no. = 27So, sum of 5 numbers = 27 Γ— 5 = 135Let excluding number = xsum of 4 numbers = 135 – x

Mean = Sumof numbers4

4135

4= βˆ’ x

β‡’ 27 – 2 = 1354βˆ’ x

β‡’ 25 Γ— 4 = 135 – x 100 – 135 = –x – x = – 35 x = 35Hence excluded number = 35

Worksheet-1I

Section A 1. Mid value = 10

Width = 6

Lower limit = mid value of class – Width2

= 10 – 62

= 10 – 3 = 7 2. The child is incorrect, because while finding median, the data should be first arranged in ascending order. 3, 5, 14, 18, 20. There are 5 terms

so median = mid-term which is 14. 3. It is not correct. Because the difference between two consecutive class marks should be equal to the class size. Here, difference between

two consecutive marks is 0.1 and class size of 1.55 – 1.73 is 0.18, which are not equal. 4. It is not correct. Because in a histogram, the area of each rectangle is proportional to the corresponding frequency of its class.

Section B 5. Mean of p, q, r, s and t is 28.

Mean of p, r and t = 24

( )p q r s t+ + + +

5 = 28

β‡’ p + q + r + s + t = 140 ...(1)

p r t+ +

3 = 24

p + r + t = 72 ...(2)

97 97

14. Statistics

MATHEMATICS - 9DDITION LA APR CTICEA

Substitute eq. (2) in eq. (1) p + q + r + s + t = 140 72 + q + s = 140 q + s = 140 – 72 q + s = 68

Mean of q and s = q s+

2

= 682

= 34.

6. Factors of 24 are 1, 2, 3, 4, 6, 8, 12, and 24.

Mean = Sumof all observationsTotalno.of observations

= 1 2 3 4 5 6 8 12 24

8+ + + + + + + +

= 608

= 7.5

7. Given mean of 100 observations = 50 Sum of observations = Mean Γ— No. of observations. = 50 Γ— 100 = 5000So we need to add this 100 to the sum of the observations to get the correct sum of observations.Hence correct sum of observations = 5000 + 100 = 5100

Correct mean = 5100100

= 51.

8. Class Size = 42 – 37 = 5Class limit are 35 – 39, 40 – 44, 45 – 49, 50 – 54, 55 – 59

Section C 9. Arranged data is 6, 14, 15, 17, x + 1, 2x – 13, 30, 32, 34, 43.

Total no. of observations = 10Here, 10 is an even number.

∴ Median will be the mean of 102

102

1

+

th th

and observations.

Median = 5 6

2thobservation thobservations+

24 = x x+ + βˆ’1 2 13

2

48 = 3x – 12 48 + 12 = 3x 60 = 3x

x = 603

x = 20

10. Mean of data = Sumof all observationsNo.of observations

β‡’ = 41 39 43 52 46 62 54 40 96 52 98 40 42 52 60

15+ + + + + + + + + + + + + +

β‡’ = 81715

β‡’ = 54.47MedianArranging in Ascending order, 39, 40, 40, 41, 42, 43, 46, 52, 52, 52, 54, 60, 62, 96, 98 No. of obs. = n = 15 (odd number)

Median = n +

1

2

th

observation

= 15 12+

th

observation

98

MATHEMATICS - 9DDITION LA APR CTICEA

= 162

th

observation = 8th observation = 52

Mode = 52 11.

Salary (xi) No. of workers (fi) fixi

300040005000600070008000900010000

16121086431

4800048000500004800042000320002700010000

Total 60 305000

Mean = ΣΣx ffi i

i

= 30500060

( x ) = 305006

= 5083.33

12.

xi fi xifi

4681012

4814113

164811211036

Total 40 322

Mean = ΣΣx ffi i

i

= 32240

= 8.05

Section D 13.

99 99

14. Statistics

MATHEMATICS - 9DDITION LA APR CTICEA

14. Mean of distribution = 1.46

xi fi xifi

012345

46f1f225105

0f12f2754025

Total 200 140 + 2f2 + f1 46 + f1 + f2 + 25+10 + 5 = 200 f1 + f2 = 114 ...(1)

Mean = ΣΣx ffi i

i

1.46 = 140 286

2 1

1 2

+ ++ +

f ff f

1.46 = 140 2 286 114

2 1+ ++

f f

1.46 Γ— 200 = 140 + f1 + 2f2 292 – 140 = f1 + 2f2 f1 + 2f2 = 152 ...(2)Subtracting eq. (1) from (2), we get

f1 – f1 + 2f2 – f2 = 152 – 114 f2 = 38Put in (1), we get

f1 + f2 = 114 f1 + 38 = 114 f1 = 114 – 38 f1 = 76β‡’ f1 = 76 and f2 = 38.

15.

Speed (km/h) 30–40 40–50 50–60 60–70 70–80 80–90 90–100

No. of Cars 3 6 25 65 50 28 14

Here’s a histogram of a given data:

100 100

MATHEMATICS - 9DDITION LA APR CTICEA

Worksheet-1

Section A 1. Events with probability 1. a. Sun will rise in the east, this has probability 1 because its a

sure event as sun will only rise in the east its definitely true and as its true, it has probability 1.

Events with probability 0. b. Sun will rise in the west, this has probability 0 because its

an impossible event. Sun will only rise in the east whatever the reason might be, and as its false and will never happen, it has probability 0.

2. P(hitting) = 420

15

=

P(not hitting) = 1 – 420

620

45

= 1 =

3. P(Event) = 45100

920

=

4. P(E) = x2

P(not E) = 1 – P(E)

23

= 1 – x2

23

11

βˆ’ = βˆ’ x2

2 33βˆ’

= βˆ’ x2

βˆ’ 13

= βˆ’ x2

x = 23

5. P(getting even no.) = 36

12

=

[Since even nos. are 2, 4, 6]

6. P(getting tail) = 44100

P(getting heads) = 100 44100

βˆ’

= 56100

= 0.56

7. Total no. of outcomes = 6 [1, 2, 3, 4, 5, 6]No. of favourable outcomes = 2[3, 6]

P(getting multiple of 3) = 26

= 13

8. No, the value can never be greater than 1. If the probability is 1 than it means that an event is a sure event. The probability of an event can be between 0 and 1.

Probability15Chapter

9. No, when we calculate the experimented probability, the formula is:

= No.of times anevent occured

No.of trials

Since neither of those values can be negative, there is no way for the result to be negative.

10. Total no. of outcomes = 1, 2, 3, 4, 5, 6 No. greater than 4 = 5, 6

P (No. greater than 4) = 26

= 13

Section B 11. Total students = 80

a. P(getting commerce/humanities) = 21 1280+

= 3380

b. P(getting science stream) = 29 1880+

= 4780

12. Total no. of outcomes = 250Total no. of outcomes of getting odd no. = 35 + 50 + 53 = 138

P(getting odd no.) = 138250

= 69125

13. Total no. of outcomes = 350

P(getting like) = 147350

P(getting dislike) = 203350

14. Total outcomes = 80No. of outcomes less than 900 hours = 10 + 12 + 23 = 45

P(getting life less than 900 hrs) = 4580

= 916

15. Total no. of outcomes = 500No. of outcomes more than one tail = 135 + 85 = 220

P(getting more than one tail) = 220500

= 2250

1125

=

Section C 16. i. P(getting more than 70%) =

No.of testsTotalno.of tests

= 36

12

= = 0.5

ii. P(getting less than 70%) = 36

12

= = 0.5

iii. P(at least 60% marks) = 56

= 0.83

17. Total no. of outcomes = 200

a. P(at least one tail) = 82 58 23

200163200

+ + =

b. P(at most one tail) = 82 37200

119200

+ =

18. Total no. of outcomes = 34

a. P(getting a green ball) = 934

101 101

15. Prob

ability

MATHEMATICS - 9DDITION LA APR CTICEA

b. P(getting a yellow ball) = 534

c. P(getting white ball) = 034

0=

Section D 19. a. Total no. of outcomes = 149

P(like pears) 13-23 years = 49149

b. Total no. of outcomes = 103

P(like Medimix) above 35 years = 25103

c. Total no. of outcomes = 136

P(like dove) 5-12 years = 53136

20. Total no. of outcomes = 50

a. P(getting French) = 1250

625

=

b. P(doesn’t opt Japanese) = 50 850

4250

2125

βˆ’ = =

c. P(either Sanskrit or German) = 14 1050

2450

1225

+ = =

Worksheet-II

Section A 1. The probability of an event cannot happen is 0. Such as event is

called impossible event. 2. Total no. of outcomes = 6[1, 2, 3, 4, 5, 6] Therefore 6 total outcomes for a die thrown. 3. Total cards in a pack = 52

Total black cards = 26

P (black cards) = 2652

12

=

4. Total no. of outcomes = 6

P(getting less than 7) = 66

= 1

5. Total no. of outcomes = 350 Women not working = 132 Women working = 350 – 132 = 218

P(not working women) = 218350

= 0.623

Section B 6. Total no. of outcomes = 36

P(getting sum of 11) = 236

118

=

7. Total no. of outcomes = 4

P(getting at most one head) = 34

8. Total no. of outcomes = 432 Nuclear families = 329Joint family = 432 – 329 = 103

P(lives in joint family) = 103432

9. Total no. of outcomes = 350

P(at least one head) = 85 102350+

= 187350

Section C 10. Total no. of outcomes = 20

a. P(A) = 420

= 15

b. P(B) = 520

= 14

c. P(AB) = 520

= 14

d. P(O) = 620

= 310

11. Total bags = 10 No. of bags contain less than 10 kg flour

i. P(less than 10 kg flour) = 410

= 25

ii. P(exactly 10 kg flour) = 210

= 15

iii. P(more than 10 kg flour) = 410

= 25

12. Total no. of outcomes = 45

a. P(weight lies in 50-60 kg) = 9 345+

= 1245

= 415

b. P(weight lies 30-50 kg) = 8 16 16

45+ +

= 4045

= 89

Section D 13. Total no. of outcomes = 700

a. P(no. defective bulbs) = 400700

= 47

b. P(defective bulb from 2 to 6) = 48 41 18 8 3

700+ + + +

= 118700

= 59350

c. P(defective bulb less than 4) = 400 180 48 41

700+ + +

= 669700

14. Total no. of outcomes = 200

a. P(No. defective part) = 50200

= 520

= 14

b. P(at least one defective part)

= 32200

850

425

= = = 0.16

c. P(not more than 5 defective part)

= 50 32 22 18 12 12

200+ + + + +

= 146200

= 73100

= 0.73

102

MATHEMATICS - 9DDITION LA APR CTICEA

15. Total no. of outcomes = 3000

P(20-35 years) = 170 80 30 20

3000+ + +

= 3003000

= 110

P(35-50 years) = 125 45 20 10

3000+ + +

= 2003000

= 230

= 115

P(50-65 years) = 250 90 40 20

3000+ + +

= 4003000

= 430

= 215

.